You are on page 1of 104

SBI CLERK 2021: The Success Guide

1 www.bankersadda.com | www.sscadda.com | www.careerpower.in | Adda247 App


SBI CLERK 2021: The Success Guide

Contents of SBI Clerk Prelims: Success Guide 2021


SBI Clerk Prelims Study Plan | Attempt Daily Quiz on Bankersadda.com & Adda247 App ............... 3

SBI Clerk 2021: Syllabus & Exam Pattern ............................................................................................................ 5

SBI Clerk Prelims Syllabus 2021 ..................................................................................................................................... 5


SBI Clerk Mains Syllabus 2021........................................................................................................................................ 7
SBI Clerk Prelims | Trend in Exam Analysis | Last 3 Years ............................................................................ 8

SBI Clerk Prelims Cut Off Trend: Last 3 Years .................................................................................................. 10

SBI Clerk Mains Cut Off Trend: Last 3 Years ..................................................................................................... 12

SBI Clerk Prelims 2020| Memory Based Paper | Questions & Solutions ................................................ 13

SBI Clerk Prelims 2019| Memory Based Paper | Questions & Solutions ............................................... 25

SBI Clerk Prelims 2018| Memory Based Paper | Questions & Solutions ................................................ 37

SBI Clerk Prelims 2016| Memory Based Paper | Questions & Solutions ............................................... 50

Most Important Questions | Quantitative Aptitude | SBI Clerk Prelims 2021 ...................................... 60

Most Important Questions | English Language | SBI Clerk Prelims 2021 ............................................... 74

Most Important Questions | Reasoning Ability | SBI Clerk Prelims 2021 .............................................. 94

2 www.bankersadda.com | www.sscadda.com | www.careerpower.in | Adda247 App


SBI CLERK 2021: The Success Guide

SBI Clerk Prelims Study Plan | Attempt Daily Quiz on


Bankersadda.com & Adda247 App
SBI Clerk 2021 official notification is released now. The prelims exam is scheduled to be held in June 2021 while the Mains
will be held on 31st July 2021. To help all the aspirant for SBI Clerk 2021 exam, we have already launched a Dedicated
Study Plan on Bankersadda.com & Adda247. Do attempt the Quizzes on Daily Basis. This will surely help you all to crack
the SBI CLERK PRELIMS 2021.
A proper Study Plan can help the students in covering all the Topics & Subjects which are relevant for that exam. The Study
Plan comprises Daily Quizzes covering all the Subjects of SBI Prelims, i.e. Quantitative Aptitude, Reasoning Ability &
English Language. This time, we are also providing Weekly Revision Tests from the Topics that are covered in that week.
Students will be provided with Weekly Revision Tests every Saturday. Lastly, we will also be covering previous year’s
Questions that are asked in SBI exams, Memory Based Practice Sets will be provided.

Do Attempt these Quizzes, Click here or


Visit: https://www.bankersadda.com/sbi-po-clerk-2021-30-days-study-plan/

SBI PO/Clerk 2021 – 30 Days Study Plan


Date Reasoning Ability Quantitative Aptitude English Language
12 June Syllogism Simplification Word usage
13 June Inequalities Missing Series Phrase rearrangement
14 June Coding-Decoding Quadratic Equation Sentence completion
15 June Series Approximation Miscellaneous
16 June Puzzle & Seating Arran Wrong Number Series Reading Comprehension
gement
17 June Direction Sense Percentage, Ratio & proportion, Avera Error Correction
ge, Number System
18 June Blood Relation Ages, SI & CI, Partnership Fillers
19 June Revision test Revision test Revision test
20 June Revision test Revision test Revision test
21 June Order & Ranking and S Profit & Loss, Time & Work, Pipe & Cist Word Usage
hort Puzzles ern
22 June Coding-Decoding Speed, Time and Distance, Trains, Boat Sentence Completion
& Stream
23 June Syllogism Simplification Error Detection
24 June Puzzle & Seating Arran Mensuration, Probability, Permutation Starters
gement & Combination
25 June Inequalities Missing Series Miscellaneous
26 June Revision test Revision test Revision test
27 June Revision test Revision test Revision test
28 June Direction Sense Table DI Fillers, Phrase replacement
29 June Blood Relation Bar Graph DI sentence based error, filler
30 June Series Pie Chart DI Miscellaneous
1 July Puzzle & Seating Arran Quadratic Equation Miscellaneous
gement
2 July Coding-Decoding Line DI & Misc DI Starters
3 July Revision test Revision test Revision test
4 July Revision test Revision test Revision test
5 July Order & Ranking and S Caselet Fillers, Phrase replacement
hort Puzzles
6 July Syllogism Quantity Based and Data Sufficiency Sentence based error, filler

3 www.bankersadda.com | www.sscadda.com | www.careerpower.in | Adda247 App


SBI CLERK 2021: The Success Guide
7 July Inequalities Approximation and Wrong Number Ser Column-based
ies
8 July Practice Set Based on Practice Set Based on memory Based Practice Set Based on memory Bas
memory Based ed
9 July Practice Set Based on Practice Set Based on memory Based Practice Set Based on memory Bas
memory Based ed
10 July Practice Set Based on Practice Set Based on memory Based Practice Set Based on memory Bas
memory Based ed

Do follow the New Study Plan for SBI Clerk 2021 Prelims. Download Adda247 App & Attempt Daily
Quizzes with Section Timings & All India Ranking. Download the App from the Link given below:

4 www.bankersadda.com | www.sscadda.com | www.careerpower.in | Adda247 App


SBI CLERK 2021: The Success Guide

SBI Clerk 2021: Syllabus & Exam Pattern


SBI Clerk is a two-stage examination consisting of Prelims and Mains followed by LPT i.e. the Language
Proficiency Test.
SBI Exam Pattern for Clerk Prelims is qualifying in nature which is followed by the SBI Clerk Mains whose
marks are added to the merit list.
The candidates applying for vacancies of a particular State should be proficient (reading, writing, speaking, and
understanding) in the specified opted local language of that State as mentioned in the notification.
The test of specified opted local language will be conducted as a part of the selection process and candidates
who will not be able to qualify for the language test will not be offered the Final Appointment.
Candidates who will produce the 10th or 12th standard mark sheet/ certificate evidencing having studied the
specified opted local language will not be required to undergo the local language test.

SBI Clerk Prelims Exam Pattern


S. No. Name of Tests (Objective) No. of Questions Maximum Marks Duration
1 English Language 30 30 20 minutes
2 Quantitative Aptitude 35 35 20 minutes
3 Reasoning Ability 35 35 20 minutes
Total 100 100 60 minutes

SBI Clerk Mains Exam Pattern


S. No. Name of Tests (Objective) No. of Questions Maximum Mar Duration
ks
1 Reasoning Ability & Computer Aptitud 50 60 45 minutes
e
2 General English 40 40 35 minutes
3 Quantitative Aptitude 50 50 45 minutes
4 General/Financial Awareness 50 50 35 minutes
Total 190 200 2 Hours 40 Minu
tes
Note: There will be negative marks of 1/4th in both prelims as well as mains exam. Also, there is a sectional timing
in both the phase of the SBI Clerk Exam.

SBI Clerk 2021: Test of Specified Opted Local Language:


After qualifying SBI Clerk Mains exam, candidates have to qualify for the specified opted local language test.
Those who produce 10th and 12th standard mark sheet having specified local language will not be
subject to any language test.
Candidates not found proficient in specified opted local language would be disqualified.

SBI Clerk Syllabus

SBI Clerk Prelims Syllabus 2021


English Language for SBI Clerk Prelims
• Reading comprehension
• Fillers (Double fillers, Multiple Sentence Fillers, Sentence Fillers)
• New Pattern Cloze Test
• Phrase Replacement
• Odd Sentence Out cum Para Jumbles
• Inference, Sentence Completion
• Connectors
• Paragraph Conclusion
5 www.bankersadda.com | www.sscadda.com | www.careerpower.in | Adda247 App
SBI CLERK 2021: The Success Guide
• Phrasal Verb Related Questions
• Error Detection Questions
• Word usage/ Vocab Based Questions
• Sentence Improvement
• Error Correction
• Idioms and Phrases
• Word Swap
• Word Rearrangement
• Word Usage
• Column Based Sentences
• Column Based Fillers
• Spellings
• Vocabulary
• Sentence Based Error
Reasoning Ability for SBI Clerk Prelims
• Direction Sense
• Coded-Direction
• Blood-Relation
• Syllogism
• Order and Ranking
• Coding-Decoding
• Machine Input-Output
• Inequalities
• Coded Inequality
• Alpha-Numeric-Symbol Series
• Alphabet Related Questions
• Coded Coding-Decoding
• Resultant and Coded Series
• Data-Sufficiency
• Input-Output or Coded Input-Output
• Circular/Triangular/Rectangular/ Square Seating Arrangement
• Linear or Direction with Linear Seating Arrangement
• Box Based Puzzles
• Floor or Floor-Flat Based Puzzles
• Comparison/ Categorized/ Uncertain/ Blood-Relation based Puzzles
• Day/ Month/ Year/ Age Based Puzzle

Quantitative Aptitude for SBI Clerk Prelims


• Data Interpretation (Bar Graph, Line Chart, Tabular, Pie Chart)
• Inequalities (Quadratic Equations)
• Number Series
• Approximation and Simplification
• Data Sufficiency
• Miscellaneous Arithmetic Problems (HCF and LCM, Profit and Loss, SI & CI, Problem on Ages, Work and Time,
Speed Distance and Time, Percentage, Probability, Mensuration, Permutation and Combination, Average,
Ratio and Proportion, Partnership, Problems on Boats and Stream, Problems on Trains, Mixture and
Allegation, Pipes and Cisterns).

6 www.bankersadda.com | www.sscadda.com | www.careerpower.in | Adda247 App


SBI CLERK 2021: The Success Guide
SBI Clerk Mains Syllabus 2021
General English for SBI Clerk Mains
• Reading comprehension
• Fillers (Double fillers, Multiple Sentence Fillers, Sentence Fillers)
• New Pattern Cloze Test
• Phrase Replacement
• Odd Sentence Out cum Para Jumbles
• Inference
• Sentence Completion
• Connectors
• Paragraph Conclusion
• Phrasal Verb Related Questions
• Error Detection Questions
• Word usage/ Vocab Based Questions.

Reasoning and Computer Aptitude for SBI Clerk Prelims


• Puzzles & Seating Arrangements
• Direction Sense
• Blood-Relation
• Syllogism
• Order and Ranking
• Coding-Decoding
• Machine Input-Output
• Inequalities
• Alpha-Numeric-Symbol
• Series
• Data Sufficiency
• Statement and Conclusion
• Statement and Assumption
• Course of Action
• Cause and Effect
• Statement and Inference
• Strength of Argument

Quantitative Aptitude for SBI Clerk Prelims


• Data Interpretation (Bar Graph, Line Chart, Tabular, Case let, Radar/Web, Pie Chart)
• Inequalities (Quadratic Equations, Quantity 1, Quantity 2)
• Number Series
• Approximation and Simplification
• Data Sufficiency
• Miscellaneous Arithmetic Problems (HCF and LCM, Profit and Loss, SI & CI, Problem on Ages, Work and Time,
Speed Distance and Time, Probability, Mensuration, Permutation and Combination, Average, Ratio and
Proportion, Partnership, Problems on Boats and Stream, Problems on Trains, Mixture and Allegation, Pipes
and Cisterns).
General/Financial Awareness for SBI Clerk Prelims
• Banking and Insurance Awareness,
• Financial Awareness,
• Govt. Schemes and Policies,
• Current Affairs,
• Static Awareness.

7 www.bankersadda.com | www.sscadda.com | www.careerpower.in | Adda247 App


SBI CLERK 2021: The Success Guide
SBI Clerk Prelims | Trend in Exam Analysis | Last 3 Years
In this article, we will analyze all three sections i.e. Quantitative Aptitude, Reasoning, and English.
By comparing the SBI Clerk Prelims paper 2020,2019 & 2018 we have prepared a chart along with a tabular
representation which will help you understand the examination trend.
We will try to explain to you through these charts, the topics that were predominant and the topics which were not
asked primarily.
This will help you in making a proper strategy to score highly in these exams. The marks distribution is a clear
indication of what you can expect in the exams.

SBI Clerk Prelims: Reasoning Section


In the Reasoning section, it can be seen that a minimum of 15
questions of puzzles and seating arrangement always ask in the SBI
clerk exam.
Questions of number series/alphanumeric series have been asked
for the last 3 years. For SBI Clerk 2021, we can say that the most
important topics that a student should cover are Puzzles, Seating
Arrangement, Syllogs & Number Series.
These topics comprise more than 70% of the Syllabus.

No. of Questions asked in SBI Clerk Prelims


Topics 2020 2019 2018
Puzzles and Seating Arrangement 15 15 15
Direction Sense 5 0 3
Numeric Series 5 1 5
Syllogism 4 5 0
Inequality 3 5 0
Alphabet Based Series 3 5 5
Blood Relation 0 4 2
Data Sufficiency 0 0 3
Coding-Decoding 0 0 2

SBI Clerk Prelims: Quantitative Aptitude Section


In the quantitative aptitude section, arithmetic word problems
always ask in the exam, so this topic should always be prepared.
Minimum 8-10 questions asked from the simplification topic and the
data interpretation is a must.
You can check the rest details from the table and chart given below.
Tabular & Pie Chart DI are the most important for SBI Clerk Prelims.
For the Arithmetic Part, the important chapters are SI, CI, Speed,
Distance & Time, Percentage, Average, Ratio, Proportion &
Partnership, Time & Work, Ages, etc.
No. of Questions asked in SBI Clerk Prelims
Topics 2020 2019 2018
Number Series (Missing) 5 5 5
DI (Data Interpretation) 5 7 5
Quadratic Equations 5 5 5
Simplification 10 10 8
Arithmetic Word Problem 10 8 12

8 www.bankersadda.com | www.sscadda.com | www.careerpower.in | Adda247 App


SBI CLERK 2021: The Success Guide
SBI Clerk Prelims: English Language Section
In the English section, most number of questions asked from the reading comprehension.
And after that topic, sentence rearrangement is the constant topic that is asked every year.
So, to crack SBI Clerk Prelims exam an aspirant need to work very hard on his reading skills so that RCs & Sentence
Rearrangement can be solved easily.
No. of Questions asked in SBI Clerk
Topic 202 201 2018
0 9
RCs (Reading 10 10 7
Comprehension)
Sentence Correction 5 5 0
Word Correction 5 0 0
Cloze Test 5 5 0
Sentence Rearrangement 5 5 5
Single Fillers (word) 0 5 5
Phrase Replacement 0 0 5
Error Detection 0 0 8

9 www.bankersadda.com | www.sscadda.com | www.careerpower.in | Adda247 App


SBI CLERK 2021: The Success Guide
SBI Clerk Prelims Cut Off Trend: Last 3 Years
In this article, we are providing you with a cut off trend for the previous years which will help you to understand
the toughness of the exam.
It will also give you a clearer picture of the cut-offs for different states which will give you an understanding of
the level of competition in different states.
If the location of the job is not an issue & you are willing to learn a new language, you can select the states with
higher vacancies.

SBI Clerk Vacancy Trend

SBI Clerk Cut Off Trend: 2018, 2019, 2020

After analyzing the cut-off of all the states from the past 3 years, we can clearly see that the Cut off for SBI Clerk
Prelims was lower in the year 2020 with respect to SBI Clerk Prelims in 2018 & 2019.
One of the major reason is the SBI Clerk 2020 exam which was Moderate as compare to the exams of 2018 &
2019 that were on the easier side. In some states, the cut off for SBI Clerk 2020 was on a higher side & in Some
States cut off was moderate due to the number of vacancies.

10 www.bankersadda.com | www.sscadda.com | www.careerpower.in | Adda247 App


SBI CLERK 2021: The Success Guide
What are the factors that can influence the SBI Clerk Cut Off?
The following factors that influence the Cut-Off:
• 1st Factor that influences the cut off is the Number of vacancies. An increase or Decrease in vacancies can
influence.
• 2nd one is the Level of Difficulty, in case the exam is easy, Cut off increases while in case of tough exams, the
cut-off will surely reduce.
• Lastly is the number of aspirants appearing in the Exam.

SBI Clerk Prelims State-Wise Cut-Off 2020 | 2019 | 2018

Name of the States 2018 2019 2020


Andhra Pradesh 71 74.75 68
Assam 51.25 57 -
Bihar 66.5 76.25 68.75
Chandigarh 71.25 77.25 76
Chhattisgarh 67.25 57.5 68.75
Delhi 71.75 71.25 76.25
Gujarat 57.5 65.5 56.75
Haryana 70.5 75.25 72.75
Himachal Pradesh 68.75 71.75 66
Jammu and Kashmir - 81.75 -
Jharkhand 69.5 75 68.25
Karnataka 63 48.5 58.75
Kerala 69 70.25 69.75
Madhya Pradesh 66.25 73.5 68.75
Maharashtra 57 62.25 59.75
Odisha 62.75 73.5 68.25
Punjab 71.5 76.25 77.5
Rajasthan 69 71 68.75
Tami Nadu 60 61.25 62
Telangana 68 68.5 66
Tripura - 70.25 -
Uttar Pradesh 69.25 72.25 71
Uttarakhand 71 75.25 69.75
West Bengal 66.5 73.25 67.5

11 www.bankersadda.com | www.sscadda.com | www.careerpower.in | Adda247 App


SBI CLERK 2021: The Success Guide
SBI Clerk Mains Cut Off Trend: Last 3 Years

Out of 200
Name of the State 2018 2019 2020
Andhra Pradesh - 99.75 88.75
Bihar 96 98 -
Chandigarh - - 96.75
Delhi 104.8 99.75 98.75
Gujarat 85 91.25 82.75
Himachal Pradesh 98.25 101.25 87.25
Karnataka 83.9 85.75 80.5
Madhya Pradesh 92.6 94.75 89.25
Maharashtra 85.65 89.75 84
Punjab 100.65 102.75 96.25
Rajasthan 96.75 97 90.25
Tamil Nadu 93.75 98 92.75
Telangana - - 86.75
Uttar Pradesh 96 97.5 90.25
Uttarakhand - 96.5 91
West Bengal 95.3 97.25 86.75
Jharkhand 86 97.5 -
Haryana 99 103.25 -
Odisha 88.8 94.75 87.75
Chhattisgarh 91.5 87.75 -
Kerala - 96.25 -
Assam - 85 -

12 www.bankersadda.com | www.sscadda.com | www.careerpower.in | Adda247 App


SBI CLERK 2021: The Success Guide

SBI Clerk Prelims 2020| Memory Based Paper |


Questions & Solutions
REASONING ABILITY
Direction (1-5): Study the following information carefully 6. Statements: Some Banana are Orange.
and answer the questions given below: All Orange are Apple.
Conclusions: I. Some Banana are not Apple.
There are eight employees of a company and all of them are II. Some Orange are Banana.
working on eight different designation of a bank viz.
Chairman, CFO, GM, DGM, AGM, Manager, Junior Manager 7. Statements: Only a few East are North.
and Clerk. All the designations given are to be considered Few North are South.
in a given order (as Chairman is considered as Senior-most All South are West.
and Clerk is considered as the Junior-most). Conclusions: I. All East being North is a possibility.
II. Some East are West.
Only two persons are senior to B. One designation lies
between B and G. The number of persons junior to G is 8. Statements: Only a few Song are Melody.
Only a few Melody are Film.
same as the number of persons senior to C. H is just senior
No Film is Award.
to E, but junior to C. More than four designations lie
Conclusions: I. All Song being Film is a possibility.
between H and F. D is junior to A.
II. Some Melody are not Award.
1. How many persons are junior to H? Direction (9-12): Study the following information
(a) None (b) One (c) More than four carefully and answer the questions given below:
(d) Four (e) Three In a certain code language:
2. Four of the following five are alike in a certain way and “Club house near located” is coded as “ol gp ox ot”
hence they form a group. Which one of the following “both club view near” is coded as “mt ox sq ot”
does not belong to that group? “make located house view” is coded as “nk ol gp sq”
(a) A-F (b) B-A (c) H-D “near club view area” is coded as “ot ox sq tm”
(d) G-A (e) E-H 9. What is the code for “both” in the given code language?
(a) ox (b) sq (c) mt
3. How many designation gaps are between A and D?
(d) ot (e) Either (b) or (c)
(a) More than three (b) Two (c) Three
(d) One (e) None 10. What is the code for “house” in the given code
language?
4. Who among the following is just senior to B? (a) ol (b) ox (c) gp
(a) A (b) D (c) C (d) Either (a) or (c) (e) None of these
(d) E (e) None of these
11. The code “nk” is stands for?
5. Who among the following is AGM? (a) make (b) both (c) area
(a) F (b) B (c) G (d) club (e) None of these
(d) D (e) None of these
12. What may be the possible code for “both house” in the
Direction (6-8): In each of the questions below are given given code language?
some statements followed by some conclusions. You have (a) gp mt (b) mt nk (c) mt sq
to take the given statements to be true even if they seem to (d) ox mt (e) gp sq
be at variance with commonly known facts. Read all the
Directions (13-17): Study the following information
conclusions and then decide which of the given conclusions
carefully and answer the questions given below:
logically follows from the given statements disregarding There are eight persons A, B, C, D, E, F, G and H sitting
commonly known facts. around a circular table facing towards the centre of the
(a) If only conclusion I follows. table but not necessarily in the same order.
(b) If only conclusion II follows. B sits third to the right of A. Only one person sits between
(c) If either conclusion I or II follows. B and D. E faces C who is not an immediate neighbour of B.
(d) If neither conclusion I nor II follows. G sits third to the left of F. A is not an immediate neighbour
(e) If both conclusions I and II follow. of F.

13 www.bankersadda.com | www.sscadda.com | www.careerpower.in | Adda247 App


SBI CLERK 2021: The Success Guide
13. Who among the following sits to the immediate left of 22. If the position of first and third digits of each of the
C? numbers are interchanged, then which among the
(a) G (b) D (c) F following is the highest number?
(d) H (e) None of these (a) 258 (b) 469 (c) 189
(d) 427 (e) 625
14. Who among the following sits to the opposite of A?
(a) H (b) G (c) F 23. If all the digits in the number are arranged in the
(d) D (e) None of these descending order within the number from left to right,
then which among the following will be the lowest
15. How many persons sit between H and G when counted
number after rearrangement?
from the left of G?
(a) 427 (b) 189 (c) 258
(a) Two (b) One (c) Three
(d) 625 (e) 469
(d) Four (e) None of these
24. What is the product of 3rd digit of 2nd lowest number
16. If all the persons sit around the circle in clockwise
and 1st digit of 2nd highest number?
direction by their names according to English
(a) 32 (b) 72 (c) 24
alphabetical order starting from A, then how many
(d) 36 (e) None of these
persons remain unchanged except A?
(a) Four (b) One (c) Three 25. If 2 is subtracted from each number then how many
(d) Two (e) None of these numbers thus formed are odd numbers?
(a) One (b) Two (c) Three
17. What is the position of B with respect to H? (d) More than three (e) None
(a) Fourth to the left
(b) Fourth to the right 26. How many such numerals are there in the number
(c) Third to the right ‘645903287’ which will remain at the same position
(d) Both (a) and (c) when arranged in ascending order from left to right?
(e) Both (a) and (b) (a) Three (b) Two (c) One
(d) Four (e) None of these
Direction (18-20): Study the following information
carefully and answer the questions given below: 27. If it is possible to make only one meaningful word with
the 1st, 3rd, 5th and 11th letters of the word
Point E is in 15m north of Point D. Point F is in 20m north ‘INHERITENCE’, which would be the third letter of the
of Point C. Point A is in 35m east of Point F. Point P is in word from the left? If more than one such word can be
25m south of Point A. Point E is in 20m east of Point P. formed give ‘Y’ as the answer. If no such word can be
18. What is the direction of point F with respect to point P? formed, give ‘Z’ as your answer.
(a) North-west (b) North-east (c) South-west (a) Y (b) R (c) I
(d) South-east (e) North (d) E (e) Z

19. Four of the following five are alike in a certain way and Direction (28-32): Study the following information
hence they form a group. Which one of the following carefully and answer the questions given below:
does not belong to that group? Eight persons A, B, C, D, E, F, G and H are buying some
(a) C-P (b) A-E (c) A-C products one after another but not necessarily in the same
(d) P-D (e) F-E order.
20. If point X is 20m south of point C, then what is the total At most two persons are buying products before F. Only
distance between point X and point D? one person is buying between D and F. C is buying just
(a) 40m (b) 35m (c) 55m before H. One person is buying between C and D. A is
(d) 65m (e) 85m buying just before E. B is buying before G and after E.

Directions (21-25): Following questions are based on the 28. How many persons are buying their products after D?
(a) None
five numbers given below, Study the given information and
(b) Two
answer the following questions.
(c) More than three
625 427 189 258 469
(d) One
21. What is the sum of the 3rd digit of second number from (e) None of these
left and 2nd digit of third number from right?
29. Who among the following is buying just after G?
(a) 10 (b) 15 (c) 14
(a) E (b) F (c) B
(d) 12 (e) None of these
(d) C (e) None of these
13 www.bankersadda.com | www.sscadda.com | www.careerpower.in | Adda247 App
SBI CLERK 2021: The Success Guide
30. If all the persons are arranged in alphabetical order Directions (33-35): In each of the question, relationships
from left to right starting from A, then find how many between some elements are shown in the statements.
These statements are followed by conclusions numbered I
persons remains at the same position (excluding A)?
and II. Read the statements and give the answer.
(a) One (b) None (c) Two (a) If only conclusion I follows.
(d) Four (e) More than Four (b) If only conclusion II follows.
(c) If either conclusion I or II follows.
31. Who among the following is buying exactly between D (d) If neither conclusion I nor II follows.
and F? (e) If both conclusion I and II follows.
(a) E (b) B (c) A 33. Statements: P > Q ≥ R = S < T = U
(d) H (e) None of these Conclusions: I. Q > S II. S = Q
34. Statements: J < D = L ≤ K ≤ Q ≥ R
32. How many persons are buying between E and C?
Conclusions: I. J < Q II. L ≤ R
(a) Five (b) Four (c) Three
35. Statements: O > P = G > B ≥ X = M ≤ H
(d) None (e) Two
Conclusions: I. G > X II. O > M

Quantitative Aptitude

Directions (36-40): Table given below shows the number 40. Find total number of male students participated from
of male and female participated in an event from five all the five schools together.
different schools (A, B, C, D & E). Study the table carefully (a) 2860 (b) 3150 (c) 2940
and answer the following questions. (d) 3200 (e) 3020

Schools Male Female Direction (41–45): What will come in the place of
A 650 450 question (?) mark in following number series:
B 540 420 41. ?, 100, 150, 375, 1312.5
C 720 500 (a) 100 (b) 200 (c) 150
D 560 450 (d) 400 (e) 50
E 680 320 42. 104, ?, 96, 120, 88, 128
36. Find average number of female participated from (a) 112 (b) 110 (c) 114
(d) 118 (e) 108
school – A, B & D.
(a) 400 (b) 380 (c) 350 43. 15, 8, 9, 15, 32, ?
(d) 440 (e) 450 (a) 66 (b) 99 (c) 80
(d) 82.5 (e) 80.5
37. Total male participated from school – B & D together
are how much more or less than total female 44. 6, 8, 14, 26, 46, ?
participated from school – A & C together? (a) 72 (b) 84 (c) 96
(a) 150 (b) 110 (c) 170 (d) 80 (e) 76
(d) 120 (e) 240 45. 72000, 36000, 12000, 3000, 600, ?
38. Total male participated from school – B & C together (a) 120 (b) 200 (c) 300
(d) 150 (e) 100
are what percent more or less than total female
participated from school – A & D together? 46. 12 men can do a work in 10 days while 10 women can
(a) 20% (b) 60% (c) 50% do the same work in 18 days. In how many days 4 men
(d) 40% (e) 30% & 6 women together can do the same work?
120 180
39. If total male participated from school – F are 40% more (a) 7 days (b) 24 days (c) 13 days
than that of from school – A and ratio of female (d) 15 days (e) 18 days
participated from school – B to that of from school – F 47. A car can cover a distance in 4 hour at speed 60 kmph
is 21:32, then find total students participated from then by what percent should the speed of car be
school – F. increased to cover the same distance in 2.5 hr?
(a) 1420 (b) 1550 (c) 1580 (a) 60% (b) 40% (c) 50%
(d) 1460 (e) 1490 (d) 100% (e) 75%
14 www.bankersadda.com | www.sscadda.com | www.careerpower.in | Adda247 App
SBI CLERK 2021: The Success Guide
48. The ratio of the ages of Ram and Rahim 10 years ago 59. A & B entered into a business by investing total capital
was 1 : 3. The ratio of their ages five years hence will of Rs 17000. B withdraws Rs 1500 after 6 months and
be 2 : 3. Then, the ratio of their present ages is : gets Rs 8100 as profit out of total profit of Rs 19500 at
(a) 1 : 2 (b) 3 : 5 (c) 3 : 4 the end of year. Find capital of B after 6 months from
(d) 2 : 5 (e) None of these starting.
(a) Rs 7000 (b) Rs 9500 (c) Rs 7500
49. Two trains of length 140m & 120m are running in same
(d) Rs 6000 (e) Rs 6500
direction on parallel tracks with speeds 132 kmph &
80 kmph respectively. How much time will they take to 60. If length of a rectangle increases by 40% while keeping
cross each other? breadth constant then area of rectangle increased by
(a) 7.09 sec (b) 18 sec (c) 11.7 sec 24 m2 and perimeter of original rectangle is 32 m. find
(d) 4.42 sec (e) Cannot be determined breadth of rectangle.
(a) 8.4 m (b) 10 m (c) 6 m
50. A person sold a book at 20% profit. If he had bought it
(d) 14 m (e) 8 m
at 10% less cost and sold for Rs 90 more then he would
have gained 40% profit. Find cost price of book. Direction (61–70): What will come in the place of (?)
(a) Rs 800 (b) Rs 1600 (c) Rs 1500 mark in following question.
(d) None of these (e) Rs 1200 61. 280 ÷ 4 ÷ 2 = 170 – ?
Direction (51–55): In each question two equations (a) 105 (b) 115 (c) 125
numbered (I) and (II) are given. You have to solve both the (d) 135 (e) 145
equations and mark appropriate answer. ?
62. (√144 + √169 ) × 3 = 5
(a) If x = y or no relation can be established
(b) If x > y (a) 375 (b) 325 (c) 350
(c) If x < y (d) 275 (e) 475
(d) If x ≥ y 63. (12 × 5 ÷ 4) × 8 = ?
(e) If x ≤ y (a) 100 (b) 140 (c) 120
51. I. 𝑥 = √25 II. y3 = 125 (d) 80 (e) 90
64. (120% of 750) ÷ ? = 25
52. I. x2 + 2x – 35 = 0 II. y2 + 15y + 56 = 0
(a) 30 (b) 36 (c) 24
53. I. x2 = 81 II. y2 = 64 (d) 18 (e) 48
54. I. 17x2 – 14x – 83 = - 80 II. y2 = 2y + 35 1 5
65. 8 2 − 4 6 =? −3 12
7

55. I. x2 + 4x – 45 = 0 II. y2 − 13𝑦 + 40 = 0 (a) 3


1
(b) 3
5
(c) 2
7
4 12 12
56. A container contains mixture of milk & water in ratio 5 (d) 7 4
1
(e) 5 3
2

: 3 respectively. If 8 lit milk is added in it then ratio of


milk to water becomes 11 : 5. Find difference between 66. 275 + 64% of 750 = 750 + ?
initial quantity of milk & that of water. (a) 25 (b) 8 (c) 10
(a) 5 lit (b) 38 lit (c) 18 lit (d) 15 (e) 5
(d) 30 lit (e) 10 lit
67. √225 + √81 + 122 =?
57. Rs 6000 when invested at a certain rate at SI for 2 (a) 168 (b) 164 (c) 162
years, it fetches Rs 1200. If same sum is invested at (d) 172 (e) 182
same rate for a year compounded half – yearly then 510
find compound interest. 68. ?
= √324 + 3.25
(a) Rs 615 (b) Rs 600 (c) Rs 1200 (a) 12 (b) 48 (c) 24
(d) Rs 585 (e) Rs 1260 (d) 6 (e) 18

58. A boat can cover 28 km downstream in 42 min. ratio of 69. 12.5% of (120 + ?) = 45
speed of boat in still water to speed of stream is 7 : 3. (a) 160 (b) 180 (c) 360
Find difference between time taken by boat to cover 60 (d) 240 (e) 120
km downstream & 40 km upstream. 70. 572 ÷ 13 × 12 – 16 = (8)?
(a) 2.25 hr (b) 1 hr (c) 1.5 hr (a) 4 (b) 2 (c) 3
(d) 0.4 hr (e) 0.9 hr (d) 5 (e) None of these

15 www.bankersadda.com | www.sscadda.com | www.careerpower.in | Adda247 App


SBI CLERK 2021: The Success Guide
English Language

Directions (71-77) Read the given passage carefully and their host-insect interactions. These are hypothetical risks
answer the following questions. Certain parts have been that have been raised by scientists, and reflect the need for
highlighted to help answer the questions. further study.
Every year, around one million people die of mosquito- 71. Why are mosquitoes considered as one of the deadliest
borne diseases according to the World Health Organization living creatures on earth?
(WHO). This is why mosquitoes are considered one of the (a) Capability to transmit life threatening diseases.
deadliest living creatures on the planet — not because (b) Mosquito bites cause fatal reactions in human
they are lethal themselves, but because many of the viruses beings
and parasites they transmit are. (c) Mosquitoes cannot be killed easily
In the absence of an effective vaccine for dengue fever, Zika (d) Both (a) and (c)
fever, chikungunya and other mosquito-borne diseases, (e) None of the above
researchers have developed genetic strategies to reduce
mosquito populations. One such strategy involves the 72. What is one of the methods being developed to control
release into the wild of genetically modified (GM) population of mosquitoes?
mosquitoes that express a lethal gene — a strategy (a) Spraying of pesticides using established
believed to have little impact on the overall DNA of wild procedures.
populations of mosquitoes. (b) Introduction of a lethal gene into wild populations
The transfer of new genes from GM organisms to wild or of mosquitoes using genetically modified ones.
domesticated non-GM populations is a key criticism of GM (c) Not letting any stagnant water build-up near
crops like soybean and corn. There are concerns that the homes
introduction of GM genes into non-target species could
(d) Both (a) and (b)
have negative consequences for both human and
(e) None of the above
environmental health.
Oxitec, a company that spun out of research at Oxford 73. What can be the potential side effect(s) of genetically
University in the early 2000s, developed and trademarked modified mosquito offspring?
GM Friendly™ mosquitoes (also known as strain OX513A (a) Evolution of a breed of mosquitoes which are more
of Aedes aegypti). These male GM mosquitoes have what difficult to control
the company describes as a “self-limiting” gene, which (b) Increase in the severity of diseases being currently
means that when these so-called friendly mosquitoes’ transmitted by mosquitoes.
mate, their offspring inherit the self-limiting gene which is (c) Change in host-insect interactions.
supposed to prevent them surviving into adulthood.
(d) Loss of natural populations of mosquitoes
In theory, when these mosquitoes are released in high
(e) (a), (b) and (c)
numbers, a dramatic reduction in the mosquito population
should follow. According to research published by Oxitec 74. Which of the statements can be considered as true with
researchers in 2015, field trials involving recurring respect to the passage given?
releases of Friendly™ mosquitoes demonstrated a (a) Field trials on Genetically modified mosquitoes
reduction of nearly 95 per cent of target populations in showed reduction of 96% target populations.
Brazil. In these field trials, experiments were not (b) Oxitec trademarked the GM Friendly mosquitoes
performed to assess whether GM mosquitoes might persist in 2015.
in the wild.
(c) The female GM mosquito has a self-limiting gene
A recent study from the Powell lab at Yale University has
(d) Both (a) and (b)
since confirmed that some of the offspring of the GM
(e) None of the above
mosquitoes didn’t succumb to the self-limiting lethal gene
and survived to adulthood. They were able to breed with 75. What is the desired effect of releasing GM mosquitoes
native mosquitoes and thereby introduce some of their containing the ‘Self-limiting Gene’?
genes into the wild population. (a) A substantial reduction in the population of
Meanwhile, the impact of mosquitoes carrying these new mosquitoes.
genes remains largely unknown. One significant worry is (b) Birth of mosquitoes without a biting mechanism
that a new breed of mosquito might emerge that is more (c) Decrease in the number of diseases caused due to
difficult to control. These new genes could also potentially
mosquitoes
alter evolutionary pressures on viruses carried by
(d) Increase in the number of male mosquitoes.
mosquitoes, like dengue fever, in unpredictable ways. This
includes potentially increasing their virulence or changing (e) Both (b) and (c)

16 www.bankersadda.com | www.sscadda.com | www.careerpower.in | Adda247 App


SBI CLERK 2021: The Success Guide
76. Which of the following word is similar to deadliest as 80. ESCALATE
mentioned in the passage given? (i) The pedestrian was jaywalking when he escalate
(a) fatal (b) demeaning (c) fulfil the busy street, walking straight into the path of an
oncoming vehicle.
(d) harmless (e) None of the above
(ii) We've tried to escalate the most likely problems,
77. Which of the statements can be considered as true with but it's impossible to be prepared for every
eventuality.
respect to the passage given??
(iii) The decision to escalate UN involvement has been
(a) GM mosquitoes are able to introduce some of their made in the hopes of a swift end to the hostilities.
genes into the wild population. (a) Only (i)
(b) Mosquitoes having GM genes have been (b) Both (i) and (ii)
thoroughly researched upon. (c) Both (ii) and (iii)
(d) Only (iii)
(c) Oxitec released its friendly mosquitoes in Brazil
(e) All of these
(d) (a) and (c)
Directions (81-85): In the following question, four
(e) All of the above
sentences are given which may be grammatically and
Directions (78-80): A word has been given in each contextually incorrect. You need to find the one which has
no error and mark that as your answer. If all the given
question and has been used in the sentences given below.
sentences are incorrect then mark option (e) i.e. ‘all are
Identify the statements where the word has been used in a incorrect’ as your answer.
contextually and grammatically correct manner. If the
81. (a) India’s chocolate market is pegged at Rs.11,000
word has been used correctly in all the statements, mark
crore, of which premium chocolates can be sized
(e), “All of these”, as your answer. anywhere among 10% and 15%.
(b) South Korea reported 52 new case of coronavirus.
78. HOSTILE
(c) ITC has plans to work with cocoa farmers directly
(i) He made hostile efficiency a key part of his in future.
budget plan. (d) The company initially invested Rs. 100 crore to set
(ii) There has been a hostile reaction to the up a greenfield chocolate manufacture facility in
government's proposed tax increase. Haridwar.
(e) All are incorrect
(iii) The security forces exercised great restraint by
not responding to hostile attacks and threats. 82. (a) Ratnakar Bank was rename as RBL Bank by the
government.
(a) Only (i)
(b) The government expects all the mergers of public
(b) Both (i) and (ii) sector banks, which were announced in 2019, to
(c) Both (ii) and (iii) become effective from April 1.
(d) Only (iii) (c) Analysts believe that lower gas prices would be
(e) All of these bad with exploration
(d) The Central Drugs Standard Control Organisation
79. VAGUE will be regulate the medical devices now.
(i) The judges determined that the law was too vague (e) All are incorrect
to be fairly enforced. 83. (a) Mahindra Renewables will sell its entire stake in 3
(ii) The merger of these two companies would vague subsidiaries to CLP India, a part of Hong Kong-
the world's biggest accounting firm based CLP Group, as nearly ₹340 crore.
(b) The agreement will be signed to Doha city between
(iii) The president had a vague reception in Ohio this
Taliban representatives and U.S. special envoy
morning. Zalmay Khalilzad.
(a) Only (i) (c) The U.S. intelligence community publicly
(b) Both (i) and (ii) concluded that Russia intervened in there security
(c) Both (ii) and (iii) matters.
(d) The injured animal was nursed back to health by
(d) Only (iii)
the zookeeper and then released back into the wild
(e) All of these (e) All are incorrect
17 www.bankersadda.com | www.sscadda.com | www.careerpower.in | Adda247 App
SBI CLERK 2021: The Success Guide
84. (a) English language have the potential to connect the Directions (91-95): In the following questions, a
world without any barriers grammatically correct and meaningful sentence is given
(b) P.K. Sinha was appointed in the PMO when which is divided into five parts, where the first part is fixed
Nripendra Misra, the then Principal Secretary, and highlighted , in the remaining parts of the sentence
resigns. namely (A),(B),(C) and (D) You have to arrange the four
(c) She drink the caramel liquid too fast and was soon parts to make a contextually and grammatically
too dizzy to stand. meaningful sentence. If no such rearrangement is possible
(d) You’re coming to the party, isn’t you? mark (e) as your answer i.e. 'No rearrangement required'.
(e) All are incorrect 91. (A) the merger of Bharti Infratel
(B) the world’s second largest
85. (a) Tamil Nadu is likely to get three textile parks and a
(C) and Indus Towers will create
technology research centre under schemes that
(D) tower company
the Union Ministry of Textiles is formulating. (a) BCDA (b) CBDA (c) ACBD
(b) What is the uses of a house if you haven't got a (d) DCAB (e) No rearrangement is required
tolerable planet to put it on?
(c) You have brushed your teeth today? 92. (A) upcoming visit to India
(d) Rahul had barely nothing to help the poor children. (B) the long-awaited trade agreement
(C) during US President Donald Trump's
(e) All are incorrect
(D) appears to be off the table
Directions (86-90): In the following questions, a sentence (a) DABC (b) BDCA (C) CDAB
is divided into four parts consisting of a highlighted word (d) ADCB (e) No rearrangement required
in each part. Choose the option reflecting the word which 93. (A) Leaders of the Railway Employees Organization
is either misspelt or grammatically incorrect. If all the (B) alleged that the government wants to
highlighted words are correct, choose option (e) i.e. “all are (C) hand over the rail infrastructure
correct” as your answer choice. (D) to the private operators for financial benefit
86. The manager balanced (A) the strength (B) of his (a) ADCB (b) BACD (c) CDBA
(d) DABC (e) No rearrangement required
team against that of their opponent (C) and sighed.
(D) 94. (A) China has not yet given
(a) balanced (b) strength (c) opponent (B) India the go-ahead to send an
(d) sighed (e) All are correct (C) to coronavirus hit Wuhan
(D) aircraft with medical supplies
87. One evening Rohan pripared (A) a nice supper (B) (a) DACB (b) ABDC (c) CBDA
and put it on low heat in the oven to keep it tepid (C) (d) BCDA (e) No rearrangement is required
while his wife dressed. (D)
(a) prepared (b) supper (c) tepid 95. (A) mammals and birds move from
(d) dressed (e) All are correct (B) with a change in season, many
(C) one country to another in search of food
88. She was able to breathe (A) easier when she stood (D) and shelter, and for breeding
outside the massive(B) fortress (C) that sat on a (a) ACBD (b) CADB (c) DABC
clearing the size of two footbal (D) fields. (d) BACD (e) No rearrangement is required
(a) breathe (b) massive (c) fortress Directions (96-100): In the following passage there are
(d) football (e) All are correct blanks, each of which has been numbered. These numbers
89. From time to time Jack would looking over(A) are printed below the passage and against each, five
Mungo's shoulder, suggesting(B) tactics (C) which options are given. Find out the appropriate word which fits
invariably proved disastrous.(D) the blank appropriately.
(a) looking over (b) suggesting (c) tactics Bumblebees, among the most important pollinators, are in
(d) disastrous (e) All are correct (96) ________________. Fuzzy and buzzy, they excel at
spreading pollen and fertilizing many types of wild flora, as
90. Horrified (A) passengers saw Olive stumble (B) and
well as crucial agricultural crops like tomatoes,
fall of (C) a platform as an express roared (D) in.
blueberries, and squash. But their numbers are (97)
(a) Horrified (b) stumble (c) fall of
________________. New research using a massive dataset found
(d) roared (e) All are correct that the insects are far less common than they used to be;
18 www.bankersadda.com | www.sscadda.com | www.careerpower.in | Adda247 App
SBI CLERK 2021: The Success Guide
in North America, you are nearly 50 percent less likely to percent less plentiful than they were in the early 20th
see a bumblebee in any given area than you were prior to century. The scientists examined the abundance of 66
1974. Moreover, several once-common species have (98) species across the two continents.
__________________ from many areas they were once found,
96. (a) extinct (b) trouble (c) dropped
becoming locally extinct in those places. For example, the
(d) growth (e) difficult
rusty patched bumblebee, which used to flourish in
Ontario, is no longer found in all of Canada—in the U.S., it’s 97. (a) increasing (b) dripped (c) removing
endangered. In a new paper published this week in the (d) dropping (e) generating
journal Science, researchers used a complex modeling 98. (a) multiplied (b) concerning (c) disappeared
process to (99) ________________ that their (d) certain (e) vanish
(100)___________________ is driven in large part by climate
change. Specifically, the scientists found that in areas that 99. (a) proposed (b) recommending
have become hotter in the last generation, or have (c) implying (d) expresses (e) suggest
experienced more extreme temperature swings, 100. (a) decline (b) deteriorate (c) rejection
bumblebees are less abundant. In Europe, they are 17 (d) lessen (e) reduced

Solutions

REASONING ABILITY
Solutions (1-5): 8. (e);

Designation Persons
Chairman F
CFO A
GM B Solutions (9-12):
Word Code
DGM C
Club Ox
AGM G
Near Ot
Manager D House/Located Ol/gp
Junior manager H Both Mt
Clerk E Make Nk
Area Tm
1. (b) 2. (d) 3. (c)
View Sq
4. (a) 5. (c)
9. (c)
6. (b); 10. (d)
11. (a)
12. (a)
Solutions (78-82):

7. (d);

13. (b); 14. (c); 15. (b);


16. (d); 17. (e);

19 www.bankersadda.com | www.sscadda.com | www.careerpower.in | Adda247 App


SBI CLERK 2021: The Success Guide
Solutions (83-85): 21. (b); 22. (c); 23. (d)
24. (a); 25. (d);
26. (c); 6 4 5 9 0 3 2 8 7
0 2 3 4 5 6 7 8 9
27. (a); Hire, Heir
Solutions (28-32):

18. (a);
19. (c);
20. (c); 28. (e);
29. (d);
30. (a);
31. (b);
32. (b);
33. (c); I. Q > S (False) II. S = Q (False)
34. (a); I. J < Q (True) II. L ≤ R (False)
35. (e); I. G > X (True) II. O > M(True)

Quantitative Aptitude

26. (d); Required average =


450+420+450
= 440 43. (d); Pattern of series -
3

37. (a); Total male participated from school – B & D


together = 540 + 560 = 1100 44. (e); Pattern of series -
Total female participated from school – A & C
together = 450 + 500 = 950
Required difference = 1100 − 950 = 150
38. (d); Total male participated from school – B & C 45. (e); Pattern of series -
together = 540 + 720 = 1260
Total female participated from school – A & D
together = 450 + 450 = 900 46. (d); let total work be 360 units
1260−900 360
Required % = 900
× 100 = 40% Efficiency of 1 man = 12×10 = 3 units/day
360
39. (b); Total students participated from school F = Efficiency of 1 woman = 10×18 = 2 units/day
360
140
× 650 + 420 ×
32
Required time = = 15 𝑑𝑎𝑦𝑠
100 21 4×3+6×2
= 910 + 640 = 1550 47. (a); distance = 240 kms
240
40. (b); Total number of male students participated from Required speed = 2.5 = 96 𝑘𝑚𝑝ℎ
all the five schools Required % =
96−60
× 100 = 60%
60
= (650 + 540 + 720 + 560 + 680) = 3150
48. (b); Let 10 years ago, ages of Ram and Rahim were x
41. (b); Pattern of series – years and 3x years, respectively.
Then, present age of Ram = (x + 10)
and present age of Rahim = (3x + 10)
According to the question,
42. (a); Pattern of series - x+10+5 2
= 3 ⇒ 3x + 45 = 6x + 30 ⇒ 3x = 15
3x+10+5
∴x=5
5+10 15
Hence, required ratio = 3×5+10 = 25 = 3 ∶ 5
20 www.bankersadda.com | www.sscadda.com | www.careerpower.in | Adda247 App
SBI CLERK 2021: The Success Guide
49. (b); required time =
140+120
5
57. (a); let rate of interest be R%
(132−80)× 6000×𝑅×2
260×18
18 ATQ, 1200 =
100
= = 18 sec
52×5 R = 10%
50. (c); let CP of book be Rs x Since compounding is done half-yearly, rate of
SP = Rs 1.2x interest = 5%
5×5
New CP = Rs 0.9x Effective rate of interest = 5 + 5 + = 10.25%
100
New SP = Rs 1.2x + 90 6000×10.25×1
140 Required interest = = Rs 615
ATQ, 0.9𝑥 × = 1.2𝑥 + 90 100
100
1.26x = 1.2x + 90 58. (b); let speed of boat in still water & speed of stream
x = Rs 1500 be 7x & 3x kmph respectively
28 42
ATQ, 7𝑥+3𝑥 = 60
51. (a); I. x = 5
II. y = 5 x=4
40 60 4
So, x=y Required difference = − = = 1 hour
7𝑥−3𝑥 7𝑥+3𝑥 𝑥
52. (d); I. x2 + 7x – 5x – 35 =0 59. (d); let amount invested by A be Rs x
x (x + 7) – 5 (x + 7) =0 Profit ratio; A : B = (x × 12) : (17000 – x) × 6 +
(x + 7) (x – 5) = 0 (15500 – x) × 6
x = −7, 5 = 2x : (32500 – 2x)
II. y2 + 7y + 8y + 56 = 0 19500
y(y + 7) + 8(y + 7) = 0 ATQ, 32500−2𝑥+2𝑥 × (32500 − 2𝑥) = 8100
(y + 7) (y + 8) = 0 32500 – 2x = 13500
y = − 8, −7 x = Rs 9500
So, x≥y required capital of B after 6 months = 15500 – x =
Rs 6000
53. (a) I. x = ± 9
II. y = ± 8 60. (c); let length & breadth of rectangle be x & y m
So, no relation can be established respectively
ATQ, 1.4xy – xy = 24
54. (a); I. 17x2 – 14x – 3 = 0
xy = 60 ………………(i)
17x2 – 17x + 3x – 3 = 0
also, 2(x + y) = 32
17x (x – 1) + 3(x – 1) = 0
x + y = 16 ………….(ii)
(17x + 3) (x – 1) = 0
3 from (i) & (ii)
x = − ,1
17 x = 10 m, y = 6 m
II. y2 – 2y – 35 = 0 breadth of rectangle = 6 m
y2 – 7y + 5y – 35 = 0
y(y −7) + 5(𝑦 − 7) = 0 61. (d); ? = 170 – 35
y = 7, - 5 ? = 135
So, no relation can be established ?
62. (a); (12 + 13) × 3 = 5
55. (e); I. x2 + 9x – 5x – 45 = 0 ? = 375
x(x + 9) – 5(x + 9) = 0
(x – 5) (x + 9) = 0 63. (c); ? = (3 × 5) × 8
x = 5, - 9 ? = 120
II. y2 – 5y – 8y + 40 = 0 120
64. (b); ( 100 × 750 ) ÷? = 25
y(y – 5) – 8(y – 5) = 0
(y – 5) (y – 8) = 0 ? = 900 ÷ 25
y = 5, 8 ? = 36
So, x≤y 6−10+7
65. (d); ? = (8 – 4 + 3)+ 12
56. (e); let initial quantity of milk & water be 5x & 3x lit 1
?=74
respectively
5𝑥+8 11
ATQ, 3𝑥 = 5 64
66. (e); 275 + 100 × 750 = 750 + ?
25x + 40 = 33x ⇒ x = 5 275 + 480 = 750 + ?
required difference = 5x – 3x = 2x = 10 lit ?=5

21 www.bankersadda.com | www.sscadda.com | www.careerpower.in | Adda247 App


SBI CLERK 2021: The Success Guide
67. (a); ? = 15 + 9 + 144 120 + ? = 360
? = 168 ? = 240
510
68. (c); = 18 + 3.25
? 70. (c); 44 × 12 − 16 = (8)?
? = 24
528 – 16 = (8)?
12.5
69. (d); × (120+? ) = 45 ?=3
100

English Language

71. (a); Reading the first paragraph of the passage it can mosquitoes’ mate, their offspring inherit the self-
be deduced that the correct answer is option(a). limiting gene which is supposed to prevent them
The relevant sentences of the mentioned surviving into adulthood In theory, when these
sentences have been quoted below: mosquitoes are released in high numbers, a
“This is why mosquitoes are considered one of the dramatic reduction in the mosquito population
deadliest living creatures on the planet — not should follow.”
because they are lethal themselves, but because
76. (a); Among the given words, ‘fatal’ is the synonym of
many of the viruses and parasites they transmit
‘deadliest’. Hence, option (a) is the
are.”
most suitable answer choice.
72. (b); Reading the second paragraph of the passage it Fatal: causing death.
can be deduced that the correct answer is Demeaning: causing someone to lose their dignity
option(b). The sentences of the passage which and the respect of others.
substantiate this are given below: 77. (d); Reading the above passage it can be seen that the
“One such strategy involves the release into the correct statements are (a) and (c). The sentences
wild of genetically modified (GM) mosquitoes that have been quoted below for your reference-
express a lethal gene — a strategy believed to “According to research published by Oxitec
have little impact on the overall DNA of wild researchers in 2015, field trials involving
populations of mosquitoes.” recurring releases of Friendly™ mosquitoes
73. (e); Reading the last paragraph of the passage it can be demonstrated a reduction of nearly 95 per
deduced that the correct answer is cent of target populations in Brazil. A recent
option(e). The relevant sentences of the study from the Powell lab at Yale University has
mentioned sentences have been quoted below: since confirmed that some of the offspring of the
“One significant worry is that a new breed of GM mosquitoes didn’t succumb to the self-limiting
mosquito might emerge that is more difficult to lethal gene and survived to adulthood. They were
control. These new genes could also potentially able to breed with native mosquitoes and
alter evolutionary pressures on viruses carried by thereby introduce some of their genes into the
wild population.”
mosquitoes, like dengue fever, in unpredictable
ways. This includes potentially increasing their 78. (c); Hostile means showing or feeling opposition or
virulence or changing their host-insect dislike; unfriendly. Therefore, sentence (ii) and
interactions.” (iii) depict the correct usage of the given word.
Hence, option (c) is the most suitable answer
74. (e); Reading the passage carefully we can see that
choice.
none of the sentences given above are factually
correct hence the correct answer will be option 79. (a); Vague means thinking or communicating in an
(e). unfocused or imprecise way. Therefore, only
sentence (i) depicts the correct usage of the given
75. (a); Reading the fourth and fifth paras of the passage word. Hence, option (a) is the most suitable
it can be deduced that the correct answer is answer choice.
option(a). The relevant sentences of the
mentioned sentences have been quoted below: 80. (d); Escalate means increase rapidly. Therefore, only
“These male GM mosquitoes have what the sentence (iii) depicts the correct usage of the
company describes as a “self-limiting” gene, which given word. Hence, option (d) is the most suitable
means that when these so-called friendly answer choice.

22 www.bankersadda.com | www.sscadda.com | www.careerpower.in | Adda247 App


SBI CLERK 2021: The Success Guide
81. (c); As we go through the analysis of the sentence, we 90. (c); Except “fall of” which is spelt as “fall off” all other
find that the sentence of option (c) is correct. words are correctly spelt so our answer choice
(a) replace ‘among’ with ‘between’. Among is used will be option (c).
for a greater number.
91. (c); The correct rearrangement of the parts of
(b) replace ‘case’ with ‘cases’. statement will be “ACBD” which will make a
(d) replace’ manufacture’ with ‘manufacturing’. grammatically correct and contextually
meaningful statement. Hence, option (c) is the
82. (b); As we go through the analysis of the sentence, we most suitable answer choice. The correct
find that the sentence of option (b) is correct. statement thus formed will be:
(a) replace ‘rename’ with ‘renamed’. “The merger of Bharti Infratel and Indus Towers
(c) replace ‘with’ with ‘for’. will create the world’s second largest tower
(d) replace ‘regulate’ with ‘regulating’. company.”
92. (b); The correct rearrangement of the parts of
83. (d); As we go through the analysis of the sentence, we statement will be “BDCA” which will make a
find that the sentence of option (d) is correct. grammatically correct and contextually
(a) replace ‘as’ with ‘for’. meaningful statement. Hence, option (b) is the
(b) replace ‘to’ with ‘in’. most suitable answer choice. The correct
(c) replace ‘there’ with ‘their’. statement thus formed will be:
“The long-awaited trade agreement appears to be
84. (e); As we go through the sentences, we find that all off the table during US President Donald Trump’s
sentences are incorrect. upcoming visit to India.”
(a) replace ‘have’ with ‘has’. 93. (e); In the given sentence the sentence elaborates on
(b) replace ‘resigns’ with ‘resigned’. allegations made against the government by
leaders of railway employee organisations with
(c) replace ‘drink’ with ‘drank’.
respect to their stand on handing over railway
(d) replace ‘isn’t’ with ‘aren’t’. infrastructure to private companies. Hence,
85. (a); As we go through the analysis of the sentence, we option (e) is the most suitable answer choice
which means the sentence requires no
find that the sentence in option (a) is correct. rearrangement. The statement thus formed will
(b) replace ‘uses’ with ‘use’. be:
(c) interchange ‘have’ with ‘you’ to maintain the “Leaders of the Railway Employees Organization
interrogative form of the sentence. alleged that the government wants to hand over
(d) replace ‘nothing’ with ‘anything’ because the rail infrastructure to the private operators for
financial benefit.”
‘barely’ will not take ‘nothing’ after it.
94. (b); The correct rearrangement of the parts of
86. (a); Except “balenced” which is spelt as “balanced” all statement will be “ABDC” which will make a
other words are correctly spelt so our answer grammatically correct and contextually
choice will be option (a). meaningful statement. The given statement talks
about India not being granted the permission to
87. (a) ; Except “pripared” which is spelt as “prepared” all send an aircraft carrying medical supplies to the
other words are correctly spelt so our answer coronavirus hit Wuhan region. The correct
choice will be option (a). statement thus formed will be:
“China has not yet given India the go-ahead to
88. (d); Except “footbal” which is spelt as “football” all send an aircraft with medical supplies to
other words are correctly spelt so our answer coronavirus hit Wuhan.”
choice will be option (d). 95. (d); The correct rearrangement of the parts of
statement will be “BACD” which will make a
89. (a); Except “looking over” which is grammatically grammatically correct and contextually
wrong all other words are correctly spelt and are meaningful statement. The given sentence talks
grammatically flawless so our answer choice will about migration in animals with changing seasons
be option (a). Replace “looking over” with “look in search of their basic needs. Hence, option (d) is
the most suitable answer choice. The correct
over”.
statement thus formed will be:
23 www.bankersadda.com | www.sscadda.com | www.careerpower.in | Adda247 App
SBI CLERK 2021: The Success Guide
“With a change in season, many mammals and 99. (e); The most suitable word for the given blank is
birds move from one country to another in search “suggest”. All the other words are either
of food and shelter, and for breeding.” grammatically or contextually incorrect. Hence,
option (e) is the correct answer choice.
96. (b); The given paragraph is describing the alarming
100. (a); The given paragraph is describing the
situation of Bumblebees where their numbers are alarming situation of Bumblebees where their
continuously declining. Thus, the most suitable numbers are continuously declining. Thus, the
word for the given blank is “trouble”. Hence, most suitable word for the given blank is
option (b) is the correct answer choice. “decline”. All the other words are either
grammatically or contextually incorrect. Hence,
97. (d); The given paragraph is describing the alarming option (a) is the correct answer choice.
situation of Bumblebees where their numbers are
continuously declining. Thus, the most suitable
word for the given blank is “dropping”. All the
other words are contextually incorrect. Hence,
option (d) is the correct answer choice.
98. (c); The given paragraph is describing the alarming
situation of Bumblebees where their numbers are
continuously declining. Thus, the most suitable
word for the given blank is “disappeared”. All the
other words are either grammatically or
contextually incorrect. Hence, option (c) is the
correct answer choice.
Disappeared means cease to exist or be in use.
Vanish means disappear suddenly and
completely.
Certain means able to be firmly relied on to
happen or be the case.

24 www.bankersadda.com | www.sscadda.com | www.careerpower.in | Adda247 App


SBI CLERK 2021: The Success Guide

SBI Clerk Prelims 2019| Memory Based Paper |


Questions & Solutions
REASONING ABILITY

Direction (1-5): Study the following information carefully 6. Who among the following faces Q?
and answer the questions given below: (a) B (b) D (c) A
Eight persons are sitting around a circular table facing to (d) C (e) None of these
the center but not necessarily in the same order. 7. Who among the following sits at the extreme end of the
Two persons sit between Q and P (either from left or right). row?
R sits immediate to the right of Q. One person sits between (a) E (b) T (c) C
R and S, who faces to T. Q and T are not immediate (d) R (e) P
neighbors of each other. W sits 2nd to the left of V. Three 8. Four of the following five are alike in a certain way and
persons sit between U and V. hence they form a group. Which one of the following
1. Four of the following five are alike in a certain way and does not belong to that group?
hence they form a group. Which one of the following (a) Q (b) A (c) T
does not belong to that group? (d) D (e) P
(a) Q-W (b) P-U (c) S-W
9. Who among the following sits 2nd to the right of P?
(d) V-T (e) Q-P
(a) S (b) R (c) Q
2. Who among the following sits immediate right of U? (d) T (e) None of these
(a) W (b) R (c) T
(d) S (e) V 10. What is the position of A with respect to E?
(a) 3rd to the left
3. The number of persons sit between Q and T , when (b) Immediate to the left
counted to right of Q is same as the number of persons (c) 2nd to the left
sit between W and ___, when counted to the left of ___? (d) 3rd to the right
(a) P (b) S (c) T
(e) 2nd to the right
(d) U (e) None of these
11. In the word ‘PRODUCE’, how many pairs of the letters
4. Who among the following faces R?
have the same number of letters between them in the
(a) U (b) V (c) P
(d) W (e) Q given word as they have in the English alphabet series?
(a) Four (b) Two (c) One
5. Who among the following sits 3rd to the right of S? (d) Three (e) More than four
(a) P (b) U (c) Q
(d) T (e) None of these Direction (12-15): In each of the questions below are
given some statements followed by some conclusions. You
Direction (6-10): Study the following information have to take the given statements to be true even if they
carefully and answer the questions given below: seem to be at variance with commonly known facts. Read
all the conclusions and then decide which of the given
Ten persons are sitting in two parallel rows containing five
conclusions logically follows from the given statements
persons in each row such a way that there is an equal
disregarding commonly known facts.
distance between adjacent persons. In the first row, A, B, C,
D and E are seated and all of them are facing north. In the 12. Statements: All Chocolate are Toffee. No Toffee is
second row, P, Q, R, S and T are seated and all of them are Muffin. Only a few Muffin are Dark.
facing south. Therefore, in the given seating arrangement, Conclusions: I. No Chocolate is Muffin.
each member seated in a row faces another member of the II. Some Toffee can never be Dark.
other row. (a) If only conclusion II follows.
E sits 2nd from one of the extreme end of the row. P faces (b) If only conclusion I follows.
the one who sits 2nd to the right of E. D sits 2nd to the left of (c) If neither conclusion I nor II follows.
B, who does not sit at the extreme end. Two persons sit (d) If either conclusion I or II follows.
between S and Q. R sits immediate left of S. C sits next to B. (e) If both conclusions I and II follow.
25 www.bankersadda.com | www.sscadda.com | www.careerpower.in | Adda247 App
SBI CLERK 2021: The Success Guide
13. Statements: No Party is Gathering. All Summary are 20. As many as persons are buying things before D as
Gathering. Only a few Gathering are Committee. after___?
Conclusions: I. Some Committee can be Summary. (a) E (b) B (c) F
II. Some Party can be Committee. (d) A (e) G
(a) If either conclusion I or II follows.
Direction (21-23): Study the following information
(b) If only conclusion I follows.
carefully and answer the questions given below:
(c) If neither conclusion I nor II follows.
(d) If only conclusion II follows. Point S is 15m west of point R. Point R is 30m south of
(e) If both conclusions I and II follow. point Q. Point P is 20m west of point Q. Point U is 15m
south of point P. Point T is 35 north of point S.
14. Statements: No Market is Home. Only a few Home are
Room. Only a few Room is Vance. 21. If point V is exactly between point Q and R, then how
Conclusions: I. Some Home are Vance. far and in which direction is point U with respect to V?
II. Some Market can never be Room. (a)15m, North-East
(a) If only conclusion I follows. (b) 15m, East
(b) If neither conclusion I nor II follows. (c) 10m, North-west
(c) If either conclusion I or II follows. (d) 20m, West
(d) If only conclusion II follows. (e) 20m, North-East
(e) If both conclusions I and II follow.
22. Four of the following are alike in a certain way, so form
15. Statements: Some Ball are Garden. All Garden are a group. Which of the following does not belong to that
Trade. Only a few Trade are Pump. group?
Conclusions: I. Some Ball are Pump. (a) P, T (b) U, Q (c)R, P
II. Some Garden can be Pump. (d) S, Q (e) U, T
(a) If only conclusion II follows.
23. If point W is in 5m east of point U, then what is the
(b) If neither conclusion I nor II follows.
(c) If either conclusion I or II follows. distance between point W and Point S?
(d) If only conclusion I follows. (a) 5m (b) 15m (c) 25m
(e) If both conclusions I and II follow. (d) 10m (e) 20m

Direction (16-20): Study the following information Direction (24-26): Study the following information
carefully and answer the questions given below: carefully and answer the questions given below:
Seven persons are buying different things. Only one person A certain number of persons are sitting in the row. All of
buy thing between C and A. B is buying thing immediately them are facing towards north. Q sits sixth from the right
before C. Two persons buy things between C and D. C buy of S. T sits forth to left of Q. Only two persons sit between
things after D. E buys thing immediately before D. More Q and P. R sits forth to the left of S. U sits between S and T.
than three persons buy things between E and F. Only three V sits second to the right of U. W is third from any of the
persons buy things between D and G. end. S is eight from the left end of the row. Six persons sit
16. How many persons buy things between A and F? between W and V.
(a) One (b) Two (c) Three 24. How many numbers of persons could sit in the row?
(d) Four (e) More than four (a) 14 (b) 18 (c) 23
17. Who among the following buy thing immediately after (d) 15 (e) 20
F? 25. What is the position of ‘W’ with respect to Q?
(a) D (b) B (c) G (a) Fourth to the right
(d) A (e) No one (b) Fifth to the right
18. Who among the following buy thing immediately (c) Fourth to the left
before G? (d) Eighth to the right
(a) D (b) B (c) C (e) Sixth to the left
(d) A (e) No one 26. If X sits immediate right of V then how many persons
19. How many person buy things before A? sit between X and P?
(a) One (b) Two (c) Three (a) Five (b) Six (c) Four
(d) Four (e)More than four (d) Three (e) None of these

26 www.bankersadda.com | www.sscadda.com | www.careerpower.in | Adda247 App


SBI CLERK 2021: The Success Guide
27. If it is possible to make only one meaningful word with A@34%ENM$86&LDS#986QYZ17%ROG
the 2nd, 4th, 6th and 7th letters of the word @2IB2U&
‘UNILATERAL’ which would be the second letter of the
word from the right end? If more than one such word 31. Which of the following element is sixth to the left of the
can be formed give ‘Y’ as the answer. If no such word fourteenth from the left end of the given arrangement?
can be formed, give ‘Z’ as your answer. (a) 6 (b) % (c) $
(a) Y (b) N (c) L (d) M (e) None of these
(d) T (e) Z
32. If all the symbols are dropped from the series, which
Directions (28-30): In each of the question, relationships element will be twelfth from the right end?
between some elements are shown in the statements. (a) 9 (b) Q (c) R
These statements are followed by conclusions numbered I
(d) Y (e) None of these
and II. Read the statements and give the answer.
(a) If only conclusion I follows. 33. How many such numbers are there in the given series
(b) If only conclusion II follows. which are immediately preceded by a symbol and
(c) If either conclusion I or II follows. followed by a letter?
(d) If neither conclusion I nor II follows. (a) None (b) One (c) Two
(e) If both conclusions I and II follow. (d) Three (e) Four
28. Statements: A ≥ B ≥ C = D > E ≤ F < G 34. How many such letters are there in the given series
Conclusions: I. E < B II. G > E which are immediately preceded by number and
29. Statements: P ≤ R < T = U; Q ≥ T ≤ S ≥ V immediately followed by a symbol?
Conclusions: I. Q > P II. V < R (a) One (b) Two (c) Three
(d) More than three (e) None of these
30. Statements: L > M = O ≥ P; N ≤ M ≥ S ≥ T
Conclusions: I. T ≤ P II. N < L 35. Find the odd one out?
Directions (31-35): Study the following sequence and (a) N64 (b) D86 (c) Y%8
answer the given questions. (d) R27 (e) 8EL

QUANTITATIVE APTITUDE
Directions (36-40): What will come in the place of 38. 4, 3, 4, 9, 32, ?
question mark (?) in the following number series: (a) 75 (b) 155 (c) 125
(d) 175 (e) 165
36. 11, ?, 16, 21, 29, 41
(a) 12 (b) 14 (c) 15 39. ?, 100, 150, 375, 1312.5
(d) 13 (e) 11 (a) 50 (b) 100 (c) 75
(d) 25 (e) 200
37. 1800, ?, 60, 15, 5, 2.5
40. 0, 6, 24, 60, ?, 210
(a) 300 (b) 600 (c) 120 (a) 130 (b) 170 (c) 90
(d) 240 (e) 360 (d) 120 (e) 150
Directions (41-45): Study the bar chart given below and answer the following questions.
Bar chart shows the number of books read by 4 different persons (A, B, C & D) in 2005 and 2006.

100
90
80
70
60
50 2005
40 2006
30
20
10
0
A B C D

27 www.bankersadda.com | www.sscadda.com | www.careerpower.in | Adda247 App


SBI CLERK 2021: The Success Guide
41. Find average number of books read by A, C & D in 2005. 54. 35% of 150 × 16 = ? – 22
(a) 64 (b) 70 (c) 75 (a) 865 (b) 932 (c) 864
(d) 60 (e) 56 (d) 862 (e) None of these
42. Find ratio of books read by B & C together in 2005 to 55. (3080 + 6160) ÷ ? = 330
books read by A & D together in 2006. (a) 26 (b) 22 (c) 28
(a) 15 : 16 (b) 5 : 6 (c) 1 : 5 (d) 29 (e) 18
(d) 4 : 7 (e) 2 : 3 56. Difference of the compound interest received in first
43. Books read by A & D together in 2005 are what percent year and second year at 20% per annum at CI is Rs
more than books read by C in 2006? 1200 then find the sum?
2
(a) 46 %
1
(b) 54 %
2
(c) 25 % (a) Rs 25,000 (b) Rs 36,000 (c) Rs 35,000
3
1
3
2
3 (d) Rs 24,000 (e) Rs 30,000
(d) 33 3 % (e) 66 3 %
57. Find the total distance covered by boat in each
44. Books read by A & C together in 2005 are how much upstream and downstream in 7 hours if the speed of
more or less than books read by B & D together in boat in still water and speed of current is 21 km/h and
2006? 3 km/h respectively?
(a) 24 (b) 14 (c) 18 (a) 280 km (b) 294 km (c) 315 km
(d) 22 (e) 28 (d) 301 km (e) 322 km
45. Books read by B & C together in 2006 are what percent 58. Ratio of income of A to that of B is 5:9. If expenditure of
3 4
of books read by B in 2005? A is th of his income and expenditure of B is th of his
8 9
(a) 100% (b) 120% (c) 250% income and sum of their saving is Rs 1950 then find the
(d) 200% (e) 160% difference between their income?
Directions (46-55): What will come in place of (?) (a) Rs 900 (b) Rs 1000 (c) Rs 880
question mark in the following questions? (d) Rs 960 (e) Rs 920
17.28÷? 59. A alone can do a work in 12 days while A and B
46. = 200
3.6 × 0.2 together can do that work in 7.5 days. Find the time
(a) 120 (b) 1.20 (c) 12 taken by C alone to do that work if C takes 3 days more
(d) 0.12 (e) None of these than that of B alone to do that work?
47. 486 ÷ ? × 7392 ÷ 66 = 1008 (a) 33 days (b) 30 days (c) 23 days
(a) 54 (b) 55 (c) 52 (d) 27 days (e) 28 days
(d) 53 (e) 51 60. Ratio of base and perpendicular side of a right-angled
2 1 triangle is 3:4 and its base is equal to the side of a
48. 14 % of 4200 ÷ √576 = (? ) 2
square having area 81 cm2. Find the perimeter of the
7
(a) 125 (b) 225 (c) 25 triangle?
(d) 5 (e) 625 (a) 30 cm (b) 36 cm (c) 33 cm
2 5 3 (d) 42 cm (e) 40 cm
49. 7 × 6 × 8 ×? = 90
Directions (61-65): In each of these questions, two
(a) 1208 (b) 1108 (c) 1008
equations (I) and (II) are given. You have to solve both the
(d) 1128 (e) 1348
equations and give answer
50. (0.05 × 6.25) ÷ 2.5 =? (a) if x>y
(a) 12.55 (b) 0.125 (c) 0.115 (b) if x≥y
(d) 1.25 (e) None of these (c) if x<y
(d) if x ≤y
51. 1496 ÷ 17 = ?% of 220
(e) if x = y or no relation can be established between x and
(a) 25 (b) 40 (c) 50
y.
(d) 75 (e) None of these
61. I. x² - 13x + 40 = 0
52. (36% of 180) ÷ 0.4 = ?
II. 2y² - y – 15 = 0
(a) 160 (b) 164 (c) 166
(d) 162 (e) 180 62. I. 5x² + 17x + 6 = 0
II. 2y² + 11y + 12 = 0
53. 0.08% of 55500 – 16.4 = ?
(a) 26.6 (b) 28 (c) 29.2 63. I. 7x² - 19x + 10 = 0
(d) 30.4 (e) 32 II. 8y² + 2y – 3 = 0

28 www.bankersadda.com | www.sscadda.com | www.careerpower.in | Adda247 App


SBI CLERK 2021: The Success Guide
64. I. 𝑥 2 − 8𝑥 + 15 = 0 1
68. Retailer sold one article at 33 3 % profit and another at
II. 𝑦 2 − 3𝑦 + 2 = 0 100% profit. Find his overall profit percentage if the
65. I. 3x² –7x + 4 = 0 selling price of both the article is same?
2
II. 2y² – 9y + 10 = 0 (a) 60% (b) 55% (c) 66 %
3
2
66. A person travels half of the distance at the speed of x (d) 75% (e) 56 %
3
km/h and remaining half of the distance at 4x km/h.
69. A mixture has milk and water in the ratio 4: 1. When
Find the value of ‘x’ if the average speed is 36.8 km/h? 50% of the mixture is taken out and replaced by 24
(a) 21 (b) 25 (c) 24 liters of water then the ratio of milk to water in the
(d) 23 (e) 20 mixture becomes 1: 1. Find initial quantity of mixture.
67. A, B and C invested in a ratio of 7: 8: 5 in a business. (a) 80 liters (b) 45 liters (c) 70 liters
They got an annual profit of Rs. 136800. If A and C (d) 60 liters (e) 75 liters
withdrew their amount at the end of 3 months and 7 70. 4 years ago, ratio of Shivam’s age to Deepak’s age was
months respectively. Then find the difference between 2: 3 and ratio of Shivam’s age 4 years ago to Deepak’s
A and C’s share of profit? age 5 years hence is 8: 15. Find present age of Shivam.
(a) Rs. 12,600 (b) Rs. 11,500 (c) Rs. 13,500 (a) 32 years (b) 28 years (c) 40 years
(d) Rs. 10,500 (e) Rs. 13,000 (d) 24 years (e) 36 years

ENGLISH LANGUAGE

Directions (71-78): Read the following passage and they are deployed - along with enhanced security through
answer the following questions based on the given the use of multiple server locations. With all this
passage. Some of the words are highlighted which would considered, it is no surprise that 42% of UK businesses
help you to answer some of the questions given. leverage some kind of cloud service, according to Eurostat.
However, all the perceived benefits of leveraging the cloud
For years, world-wide organisations have become are redundant if organisations come up against barriers to
increasingly excited about the prospect of a cloud-based accessing cloud services. Cloud-based data pipelines still
future. As the dream becomes an ever closer reality for suffer from complexity challenges at the moment, along
many kinds of business and, Forrester predicted that with the lack of visibility into cost and resource usage at
enterprise spending on cloud services is set to surge. IDC the application and user level. The answer to this is
also predicted that global spending on public cloud automation fueled by robust Machine learning training
services and infrastructure would reach $210bn in 2019, models and artificial intelligence. These concepts and the
an increase of 24% from 2018. But one obstacle stands tools that enable them can determine the prerequisites of
create friction and introduce risk: the process of migration. cloud infrastructure, application dependencies,the
As all indications point to a massive shift in data appropriate target cloud instance profiles, and provide
deployments to the cloud, it is more important than ever troubleshooting in real-time.
that the transition from on-premises to Cloud is as risk free
To summarise, the promise of the cloud has created a sense
as possible. In today's climate any loss or disruption to data
can have a huge business impact. It’s a complex process, is of excitement amongst enterprises, however, they have
frequently underestimated and many organisations have proceeded to go full steam ahead into adopting a cloud
found there’s lots that can go wrong that can impact the service, without sufficient data to ensure performance
business. service level agreements (SLAs).

Organisations across the globe have found the cloud to be 71. How can we tackle the risks associated with the
an ideal place to run modern data applications due to big process of migration?
data’s elastic resource requirements. Furthermore, with (a) By ensuring that organisations only migrate the
the lack of data talent an ever-looming issue for most apps to the cloud that will thrive in the cloud.
companies today they have been determined to adopt a (b) The surge in the investment in the cloud
cloud-first strategy to ensure business operations are technology.
accessible for a range of employees. (c) The use of predictive power of Artificial
Intelligence.
The cloud offers great promise for developers especially, as
(d) The transition from on-premises to Cloud
it can increase the speed at which they develop software
features and increase the resilience of applications once (e) None of these

29 www.bankersadda.com | www.sscadda.com | www.careerpower.in | Adda247 App


SBI CLERK 2021: The Success Guide
72. Why the promise of the cloud has created a sense of 78. Which of the following words is most OPPOSITE to the
excitement amongst enterprises? word
(a) As it ensures that business operations are “MASSIVE” as given in the passage?
accessible for a range of employees. (a) Prominent
(b) As the enterprise spending on cloud services is (b) Insignificant
(c) Enlarged
surging.
(d) Filthy
(c) Reduction in the cost of operation it may entail.
(e) Vouch
(d) The secure platform it provides to the data it
stores. Directions (79-85): Read each sentence to find out if there
(e) None of these is any error in it. The error, if any will be in one part of the
sentence. The number of that part is the answer if there is
73. How the process of migration stands as an obstacle to no error, the answer is (e).
the cloud technology?
79. It is widely believe (a)/ that the village which is
(a) The expensive cost of providing safety to the data. situated on the (b)/ eastern coast of India will be wiped
(b) The lack of sound technology in Artificial out (c)/ within a decade. (d)/ No error. (e)
Intelligence.
(c) Complexities of the data migrated to the cloud 80. Yesterday I met (a)/ an old friend (b)/ when I am going
(c)/ to the market. (d)/ No error.(e)
(d) It exposes the companies to the risk of losing
important information. 81. The criminal was (a)/ sentenced to the (b)/ death and
(e) None of these was (c) hung for his crime (d)/ No error.(e)

74. Which of the following statement(s) is/are correct in 82. FATF strongly urges (a)/ Pakistan to swiftly complete
context of the passage? its action plan (b)/ by October 2019 to stop (c)/ terror
funding in the country (d)/ No error.(e)
(1) The cloud technology can help in increasing the
speed at which the software features are 83. Elon musk is determined (a)/ to be success (b)/ in
developed. whatever field (c)/ he chooses. (d)/ No error.(e)
(2) There is no risk associated with the cloud 84. New purchasing power will increasingly (a)/ come
technology. from Asia and Africa where (b)/ the demographics are
(3) Cloud technology is not a very complex process. still favorable (c)/ for high income growth. (d)/ No
(a) Only (2) error (e).
(b) Both (1) and (2) 85. Not only the (a)/ students but also (b)/ the principal
(c) Only (3) were(c)/ laughing at the joke he cracked.(d)/No error
(d) Only (1) (e).
(d) All are correct
Directions (86-90): In each of the following sentences,
75. What is the meaning of the phrase “full steam ahead” ? there is a blank space. Below in each sentence, there are
(a) Interrupting an activity five words out of which one can be used to fill the blank to
(b) Acting cautiously make the sentence grammatically and coherently correct.
(c) Great increase Find the most appropriate word that fit into the blank
(d) Progressing quickly. contextually.
(e) Prudent progression 86. There is nothing to indicate the building's past, _______
the fireplace.
76. Which of the following words is most similar to the
(a) except (b) included (c) belonged
word “ADOPT” as given in the passage? (d) barred (e) Foster
(a) mold (b) deduce (c) Embrace
(d) Creative (e) Fluster 87. Two circus elephants that performed together _______
when crossing paths 23 years later
77. Which of the following words is most similar to the (a) happy (b) believe (c) angered
word “ENSURE” as given in the passage? (d) rejoiced (e) greater
(a) Confirm 88. The law _______ farmers not only to save and resow
(b) Paced (multiply) seeds, but also to sell them to other farmers,
(c) Redundant no matter what the original source of the seed is.
(d) Weaken (a) coverts (b) implements (c) emits
(e) Reject (d) permits (e) deserts

30 www.bankersadda.com | www.sscadda.com | www.careerpower.in | Adda247 App


SBI CLERK 2021: The Success Guide
89. Small farmers must be educated and ______ with proper 95. Which of the following should be the FIFTH sentence
incentive structures, to engage with agriculture. after the rearrangement?
(a) fascinated (a) A (b) B (c) C
(b) encouraged (d) D (e) E
(c) discouraged
Directions (96-100): In each the following questions four
(d) harvested
words are highlighted. Choose the option reflecting the
(e) invited
word which is either misspelt or grammatically incorrect.
90. Raipur despite being _______ between two rivers, the If all the highlighted words are correct, choose option (e)
Krishna and the Tungabhadra, it is a dust bowl in May i.e. “all are correct” as your answer choice.
first week.
96. A British court has ruled that British arms sales to
(a) Surrounded
Saudi Arabia wear unlawful on Humanitarian
(b) Located
Grounds.
(c) Revealed
(a) ruled (b) sales (c) wear
(d) Settle
(d) Grounds (e) All are correct
(e) None of these.
97. I'm tyred of hearing politicians making pious
Directions (91-95): Given sentences are not in their exact
pronouncements about their devotion to the people
position. Rearrange them to make a coherent paragraph
(a) tyred (b) hearing (c) pious
and then answer the questions given below.
(d) devotion (e) All are correct
A. It can even make exercise more fun and productive.
98. It is likely that weaving prospered and partly
B. Plus, hearing others laugh, even for no apparent
relocated towards areas within easy reach of the
reason, can often trigger genuine laughter.
army contractors.
C. To add simulated laughter into your own life, search (a) prospered (b) relocated (c) within
for laugh yoga or laugh therapy groups. Or you can (d) contractors (e) All are correct
start simply by laughing at other people’s jokes, even if 99. This cat and mouse tactic was purposely designed to
you don’t find them funny. provoke, inrage and panic the unemployed.
D. A Georgia State University study found that (a) tactic (b) purposely (c) inrage
incorporating bouts of simulated laughter into an (d) panic (e) All are correct
exercise program helped improve older adults’ mental 100.Indian democracy’s robustness is underscored by
health as well as their aerobic endurance. high voter turnouts, and large number of candedates
in the fray.
E. Believe it or not, it’s possible to laugh without
(a) robustness (b) turnouts (c) candedates
experiencing a funny event—and simulated laughter
(d) fray (e) No correction required
can be just as beneficial as the real thing.
91. Which of the following should be the FOURTH sentence
after the rearrangement?
(a) A (b) B (c) C
(d) D (e) E
92. Which of the following should be the SECOND sentence
after the rearrangement?
(a) A (b) B (c) C
(d) D (e) E
93. Which of the following should be the THIRD sentence
after the rearrangement?
(a) A (b) B (c) C
(d) D (e) E
94. Which of the following should be the FIRST sentence
after the rearrangement?
(a) A (b) B (c) C
(d) D (e) E

31 www.bankersadda.com | www.sscadda.com | www.careerpower.in | Adda247 App


SBI CLERK 2021: The Success Guide
Solutions
REASONING ABILITY
Direction (1-5): 15. (a);

Direction (16-20):

Persons
1. (a); 2. (c); 3. (b); E
4. (c); 5. (b); D
A
Direction (6-10): B
C
G
F
16. (c); 17. (e); 18. (c);
19. (b); 20. (e);

6. (c); 7. (b); 8. (e); Direction (21-23):

9. (a); 10. (d); 21. (d);

11. (c);

Direction (12-15):
12. (b);

22. (c);
13. (e);

14. (b);

32 www.bankersadda.com | www.sscadda.com | www.careerpower.in | Adda247 App


SBI CLERK 2021: The Success Guide
23. (b); Direction (28-30):

28. (e); I. E < B (True) II. G > E (True)

29. (a); I. Q > P (True) II. V < R (False)

30. (b); I. T ≤ P (False) II. N < L (True)

Direction (31-35):

31. (d); M

Direction (24-26): 32 (d); Y

33. (b); One–@2I

24. (e); 25. (a); 26. (c); 34. (a); One – 2U&

27. (b); One word formed- lent 35. (e); 8EL

QUANTITATIVE APTITUDE

36. (d); 43. (e); Required % =


{(72+48)−72}
× 100
72
200 2
= 3
% = 66 3 %

44. (b); Required difference = (72 + 90) – (78 + 70)


= 162 – 148
= 14
37. (a);
78+72
45. (c); Required % = 60
× 100
150
= × 100 = 250%
60

46. (d); 17.28 ÷? = 200 × 3.6 × 0.2


17.28
38. (b); ?=
144
= 0.12
486 7392
47. (a); ?
× 66
= 1008
486 1008
=
39. (e); ? 112
486
?= 9
? = 54
1
100 1
40. (d); 48. (e); 700
× 4200 × 24 = (? )2
1
25 = (? )2
? = 625
90×7×8
49. (c); ? = = 1008
5

72+90+48 0.3125
41. (b); Required average = 3 50. (b); ? =
2.5
= 70 ? = 0.125
60+90
42. (a); Required ratio = 90+70 51. (b);
1496 ?
= 100 × 220
17
150
= = 15: 16 ? = 40
160

33 www.bankersadda.com | www.sscadda.com | www.careerpower.in | Adda247 App


SBI CLERK 2021: The Success Guide
52. (d);
36
× 180 ×
10
=? 62. (e); I. 5x² + 17x + 6 = 0
100 4
? = 162 5x² + 15x + 2x + 6 = 0
5x (x + 3) +2(x + 3) = 0
53. (b); 0.08 × 555 − 16.4 =? x = −3, − 5
2
? = 44.4 − 16.4 ⇒ ? = 28
II. 2y² + 11y + 12 = 0
35
54. (d); × 150 × 16 + 22 =? 2y² + 8y + 3y + 12 = 0
100
? = 840 + 22 = 862 2y (y + 4) + 3 (y + 4) = 0
3
9240 y = −4, − 2
55. (c); = 330
? No relation
? = 28
63. (a); I. 7x² - 19x + 10 = 0
56. (e); Let the sum be Rs 100x 7x² - 14x – 5x + 10 = 0
CI in first year= Rs 20x
7x (x - 2) – 5 (x - 2) = 0
CI in two years= 44% of 100x= Rs 44x 5
CI in 2nd year= 44x-20x= Rs 24x x = 2, 7
ATQ II. 8y² + 2y – 3 = 0
24x-20x=1200 8y² + 6y – 4y – 3 = 0
x=300 2y (4y + 3) – 1 (4y + 3) = 0
Required sum=Rs 30,000 y= ,
−3 1
4 2
57. (b); Speed in upstream=18 km/hr x >y
Speed in downstream= 24 km/hr
Required total distance= (24 + 18) × 7 = 64. (a); I. x 2 − 8x + 15 = 0
294 km ⇒ x 2 − 5x − 3x + 15 = 0
⇒ x(x − 5) − 3(x − 5) = 0
58. (d); Let income of A and B be Rs 5x and Rs 9x ⇒ (x − 3)(x − 5) = 0
respectively ∴ x = 3 or 5
15
Expenditure of A=Rs x II. y 2 − 3y + 2 = 0
8
Saving of A=Rs 8 x
25 ⇒ y 2 − 2y − y + 2 = 0
⇒ y(y − 2) − 1(y − 2) = 0
Expenditure of B= Rs 4x
⇒ (y − 1)(y − 2) = 0
Saving of B= Rs 5x
∴ y = 1 or 2
ATQ
65 ∴x>y
8
x = 1950 ⇒ x=240
Required difference= Rs 960 65. (c); I. 3x² –7x + 4 = 0
⇒ 3x²– 4x - 3x +4 = 0
59. (c); Let total work be 60 units (LCM of 12 and 7.5) ⇒ (3x – 4) (x -1) = 0
Efficiency of A= 5 units/ day 4
x = or 1
Efficiency of A and B together= 8 units/ day 3
Efficiency of B= 3 units/ day II. 2y² -9y + 10 = 0
Time taken by B alone to do that work=20 days ⇒ 2y² - 4y - 5y + 10 = 0
Time taken by C alone=23 days ⇒ (2y - 5) (y -2) =0
5
⇒ y = or 2
60. (b); Side of the square=9 cm 2
Perpendicular side of the triangle= 12 cm y>x
Hypotenuse of the triangle=√81 + 144 = √225 = 66. (d); Let the distance be D km
15 cm ATQ
Perimeter of the triangle= 36 cm D
D D = 36.8
( + )
61. (a); I. x² -13x + 40= 0 2x 8x

x² - 5x – 8x + 40 = 0 x=23
x (x -5) – 8 (x - 5) = 0 67. (a); Ratio of their profit sharing
x = 5, 8 A: B: C = 7 × 3 ∶ 8 × 12 ∶ 5 × 7 = 21 ∶ 96 ∶ 35
II. 2y² - y – 15 = 0 Annual profit = 136800
2y² - 6y + 5y – 15 = 0 Difference b/w A and C’s share of profit
2y (y - 3) + 5 (y - 3) = 0 14
y=3, -5/2 = × 136800
152
x>y = Rs 12,600

34 www.bankersadda.com | www.sscadda.com | www.careerpower.in | Adda247 App


SBI CLERK 2021: The Success Guide
15x = 24
68. (a); Let SP of both article = 8x
x = 1.6
ATQ,
So, required quantity = 40x + 10x
= 50x
= 80 liters

16x−10x
70. (b); Let age of Shivam and Deepak 4 years ago be ‘2x
Profit % = × 100 = 60% years’ and ‘3x years’ respectively.
10x
ATQ,
69. (a); Let initial quantity of milk and water in the 2x 8
mixture be ‘40x liters’ and ‘10x liters’ 3x+4+5
= 15
2x 8
respectively. = 15
3x+9
ATQ, 30x = 24x + 72
1
(40x× )
2
=
1 6x = 72
1
10x× +24
2
1 x = 12
20x 1
=1 So, present age of Shivam = 2x + 4
5x+24
20x = 5x + 24 = 28 years

ENGLISH LANGUAGE

71. (c); Refer the last few lines of the 5th paragraph of the
81. (d); Use “hanged” in place of “hung”
passage, “The answer to this is automation fueled
by robust Machine learning training models and 82. (e); There is no error in the sentence.
artificial intelligence……………. provide
83. (b); There is an error in part (b) of the sentence. Use
troubleshooting in real-time.”
“to succeed” in place of “to be success” because
72. (a); Refer the 3rd paragraph of the passage, after “determined” we use “to infinitive”.
“Furthermore, with the lack of data talent an
ensure business operations are accessible for a 84. (e); There is no error in the sentence.
range of employees.” 85. (c); Replace 'were' with was'.
73. (d); Refer to the 2nd line of the 2nd paragraph of the When two subjects are joined by 'not only...but
passage, “In today's climate any loss or disruption also', the verb must agree with the second subject.
to data can have a huge business impact.” The correct sentence should be: “Not only the
students but also the principal was laughing at the
74. (d); Statement (1) is correct. Refer the 4th paragraph
joke he cracked.
of the passage, “The cloud offers great promise for
The same rule applies when two subjects are
developers the use of multiple server locations”
joined by 'or', the verb must agree with the second
75. (c); If something such as a plan or a project goes full subject.
steam ahead, it progresses quickly. For e.g. Krish and his brothers were there.
76. (c); Embrace means accept (a belief, theory, or (Second subject is 'plural') Mohan or Sohan is
change) willingly and enthusiastically hence it is responsible for this. (Second subject is 'singular')
similar in meaning to adopt. 86. (a); “Except” which means “not including” is the word
77. (a); Ensure means make certain that (something) will which should fit the
occur or be the case hence confirm is the word blank.
which is most similar in meaning.
87. (d); “Rejoiced” which means feel or show great joy or
78. (b); Massive means exceptionally large hence delight will fit the blank most appropriately.
insignificant is the word which is most opposite in
meaning. 88. (d); Here, “permits” which means “officially allow
(someone) to do something” perfectly fits in the
79. (a); There is an error in part (a) of the sentence in given blank making the sentence grammatically
place of “believe” we will use “believed”. correct and contextually meaningful. Hence,
80. (c); There is an error in part (c) of the sentence. option (d) is the most suitable answer choice.
In place of “am” we will use “was” because the Coverts: a thicket in which game can hide.
events of the sentences are in past.

35 www.bankersadda.com | www.sscadda.com | www.careerpower.in | Adda247 App


SBI CLERK 2021: The Success Guide
Implements: put (a decision, plan, agreement,
94. (e); The correct sequence for the given sentences is
etc.) into effect. EADBC.
Emits: produce and discharge (something,
especially gas or radiation). 95. (c); The correct sequence for the given sentences is
Deserts: what a person deserves with regard to EADBC.
reward or (more usually) punishment. 96. (c); “Were” should be used in place of “wear”
89. (b); Here, “encouraged” which means “give support, 97. (a); Correct Spelling is Tired
confidence, or hope to (someone)” perfectly fits in 98. (e); All the given words are correct.
the given blank making the sentence
grammatically correct and contextually 99. (c); The correct spelling is “enrage”
meaningful. Hence, option (b) is the most suitable 100.(c); From the given highlighted words, ‘Candedates’ is
answer choice. misspelled, correct spelling is ‘Candidates’ Hence,
Fascinated: strongly attracted and interested. option (c) is the correct answer.
Discouraged: having lost confidence or
enthusiasm; disheartened. Harvested: gather (a
crop) as a harvest.
Invited: make a formal or polite request for
(something) from someone
90. (b); ‘Surrounded’ should be followed by ‘By’ or ‘with’,
hence it is an incorrect option. Also, option (d) is
grammatically incorrect and option (c) doesn’t
make a meaningful sentence. So, option (b) is the
only option which makes the sentence both
meaningfully and grammatically correct. Hence,
option (b) is the most suitable answer choice.
91. (b); The correct sequence for the given sentences is
EADBC.
92. (a); The correct sequence for the given sentences is
EADBC.
93. (d); The correct sequence for the given sentences is
EADBC.

36 www.bankersadda.com | www.sscadda.com | www.careerpower.in | Adda247 App


SBI CLERK 2021: The Success Guide

SBI Clerk Prelims 2018| Memory Based Paper |


Questions & Solutions
REASONING ABILITY

Direction (1–3): Read the information carefully and Direction (7 – 8): Read the information given below and
answer the questions: answer the questions.
All the given members belong to the same the family. J is
A company ABC printed different number of books in the brother of L. J is the only son of R. W is the father-in-
different years 1947, 1956, 1987, 1998, 2002 such that law of L. D is the maternal grandfather of P, who is a male.
number of books printed are not same in any year. 66 Q is the only son of W. W is the grandfather of N and C is
books were printed in an odd numbered year which is not the daughter of N.
1947.The number of books printed in 1947 is 10 less than
that printed in 1987. 59 books were printed in an year 7. How L is related to C?
before the year in which 61 books are printed but not (a) Mother (b) Son (c) Brother
(d) Father (e) None of these
immediate before. The number of books printed in 2002 is
2 more than that printed in 1998. 8. How is P related to N?
(a) Mother (b) Son (c) Brother
1. How many books were printed in 1947?
(d) Father (e) None of these
(a) 56 (b) 66 (c) 63
(d) 61 (e) none of these Direction (9 – 11): Read the information carefully and
answer the question:
2. What is the difference between the number of books Point U is 10m north of point Q. Point T is 10m east of point
printed in 1956 and 2002? U. Point S is 15m south of point T. Point P is 20m south of
(a) 7 (b) 10 (c) 8 point Q. Point R is 25m east of point P. Point L is 15m east
(d) 4 (e) none of these of point S. Point M is the midpoint of point U and P.
3. In how many years the number of books printed are 9. What is the distance between point L and R?
more than that printed in 1998? (a) 10m (b) 15m (c) 5m
(a) two (b) one (c) none (d) 20m (e) 25m
(d) three (e) four 10. In which direction is point T with respect to P?
4. How many words can be formed from the 1st, 6th, 8th (a) north-west
and 9th letter of a word ‘EMANICIPATE’ by using each (b) south-west
letter once in the word? (c) south-east
(a) two (b) one (c) none (d) north-east
(d) three (e) more than three (e) none of these
11. Which of the following points are inline?
5. If all the letters in the word FIGURES are arranged in
(a) P, R, S (b) Q, M, L (c) U, S, T
alphabetical order from left to right in such a way that
(d) M, S, L (e) Q, S, L
vowels are arranged first followed by consonants, then
how many letters are there in between U and R after Directions (12–16): Read the following information
the arrangement? carefully and answer the given questions.
(a) two (b) one (c) none Twelve persons sitting in two rows. D, E, F, K, L and M
(d) three (e) four sitting in row-1 and facing north. S, T, U, X, Y and Z sitting
in row-2 and facing south direction. E sits third from one of
6. If in the number 39682147, 1 is added to each of the the extreme ends. S sits second to the left of the one who
digit which is less than five and 1 is subtracted from faces E. Only three persons sit between S and T. K sits
each of the digit which is greater than five then how somewhere right of M. More than three persons sit
many digits are repeating in the number thus formed? between X and T. F faces one of the immediate neighbours
(a) two (b) one (c) none of T. Z sits second to the right of Y. The one who faces L sits
(d) three (e) four third to the left of U. D faces S.

37 www.bankersadda.com | www.sscadda.com | www.careerpower.in | Adda247 App


SBI CLERK 2021: The Success Guide
12. Who among the following faces K? 19. Who sits immediate to the left of Ravi, who is sitting in
(a) T (b) S (c) X row. All the persons who are sitting in a row facing
(d) Y (e) none of these north direction?
I. There are only two persons sit between Sahil and
13. Who among the following faces the immediate Geeta. More than three persons sit to the left of
neighbor of M? Geeta.
(a) Z (b) K (c) D II. Not more than 8 persons can sit in a row. Ravi sits
(d) L (e) None of these second to the left of Sahil. Diya sits 6 places away
14. Four of the following five from a group, which among from Geeta.
the following does not belong to this group? 20. What is the code of ‘right’ in a certain code language?
(a) T, E (b) U, D (c) Y, L I. The code of ‘every right to reject’ is ‘%47 *32 $53
(d) Z, E (e) Z, K *95’,
II. The code of ‘never reject right turn’ is ‘%62 %47
15. Who among the following faces the one who sit to the
$51 *32’.
immediate left of Y?
(a) U (b) D (c) X 21. Find the number of boys and number of girls in the
(d) Z (e) none of these row?
I. R sits 18th from left end of the row and Y sits 11th
16. How many persons sit between M and D? from the right end of the row. R and Y interchange
(a) one (b) two (c) three their positions, after interchanging the position R’s
(d) five (e) four position is 20th from left end.
Direction (17): Five people A, B, X, Y, and Z live on five II. Total 43 students are in the row and all are facing
is same direction.
different floors of a building (such as ground floor
numbered as 1 and top is numbered as 6). There are three Directions (22–26): Study the following arrangement
floors between A and B. X lives one of the floors above Y. carefully and answer the questions given below:
B5R1@EK4F7©DAM2P3%9HIW8*6UJ$V
17. Who among the following lives on third floor? Q#
(a) B (b) A (c) X
(d) Z (e) Cannot be determined 22. Which of the following is the fifth to the left of the
seventeenth from the left end of the above
18. Which of the following elements should come in a place arrangement?
‘?’ ? (a) D (b) W (c) *
AB3 CE6 FI10 JN15 ? (d) 4 (e) None of these
(a) OT20 (b) TO21 (c) OT21
23. Which of the following is exactly in the middle between
(d) TS21 (e) None of these
D and U in the above arrangement?
Directions (19–21): Each of the questions below consists (a) % (b) H (c) 9
of a question and two statements numbered I and II given (d) 3 (e) None of these
below it. You have to decide whether the data provided in 24. Four of the following five are alike in a certain way
the statement are sufficient to answer the question. Read based on their position in the above arrangement and
both the statements and so form a group. Which is the one that does not belong
Given answer: to that group?
(a) If the data in statement I alone are sufficient to answer (a) R1E (b) F7D (c) M23
the question, while the data in statement II alone are (d) 9HW (e) UJ6
not sufficient to answer the question.
25. How many such symbols are there in the above
(b) If the data in statement II alone are sufficient to answer
arrangement each of which is immediately preceded
the question, while the data in statement I alone are
by a number but not immediately followed by a
not sufficient to answer the question. consonant?
(c) If the data either in statement I alone or in statement II (a) None (b) One (c) Two
alone are sufficient to answer the question. (d) Three (e) More than three
(d) If the data even in both statements I and II together are
not sufficient to answer the question. 26. Which of the following is the tenth to the left end of the
(e) If the data in both statement I and II together are thirteenth from the right end?
(a) F (b) M (c) @
necessary to answer the question.
(d) % (e) 3

38 www.bankersadda.com | www.sscadda.com | www.careerpower.in | Adda247 App


SBI CLERK 2021: The Success Guide
Direction (27–28): Read the information carefully and
Directions (31-35): The following questions are based on
answer the questions:
the six three digits numbers given below:
Eight persons A, B, C, D, E, F, G, H are sitting around a 563 218 732 491 929
circular table facing centre. H faces B. Two persons sit 31. If 2 is subtracted from the second digit of all odd
between F and B. E sits 2nd right to D. F sits 2nd right to C, numbers and 2 is added in the first digit of all even
numbers, then which number is lowest number after
who is one of the immediate neighbors of G. C is not an
the arrangement?
immediate neighbor of B.
(a) 218 (b) 732 (c) 491
27. Who among the following sits 3rd left to F? (d) 929 (e) None of these
(a) D (b) C (c) B 32. If third digit of highest number is divided by the first
(d) A (e) none of these digit of lowest number, then what will be the resultant?
(a) 4 (b) 6 (c) 4.5
28. Who among the following faces A?
(d) 5 (e) None of these
(a) D (b) C (c) B
(d) G (e) none of these 33. If all the digits in each number are arranged in
increasing order, then which number will be the
Directions (29-30): Study the following information highest number after the rearrangement?
carefully and answer the given questions. (a) 218 (b) 732 (c) 491
In a certain code language, (d) 563 (e) None of these
‘good key friends’ is coded as ‘xo pe cm’ 34. How many numbers will be there in the given series in
‘key law found’ is coded as ‘xo og bt’ which addition of first and third digit is greater than
‘data key good’ is coded as ‘tu xo pe’ second digit?
(a) One (b) Two (c) Three
29. Which of the following is the code for ‘good’?
(d) Four (e) None of these
(a) xo (b) pe (c) tu
(d) cm (e) None of these 35. How many numbers will be there in the given series in
which difference of first and third digit is greater than
30. Which of the following word is coded as ‘og’? second digit?
(a) law (b) good (c) found (a) One (b) Two (c) Three
(d) Either (a) or (c) (e) key (d) Four (e) None of these

Quantitative Aptitude

36. Value A is three times of value B. If value of C is 62.5% 39. In a mixture the ratio of milk and water is 7 : 5. When
of difference between value of A and B, then value C is 56 litres of water is added to this mixture the ratio of
what percent of B? milk and water in mixture becomes 7 : 3. Find the
(a) 120% (b) 125% (c) 115%
initial quantity of milk and water?
(d) 110% (e) 105%
(a) 144 liters (b) 120 liters (c) 116 liters
37. A is 40% more efficient than B and both can complete (d) 140 liters (e) 160 liters
a work in 20 days. A and B start work and do it for eight
day. If remaining work is complete by C in 24 days, 40. A man invested Rs 8400 at the rate R% on SI for two
then find in how many days C will complete work years and gets a total interest of Rs. 2016. If he invested
alone? Rs. 600 more amount on half of the previous rate of
(a) 30 days (b) 24 day (c) 20 days interest for two years, then find the interest get by
(d) 16 days (e) 40 days
man?
38. The length of rectangle is thrice the side of square. (a) 960 Rs. (b) 840 Rs. (c) 1080 Rs.
Difference between perimeter of rectangle and square (d) 1020 Rs. (e) 1040 Rs.
is 40 cm. If breadth of rectangle is 8 cm, then find area
of square? Directions (41-45): Data given about total number of
(a) 169 cm2 (b) 121 cm2 (c) 196 cm2 students visited three towns, read the data carefully and
(d) 144 cm2 (e) 256 cm2 answer the question:

39 www.bankersadda.com | www.sscadda.com | www.careerpower.in | Adda247 App


SBI CLERK 2021: The Success Guide
There are 400 students visited three towns A,B and C. Out 51. A boat travels110 km downstream and come back
of total students 20% visited only town A, students visited upstream in total 32 hours, if speed of stream is 62.5%
only town B are 15% less than that of students visited only
less than speed of boat in still water then find the speed
town A. 18% of total students visited only town C. Student
visited town both A & B but not C are 8 less than students of stream?
visited only town B, students visited town B & C but not A (a) 5 km/hr (b) 3 km/hr (c) 7 km/hr
are half the students visited town A & B but not C. Students (d) 8 km/hr (e) 10 km/hr
visited A & C but not B are half the sum of students visited
52. A and B invested total Rs. 20,000 and start a business.
only Town B and only town C together, remaining students
visited all the three towns. If ‘B’ gets total profit of Rs. 4500 and ratio between
profit share of B and total profit is 9 : 16, then find
41. Find the total number of students visited at least two amount invested by A if both A and B invested for same
town?
time period?
(a) 180 (b) 106 (c) 160
(d) 140 (e) 120 (a) 8250 Rs. (b) 8060 Rs. (c) 8450 Rs.
(d) 8560 Rs. (e) 8750 Rs.
42. Total students visited town A is how much more than
53. A 120 meters long train cross a pole in 4.8 sec, then find
total students visited town C?
(a) 48 (b) 40 (c) 38 in what time train will cross a platform, whose length
(d) 36 (e) 30 is three times of the length of the train?
(a) 19.2 sec (b) 18.6 sec (c) 12.8 sec
43. Total students who visited all the three towns is what
(d) 20.4 sec (e) 26.8 sec
percent of total students visited town B & C but not A?
1 2 2
(a) 33 % (b) 14 % (c) 16 % 54. Four years ago, Tina is 18 years younger than Ruchi. If
3 7 3
(d) 66 %
2
(e) 60 %
2 present total age of Ruchi and Tina is 50 years, then
3 3
find the ratio between present age of Tina to present
44. Total students visited only town C is what percent less age of Ruchi?
than total students visited only town A? (a) 9 : 17 (b) 17 : 8 (c) 17 : 9
(a) 10% (b) 12% (c) 15%
(d) 6 : 17 (e) 8 : 17
(d) 8% (e) 18%
45. Find the ratio between total students visited only town 55. A shopkeeper marked up an article 40% above its cost
B & only town C together to students visited only town price and gave two successive discounts of 25% and
A? 10% respectively. If he made a loss of Rs. 352, then find
(a) 4 : 5 (b) 7 : 4 (c) 7 : 2 the cost price of the article?
(d) 7 : 5 (e) 7 : 3 (a) 5600 Rs. (b) 5000 Rs. (c) 6400 Rs.
Direction (46 – 50): What will approximate value come at (d) 6800 Rs. (e) 7200 Rs.
the place of question mark:
11.98 ×15.89 + ∛215.98 ?
46. √288.98
- 51.98 ×2 −52.98 = 0
(a) 485 (b) 459 (c) 559
(d) 594 (e) 694
1.98
47. 12.98 % of 399.98 + √? = of 125.98
2.93
(a) 1024 (b) 1296 (c) 961
(d) 900 (e) 848
48. 567.93 + 455.97 - ? = (27.98)2
(a) 120 (b) 180 (c) 160
(d) 220 (e) 240
3
√?
49. 124.99 × 24.99 + 6.02 × 8.98 = (7.99)2
(a) 6 (b) 8 (c) 4
(d) 5 (e) 7
4.98
50. √?+ 135.98 = 7.89 of 319.98
(a) 3025 (b) 2205 (c) 4096
(d) 4098 (e) 4047

40 www.bankersadda.com | www.sscadda.com | www.careerpower.in | Adda247 App


SBI CLERK 2021: The Success Guide

Directions (56 – 60): Given below bar graph shows total number of books sold by six book stores P, Q, R, S, T and U on
15 April 2015. Read the data carefully and answer the question:
180
160
140
120
100
80
60
40
20
0
P Q R S T U

56. If ratio between books sold by store R on 15 April, (a) 21 (b) 23 (c) 24
2015 and on 15 May, 2015 is 15 : 21. 75% of total books (d) 19 (e) 27
sold on 15 May, 2015 are comic books, then find total
62. (12)2 + (15)2 + ? % of 200 = (22)2 - 5
books sold on 15 May,2015 which are not comic (a) 55 (b) 45 (c) 35
books? (d) 30 (e) 65
(a) 36 (b) 42 (c) 48
(d) 52 (e) 56 63. 145 % of 180 + ? % of 320 = 741
(a) 180 (b) 130 (c) 120
57. Total books sold by store T on 15 April, 2015 is what (d) 140 (e) 150
percent more than total books sold by store U on 15
April, 2015? 64. 46 ÷ 162 × √16 = ?
(a) 80% (b) 60% (c) 40% (a) 56 (b) 60 (c) 64
(d) 70% (e) 80% (d) 72 (e) 68

58. Find difference between average number of books sold 65. (√529 − √289) × 12 = 136 − ?2
by store R & S on 15 April, 2015 and average number (a) 8 (b) 10 (c) 12
of books sold by store Q & U on 15 April, 2015? (d) 16 (e) 20
(a) 40 (b) 25 (c) 35 66. 45% of 160 + ? % of 180 = 19 % of 900
(d) 56 (e) 60 (a) 50 (b) 55 (c) 65
59. Total books sold by store R on 15 April,2015 is what (d) 45 (e) 35
percent less than total book sold by store T on 15 April, 2
67. 13 − 7 = ? +1
1 1
2015? 3
11
4 2
11 11
1
(a) 22 %
1
(b) 20 %
1
(c) 25 % (a) 5 12 (b) 7 12 (c) 4 12
3 3 3 11 11
1
(d) 28 %
1
(e) 33 % (d) 9 12 (e) 11 12
3 3
18×11+232.5
60. Total books sold by store T on 15 May, 2015 are 20% 𝟔𝟖. 17×9−30
=?
more than total books sold by same store on 15 April, (a) 2.4 (b) 4.5 (c) 2.5
2015, then find total number of books sold by T on 15 (d) 3.5 (e) 6.5
May, 2015?
69. 8.4% of 300 + 4. 8% of ? = 102
(a)332 (b) 216 (c) 244
(a) 1200 (b) 800 (c) 600
(d) 264 (e) 316
(d) 1800 (e) 1600
Directions (61-70): What will come at the place of 70. 734 + ? = 12.8 × 64
question mark: (a) 80.2 (b) 85.2 (c) 84.8
61. 40% of 50% of 60% of 1200 = ? + 53 (d) 89.5 (e) 78.2

41 www.bankersadda.com | www.sscadda.com | www.careerpower.in | Adda247 App


SBI CLERK 2021: The Success Guide

English Language

Directions (71-75): In the question given below, some off from cultural and political-economic processes
sentences/phrases are given out of which one maybe or involved in most conceptualizations of globalization, the
may not be grammatically correct. Find the grammatically impacts of globalization processes upon education are
correct sentence/phrase out of the given options. If all of often considered wide-ranging, though many are also
the given sentences/phrases are correct, mark option (e) controversial.
i.e. “All are correct” as your answer. Major trends: From a functionalist perspective, the
globalization of educational systems has been influenced
71. (a) Do I knows you?
by new demands and desires for educational
(b) He gets tough at times.
transferability, of students and educators. In place of
(c) You must has acknowledged.
dichotomous systems in terms of academic levels and
(d) We didn’t knew. credentialing, curriculum, and assessment, increasing
(e) All are correct convergence can be observed today, as it is recognized that
72. (a) Do you know what time it is in Boston? standardization makes movement of people in education
(b) Won't somebody please help me? across societies more readily feasible, and that such
(c) I quit a long time ago. movement of people can __________ education in a number
(d) I had to work hard when I was young. of ways (to achieve diversity, to increase specialization and
(e) All are correct the promotion of dedicated research centers, to enhance
global employability, and so on). Thus, the mobility and
73. (a) She agreed in him that I should go to the meeting. paths of movement of students and academics, for
(b) To tell the truth, I used to smoking two packs a day. education and better life opportunities, have been a rapidly
(c) We agrees to leave soon. expanding area of research.
(d) I hope my last mail didn't sound too harsh.
(e) All are correct 76. Which of the following words is similar in meaning to
associated?
74. (a) Neither her friends nor Mary were in a talkative (a) chisel (b) innate (c) equivocal
mood. (d) eclectic (e) None of these
(b) None of the story he tells is true.
(c) He is a cut above the average college student. 77. Choose the word that appropriately fits in blank given
(d) I don't wanted to play this game anymore. in the passage.
(e) All are correct (a) distinct (b) channelize (c) create
75. (a) Two against one is not a fair fight. (d) enhance (e) influence
(b) She accepts criticism from anyone but her parents. 78. Which of the following is the most appropriate title of
(c) Lend him as much money as he needs. the passage?
(d) Do you think I should go alone? (a) Aspects of contemporary education
(e) All are correct (b) Political affairs affecting education
Directions (76 – 80): Read the following passage carefully (c) Modern Education
and answer the questions given below them. Certain (d) Globalization’s Impact on Education
words/phrases have been printed in bold to help you (e) Achieving diversity among nations
locate them while answering some of the questions. 79. Which of the following sentences is in accordance with
Global and transnational processes and practices have the theme of the passage?
been observed to influence and impact various aspects of (a) Globalization is understood as a process or
contemporary education within many geographical condition of contemporary human life, at the
contexts, and thus the fields of research related to broadest level, rather than a single event or
education and globalization are vast: they are not activity.
contained simply within one field or subfield, but can be (b) Globalization in education cannot be merely
seen to cross sub-disciplinary borders, in policy studies, described as harmful or beneficial, but depends on
curriculum, pedagogy, higher education studies, one’s position, perspective, values, and priorities.
assessment, and more. (c) Global citizenship education has been conceived
As mentioned previously, modern education can itself be by political theorists and educational
seen as one most basic instance of globalization, connected philosophers.
to increased interdependency of communities around the (d) Education held by scholars and political leaders is
world in economic and political affairs first associated a key to enhance the modern human condition.
with imperialism and colonialism, and more recently with (e) Globalization as a contemporary condition or
the capitalist world economy. And as the modern process clearly shapes education around the globe,
educational system cannot be seen as removed or sealed in terms of policies and values.

42 www.bankersadda.com | www.sscadda.com | www.careerpower.in | Adda247 App


SBI CLERK 2021: The Success Guide
80. According to the passage, in what way the education 87. They feared to spare him lest he would report the
field is vast? matter to the king.
(a) As it is not confined to any geographical region. (a) lest he would not report
(b) as the impact of globalization on education is wide (b) lest he should report
ranging. (c) lest he will report
(c) As it has various aspects that increase the (d) lest he shall report
dependency on other communities of the world. (e) no replacement required
(d) both (a) and (b)
88. As soon as he finished reading the collect letter, than
(e) All of the above. he flew into a rage and tore it into pieces.
Directions (81–85): Each of the sentences given below (a) than he flown into a rage
contains a blank. Identify the most suitable alternative (b) then he flown into a rage
among the five given that fits into the blank to make the (c) then he flew into a rage
sentence logical and meaningful. (d) he flew into a rage
(e) no replacement required
81. All this at the time seemed merely strange to Pierre: he
felt he could not _____________its significance. 89. This is perhaps the same car which ran her over
(a) appreciate (b) hold on (c) encrypt while she was returning from school.
(d) grasp (e) overlook (a) that ran her over while she was
(b) which run her over while she was
82. History bears several instances of how ambitions of (c) which run her over when she was
individuals have ruined countries and brought untold (d) that running her over while she was
______________to the people. (e) no replacement required
(a) joys (b) injuries (c) miseries
(d) comforts (e) heartaches 90. If you will set your mind to a goal, you will eventually
83. Labour ______________ is not just good for those on the achieve it.
move but it also has a positive effect on the economy. (a) would set your mind
(a) mobility (b) constancy (c) durabilty (b) shall set your mind
(d) adaptability (e) resilience (c) set your mind
(d) should set your mind
84. Sometimes it is not only a book but the book read with (e) no replacement required
an ________________review of it that gives us an
understanding of a reality. Directions (91–94): Rearrange the following six
(a) alert (b) insightful (c) rational sentences (A), (B), (C), (D), (E) and (F) in the proper
(d) artful (e) obtuse sequence to form a meaningful paragraph then answer the
questions given below them.
85. An _________________ link exists between democracy and (A) The cow made a request to the tiger, “I have got a calf
public freedom. at home and the calf is very hungry.”
(a) determinable (b) avoidable (B) The tiger was convinced with her plea and allowed her
(c) inextricable (d) involved to return to the village.
(e) elaborated (C) All the cows became scared and started running
Directions (86–90): In the questions given below, there is towards the village, but one cow was left behind.
a sentence in which one part is given in bold. The part given (D) The cow said to the tiger, “I will come back after giving
in bold may or may not be grammatically correct. Choose my milk to the calf. I promise.”
the best alternative among the four given which can (E) Some cows were grazing in a forest. A tiger pounced
replace the part in bold to make the sentence upon them.
grammatically correct. If the part given in bold is already (F) “It is impossible. Because if I leave you now, you will
correct and does not require any replacement, choose not come back” said the tiger.
option (e), i.e. “No replacement required” as your answer. 91. Which of the following will be the SECOND sentence
after rearrangement?
86. Despite of the government's protecting, he was the (a) A (b) B (c) C
victim of an assassination attempt which killed him. (d) D (e) E
(a) Despite being the government’s protecting
92. Which of the following will be the FOURTH sentence
(b) Despite being the government’s protection
after rearrangement?
(c) Despite the government’s protection (a) A (b) D (c) C
(d) Despite of the government’s protection (d) F (e) E
(e) No replacement required

43 www.bankersadda.com | www.sscadda.com | www.careerpower.in | Adda247 App


SBI CLERK 2021: The Success Guide
93. Which of the following will be the THIRD sentence (B) Jack could not
after rearrangement? (C) enough money
(a) A (b) B (c) F (D) with us because
(d) D (e) C
(E) he didn't have
94. Which of the following will be the SIXTH (LAST) (a) BACED (b) BAEDC (c) BCADE
sentence after rearrangement? (d) BADEC (e) None of these
(a) A (b) B (c) D
(d) F (e) E 98. (A) Sean had to
Directions (95 – 100): In each question below some (B) pumped because
sentences/phrases are given which are named A, B, C, D (C) he had accidentally
and E. Rearrange the given five sentences/phrases to make (D) have his stomach
a coherent sentence/paragraph. The rearranged sequence (E) eaten some rat poison
of the sentences/phrases will be your answer. If after (a) AECBD (b) ADBCE (c) ABCDE
rearrangement, no option matches, and option (e) i.e. (d) ADEBC (e) None of these
“None of these” is given, mark it as answer.
95. (A) Speaking French 99. (A) Graduated from our
(B) a native speaker (B) English with a native speaker
(C) who hear Tom (C) university has studied
(D) many people (D) for at least two years
(E) think he is (E) every student who has
(a) DCBAE (b) DCEAB (c) DABCE (a) DECAB (b) EBCAD (c) EACBD
(d) DCAEB (e) None of these
(d) EADCB (e) None of these
96. (A) That Spanish is
(B) one of the most 100. (A) About how he
(C) some people say (B) telling Mary the story
(D) in the world (C) being eaten by cannibals
(E) beautiful languages (D) Michael remembers
(a) EDABC (b) BDACE (c) CABED (E) had escaped from
(d) CBEDA (e) None of these
(a) AEBDC (b) DBAEC (c) DCAEB
97. (A) Go to Boston
(d) ACEDB (e) None of these

Solutions
REASONING ABILITY
Solutions (1–3)
6. (b);The given number is- 39682147
Years Books After applied operation- 48573256
1947 56 So, only digit 5 is repeated in the number thus
1956 59 formed.
Solutions (7–8)
1987 66
1998 61
2002 63

1. (a); 2. (d); 3. (a);


4. (a); The 1st, 6th , 8th and 9th letter of a word
‘EMANICIPATE’ are E, C, P, A
So, the words formed are PACE and CAPE
5. (a); The given word is- FIGURES 7. (e);
After the rearrangement- EIUFGRS 8. (c);
So, the letters between U and R will be two.

44 www.bankersadda.com | www.sscadda.com | www.careerpower.in | Adda247 App


SBI CLERK 2021: The Success Guide
Solutions (9–11): Solutions (27–28):

9. (b); 10. (d); 11. (d); 27. (e);


28. (d);
Solution (12–16):
Directions (29–30):
Word Code
friends cm
key xo
good pe
law/found og/bt
data tu
12. (d); 13. (a); 14. (d);
29. (b);
15. (b); 16. (c);
30. (d);
17. (e); Solution (31–35):
Floors Persons
31. (a); 543 418 932 471 909
5 A/B
4 X/Z 32. (c); 9/2= 4.5
3 X/Z/Y 33. (d);356 128 237 149 299
2 Z/Y 34. (d);563 218 732 929
1 B/A
35. (b);218 732
18. (c); OT21
19. (e); From both the statements I and II we can find that
Diya sits immediate to the left of Ravi.

20. (d);right can be coded either %47 or *32.


21. (d);
Solutions (22–26)
22. (a); 5th to left of 17th from left= 17-5= 12th from left = D
23. (c); 9
24. (e); UJ6
25. (d);1@, 3%, 8*
26. (a);

45 www.bankersadda.com | www.sscadda.com | www.careerpower.in | Adda247 App


SBI CLERK 2021: The Success Guide
Quantitative Aptitude
36. (b);Lets A is 3x and B is x 43. (d);Required percentage = 30 × 100 = 66 3 %
20 2
62.5 5
C = (3x – x)× 100 = 2x × 8 =1.25x
80 −72
1.25x 44. (a); Required percentage = × 100 = 10%
Required percent = x
× 100 = 125% 80
68+72
37. (e); A and B ratio of efficiency = 140x : 100x 45. (b);Required ratio = =7:4
80
= 7x : 5x 12 ×16 + ∛216 ?
Total work = 20 (7x + 5x) = 240x units 46. (d); √289
- 52 ×2 −53 = 0
A and B is eight days = 8 × (7x + 5x) = 96x units 192+6 ?
- 51 = 0
240x −96x 144x 17
Efficiency of C = = 24 = 6x units/day 198 ?
240x
24 − =0
17 51
C alone = = 40 days 594 − ?
6x
51
= 0 ⇒ ? = 594
38. (d);Lets length of rectangle is 3x meters and side of
2
square x meters 47. (a); 13 × 4 + √? = 3 × 126
2(3x + 8) – 4x = 40 52 + √? = 84
6x + 16 – 4x = 40
√? = 84 − 52
2x = 24 ⇒ x = 12 cm
Area of square = a2 √? = 32 ⇒ ? = 1024
Area of square = 122 = 144 cm2 48. (e); 568 + 456 - ? = (28)2
39. (a); Let milk and water in mixture be 7x and 5x liters ? = 1024 – 784 ⇒ ? = 240
respectively 3
√?
7x+56 7 49. (b);25 × √125 + 6 × 9 = (8)2
5x
=3 3
35x – 21x = 168 ⇒ x = 12 liters 5 × √? = 64 − 54
3
initial mixture = (7 + 5) × 12 = 144 liters √? = 2 ⇒ ? = 8
2016 ×100 5
40. (c); R = 50. (c); √?+ 136 = of 320
8400 ×2 8
R = 12% √? = 200 − 136
12
New rate = = 6% √? = 64 ⇒ ? = 4096
2
(8400+600) ×6×2
interest = 100
= 1080 Rs. 51. (b);Let speed of boat in still water and speed of current
be 8x km/hr and
Solutions (41 – 45): 3x km/hr respectively
20
Total students visited only town A = 400 × = 80 ATQ –
100
85 110 110
Total students visited only town B = 80 × = 68 +
8x+3x 8x−3x
= 32
100
Total students visited only town C = 400× 100 = 72
18 32x = 32 ⇒ x = 1 km/h
Speed of stream = 3 × 1 = 3 km/hr
Total students visited town A & B but not C = 68 -8
= 60 52. (e); Let investment of A is Rs. X then investment of B is
Total students visited town B & C but not A Rs. (20,000 – x)
60 16
= 2 = 30 Total profit = 4500 × 9 = 8000 Rs.
68+72
Total students visited A & C but not B = = 70 ATQ –
2 x 8000−4500
Total students visited all the three towns (20000−x)
= 4500
= 400 – (80 + 68 +72+60+30+70) = 20 16x = 140000 ⇒ x = 8750 Rs.
41. (a); Total number of students visited at least two
53. (a); Let speed of train is S m/s
towns 120
= 60 + 30 + 70 + 20 = 180 S = 4.8 = 25 m/s
Let train will cross the platform in T sec
42. (c); Total students visited town A 120+120×3
= 80 + 60 + 70 + 20 = 230 25 =
T
Total students visited town C 25 × T = 480
480
= 72 + 30 + 70 + 20 = 192 T= ⇒ T = 19.2 sec
25
Required difference = 230 – 192 = 38

46 www.bankersadda.com | www.sscadda.com | www.careerpower.in | Adda247 App


SBI CLERK 2021: The Success Guide
54. (e); Four years ago – 61. (d);
40
×
50
×
60
× 1200 = ? +125
100 100 100
(T – 4) + 18 = (R −4) 2 1 3
× × × 1200 = ? +125
R - T = 18 … (i) 5 2 5
Given, R + T = 50 … (ii) ? = 144 – 125 ⇒ ? = 19
From (i) and (ii) ?
62. (a); 144 + 225 + × 200 = 484 - 5
2R = 68 100

R = 34 years 2 × ? = 479 − 369 ⇒ ? = 55


T = 16 years 145
63. (e); 100 × 180 +
?
× 320 = 741
16 100
Required ratio = 34 = 8 : 17 ?
261 + 100 × 320 = 741
55. (c); Let cost price of article = 100x Rs. ?
× 320 = 741 − 261
100
Marked price of article = 140x Rs. 480×100
75 90 ?= ⇒ ? = 150
Selling price of Article = 140x × 100 × 100 320

= 94.5x Rs. 64. (c); (22)6 ÷ (24)2 × 4 = ?


ATQ – 24 × 22 = ? ⇒ ? = 64
100x – 94.5x = 352
65. (a); (23 – 17) × 12 = 136 − ?2
5.5x = 352 ⇒ x = 64 Rs.
?2 = 136 – 72 ⇒ ?2 = 64 ⇒ ? = 8
Cost price = 64× 100 = 6400 Rs.
45 ? 19
56. (b);Total books sold on 15 May,2015 which are not 66. (b); × 160 + × 180 = × 900
100 100 100
?
comics by store R 72 + 100 × 180 = 171
120 25 1
= 15
× 21 × 100 = 168 × 4 = 42 ?
× 180 = 171 – 72
100
?
57. (a); Required percentage =
180 −100
× 100 100
× 180 = 99
100
99 ×100
=
80
× 100 = 80% ?= 180
⇒ ? = 55
100
8−3−6
58. (b);Average number of books sold by store R & S on 15 67. (c); ? = (13 – 7 – 1)+ 12
April, 2015 1 11
120+90
? = 5 – 12 ⇒ ? = 4 12
= = 105
2 198+232.5
Average number of books sold by store Q & U on 68. (d); =?
153−30
60+100 430.5
15 April, 2015 = 2
= 80 ?= ⇒ ? = 3.5
123
Required difference = 105 − 80 = 25 8.4 4.8
69. (e); × 300 + ×? = 102
180−120 1 100 100
59. (e); Required percentage = × 100 = 33 % 4.8
180 3 ×? = 102 − 25.2
100
60. (b);Total number of books sold by store T on 15 May, ?=
76.8 ×100
⇒ ? = 1600
120 4.8
2015 = 180 × = 216
100
70. (b); ? = 819. 2- 734 ⇒ ? = 85.2

English Language

71. (b);Out of all the given sentences/phrases only the 73. (d);Out of all the given sentences/phrases only the
second sentence is grammatically correct. Hence, fourth sentence is grammatically correct. Hence,
option (b) is the correct answer. option (d) is the correct answer.
72. (e); All of the given sentences/phrases are 74. (c); Out of all the given sentences/phrases only the
grammatically correct. Hence, option (e) is the third sentence is grammatically correct. Hence,
correct answer. option (c) is the correct answer.
75. (e); All of the given sentences/phrases are
grammatically correct. Hence, option (e) is the
correct answer.

47 www.bankersadda.com | www.sscadda.com | www.careerpower.in | Adda247 App


SBI CLERK 2021: The Success Guide
85. (c); Option (c) is the right choice.
76. (e); None of the given words is similar in meaning to
‘associated’. Inextricable means inseparable.
Determinable means able to be definitely decided
77. (d);The paragraph talks about the impact of or ascertained.
globalization on education. Movement of people Elaborated means presented in further detail.
across the communities for standardizing
education is ultimately improving education. 86. (c); The most appropriate phrase to replace the phrase
Hence ‘enhance’ is the most appropriate word to given in bold is “Despite the government’s
be filled in the blank making the sentence protection”. It is to be noted that “despite” does not
contextually correct. take any preposition with it. It is always followed
78. (d);“Globalization’s Impact on Education” is the most with a noun, pronoun or a gerund. Since option (c)
appropriate title of the passage. is in the precise grammatical syntax, it becomes
the most suitable answer choice.
79. (e); Sentence (e) is in accordance with the theme of the
passage which is summarizing the whole passage. 87. (b);The most appropriate phrase to replace the
Other sentences are not delivering the exact incorrect bold phrase in the sentence is “lest he
explanation. should report”. It is to be noted that ‘lest’ is
80. (e); All the sentences are correct in context of the followed by ‘should’ and not ‘would’. Therefore,
passage. option (b) becomes the most suitable answer
choice.
81. (d);The most appropriate word that would fill the 88. (d);The most suitable phrase that should replace the
blank is ‘grasp’ which means seize and hold firmly. phrase given in bold is “he flew into a rage”. It is to
Encrypt means conceal data in (something) by
be noted that after ‘As soon as, So long as, As long
converting it into a code. All the other words do not
as’ we do not use ‘than’. Except for option (d) none
fill the blank appropriately; hence option (d) is the
other options are in absolute grammatical syntax.
most appropriate choice.
82. (c); The correct option is (c). ’Miseries’ is a plural form Hence, it becomes the most viable answer choice.
of noun and it is the only word that fits the blank 89. (a); The most appropriate phrase to replace the given
appropriately. phrase in bold to make the sentence grammatically
Miseries mean feelings of great physical or mental correct is “that ran her over while she was”. It is to
distress or discomfort. be noted that we will use ‘that’ in place of ‘which’
Heartaches are the emotional anguish or grieves, after ‘the same’ if the verb is clear. Here the word
typically caused by the loss or absence of someone ‘was’ shows that the verb is clear. Hence,
loved. considering the given rationale option (a) becomes
83. (a); Option (a) is the correct choice. the most suitable answer choice.
Mobility means the ability to move or be moved 90. (c); The appropriate phrase to replace the phrase in
freely and easily. bold to make the sentence grammatically correct is
Constancy means the quality of being enduring and “set your mind”
unchanging. It is to be noted that this sentence is an example of
Durability means the ability to withstand wear, First Conditional Sentences. In the case of First
pressure, or damage. Conditional Sentences, we use the simple present
Adaptability means the quality of being able to tense in the if-clause and simple future tense in the
adjust to new conditions. main clause—that is, the clause that expresses the
Resilience means the capacity to recover quickly likely outcome. This is how we indicate that under
from difficulties; toughness.
a certain condition (as expressed in the if-clause),
84. (b);The correct option is option (b).The statement a specific result will likely happen in the future.
talks about an insightful review of a book that gives Hence, considering the given option (c) becomes
an understanding of a reality. the most suitable answer choice.
Alert means cautious. Solutions (91–94): The proper sequence of sentences to
Insightful means perceptive.
form a meaningful paragraph will be ECAFDB.
Rational means based on or in accordance with
reason or logic. 91. (c); 92. (d); 93. (a); 94. (b);
Artful means clever or skillful, especially in a crafty 95. (d);The correct sequence of the parts to form a
or cunning way. grammatically correct and contextually
Obtuse means annoyingly insensitive or slow to meaningful sentence is DCAEB. Hence, option (d)
understand. is the most suitable answer choice.

48 www.bankersadda.com | www.sscadda.com | www.careerpower.in | Adda247 App


SBI CLERK 2021: The Success Guide
96. (c); The correct sequence of the parts to form a 100. (b);The correct sequence of the parts to form a
grammatically correct and contextually grammatically correct and contextually
meaningful sentence is CABED. Hence, option (c) is meaningful sentence is DBAEC. Hence, option (b)
the most suitable answer choice. is the most suitable answer choice.
97. (d);The correct sequence of the parts to form a
grammatically correct and contextually
meaningful sentence is BADEC. Hence, option (d)
is the most suitable answer choice.
98. (b);The correct sequence of the parts to form a
grammatically correct and contextually
meaningful sentence is ADBCE. Hence, option (b)
is the most suitable answer choice.
99. (c); The correct sequence of the parts to form a
grammatically correct and contextually
meaningful sentence is EACBD. Hence, option (c) is
the most suitable answer choice.

49 www.bankersadda.com | www.sscadda.com | www.careerpower.in | Adda247 App


SBI CLERK 2021: The Success Guide

SBI Clerk Prelims 2016| Memory Based Paper |


Questions & Solutions
REASONING ABILITY
Directions (1-5): To answer these questions study 8. Statements : S > M = Z > T < Q > V
carefully the following arrangement of letters, digits and Conclusions : I. V = S II. Q > M
symbols.
9. Statements : T < U = V  S > P  Q
M 7 Σ 8 L P @ ? 6 N B T Y 3 2 = E $ 4 9 © G H 5. Conclusions : I. S > T II. V  Q
1. How many such letters are there in the arrangement 10. Statements : M  N > R > W, E = J > L  W
each of which is immediately followed by a number? Conclusions : I. E > W II. M > L
(a) Three (b) Four (c) One
Directions (11-15): The following questions are based on
(d) Two (e) None of these
the five three – digit numbers given below:
2. How many such symbols are there in the arrangement 684 512 437 385 296
each of which is immediately preceded by a number? 11. If 2 is added to the first digit of each of the numbers
(a) Two (b) Three (c) Four how many numbers thus formed will be divisible by
(d) Nil (e) None of these three?
(a) None (b) One (c) Two
3. If all the symbols are deleted from the arrangement, (d) Three (e) None of these
then which of the following will be fourth to the left of
the 17th element from the left end? 12. If all the digits in each of the numbers are arranged in
descending order within the number, which of the
(a) 9 (b) E (c) 2
following will be the highest number in the new
(d) Y (e) None of these arrangement of numbers?
4. '78' is related to `P ? 6' and ' ?N' is to`T32'in the same (a) 684 (b) 385 (c) 296
way as'2E'is to…….. in the arrangement. (d) 437 (e) None of these
(a)4©H (b)49G (c)4©G 13. What will be the resultant number if the second digit of
(d)9GH (e)None of these the second lowest number is divided by the third digit
of the highest number?
5. If all the numbers are deleted from the arrangement (a) 2 (b) 3 (c) 0
then which of the following will be fifth to the right of (d) 1 (e) 4
the 13th element from the right end?
14. If 1 is added to the first digit and 2 is added to the last
(a) B (b) N (c) Y
digit of each of the numbers then which of the
(d) T (e) None of these following numbers will be the second highest number?
Directions (6-10): In these questions, a relationship (a) 385 (b) 684 (c) 437
between different elements is shown in the statements(s). (d) 296 (e) 512
The statements are followed by two conclusions. Give 15. If in each number the first and the second digits are
answer interchanged then which will be the highest number?
(a) if only conclusion I is true. (a) 296 (b) 512 (c) 437
(b) if only conclusion II is true. (d) 684 (e) 385
(c) if either conclusion I or II is true. Directions (16-17): Study the following information
(d) if neither conclusion I nor II is true. carefully and answer the questions given below:
(d) if both conclusions I and II are true. P is to the north of Q and S is to the east of P, who is to the
south of W. T is to the west of P.
6. Statements : A > B  C < D, C = E > G
16. Who among the following is towards south of W and
Conclusions : I. D > E II. B > E
north of Q?
7. Statements : P  Q > M  N, Q = S (a) P (b) T (c) S
Conclusions : I. S > P II. N < S (d) Q (e) None of these

50 www.bankersadda.com | www.sscadda.com | www.careerpower.in | Adda247 App


SBI CLERK 2021: The Success Guide
17. W is in which direction with respect to T? 26. Who among the following lives on the topmost floor?
(a) North (b) Northeast (c) Southwest (a) I (b) Q (c) P
(d) West (e) None of these (d) L (e) None of these
Directions (18-22): Study the following information 27. Which of the following combinations is true?
carefully and answer the questions given below: (a) First floor-S (b) Fourth floor-R
Dhondu, Chintu, Titu, Chiku, Sonu, Monu, Bittu and Sonty (c) Third floor-M (d) Sixth floor-I
are sitting around a circular table facing the center. Sonty (e) None of these
is third to the right of Titu and second to the left of Sonu.
Chintu is not an immediate neighbor of Sonty and Titu. 28. How many pairs of letters are there in the word (in
Monu is second to the right of Chiku and is an immediate forward direction) APPLICATION, each of which have
neighbor of Titu. Bittu is not the neighbor of Sonu. as many letters between then in the word as they have
18. Who among the following is second to the right of Titu? between then in the English alphabet?
(a) Sonty (b) Bittu (c) Monu (a) One (b) Two (c) Three
(d) Sonu (e) None of these (d) Four (e) None of these.

19. Who among the following is an immediate neighbor of 29. In a certain coding system, PAPER is written as PERPA
Sonty and Sonu? and SUBJECT is written as JECTSUB, what should be
(a) Dhondu (b) Chintu (c) Titu the code for COUNCIL?
(d) Bittu (e) None of these (a) NCILCOU (b) LICNOUC (c)NCOUCIL
(d) NLICUOC (e) NILCCOU
20. In which of the following pairs the second person is
sitting on the immediate right of the first person? 30. In a certain code language ‘lu ja ka hu’ means ‘will you
(a) Dhondu, Sonty (b) Titu, Chiku meet us’, ‘lu ka hu pa means ‘will you sold us’. Then
(c) Bittu, Sonty (d) Sonu, Sonty What is the code of ‘meet’ in this code language?
(e) Monu, Titu (a) ja (b) lu (c) ka
21. Who among the following is second to the left of (d) hu (e) cannot be determined
Chintu? 31. In a certain code language COMBINE is written as
(a) Titu (b) Sonty (c) Monu XLNYRMV. How will TOWARDS be written in that code
(d) Dhondu (e) None of these language?
22. Who among the following is opposite Chiku? (a) FLDZIWJ (b) GLDZIWH (c) GLEZJWH
(a) Dhondu (b) Bittu (c) Sonty (d) FLEZIWH (e) None of these
(d) Sonu (e) None of these
32. 37 girls are standing in a row facing the school building
Directions (23-27): Study the following information Ayesha is fifteenth from the left end. If she is shifted six
carefully to answer the given questions: places to the right what is her position from the right
Seven neighbours S, P, L, Q, R, M and I live on different end?
floors in the same building having seven floors numbered (a) 16th (b) 21st (c) 20th
one to seven. (The first floor is numbered one, the floor (d) 18 th (e) None of these
above it is numbered two and so on and the topmost floor
is numbered as seven.) 33. X's mother is the mother-in-law of the father of Z. Z is
Three persons live between I and M. M lives on the floor the brother of Y while X is the father of M. How is X
immediately above S, who does not live on an odd- related to Z?
numbered floor. P is neither live on odd number nor (a) Paternal uncle (b) Maternal uncle
topmost floor. I does not live on the first floor. (c) Cousin (d) Grandfather
Two persons live between R and S. Q lives neither on the (e) Brother-in-law
first floor nor on the fourth floor.
34. If A is a brother of B, C is the sister of A, D is the brother
23. Who lives on the floor just above M? of E, E is the daughter of B, F is the father of C. than who
(a) L (b) P (c) Q is the uncle of D?
(d) R (e) None of these (a) A (b) C (c) B
24. How many persons live between L and P? (d) None of these (e) Can’t be determined
(a) None (b) One (c)Two
35. A said to B that B’s mother was the mother-in-law of
(d) Three (e) Can’t be determined
25. Which of the following pairs live on the first floor and A’s mother. How is A’s mother related to B’s mother?
the topmost floor respectively? (a) Daughter-in-law (b) Mother-in-law
(a) L, Q (b) Q, P (c) I, Q (c) Sister (d) Aunt
(d) L, I (e) Can’t be determined (e) Sister-in-law

51 www.bankersadda.com | www.sscadda.com | www.careerpower.in | Adda247 App


SBI CLERK 2021: The Success Guide
QUANTITATIVE APTITUDE

Directions (36-40): What should come in the place of 44. The total number of Assaults and Murders together in
question mark (?) in the following questions? Bihar is what per cent of the total number of crimes in
7 3 that state?
36. 5 of 58 + 8 of 139.2 =? (a) 29.82% (b) 39.82% (c) 25%
(a) 133.4 (b) 137.2 (c) 127.8 (d) 21.82% (e) 25.5%
(d) 131.6 (e) None of these 45. Find the difference between the number of various
crimes committed in Bihar and that in Rajasthan.
37. 12% of 555 + 15% of 666 =?
(a) 105 (b) 98 (c) 145
(a) 166.5 (b) 167.5 (c) 168.5
(d) 139 (e) 104
(d) 169.5 (e) None of these
Directions (46-50): What should come in the place of
38. 84368 + 65466 − 72009 − 13964 =?
question mark (?) in the following questions?
(a) 61481 (b) 62921 (c) 63861
(d) 64241 (e) None of these 46. 4376 + 3209 – 1784 + 97 = 3125 + ?
(a) 2713 (b) 2743 (c) 2773
39. 337.8 × 331.2 ÷ 335 = 33 × 33? (d) 2793 (e) 2737
(a) 2.8 (b) 3 (c) 3.2
(d) 4 (e) 6 47. √? + 14 = √2601
? 324
(a) 1521 (b) 1369 (c) 1225
40. 529
= ? (d) 961 (e) 1296
(a) 404 (b) 408 (c) 410
48. 85% of 420 + ?% of 1080 = 735
(d) 414 (e) 416
(a) 25 (b) 30 (c) 35
Directions (41-45): Study the following table carefully (d) 40 (e) 45
and answer the given questions: 7 5 1
49. of of of 3024 = ?
The number of various crimes, as supplied by national 3 4 9
crime record, reported in different states in the year (a) 920 (b) 940 (c) 960
2012-13. (d) 980 (e) 840
50. 30% of 1225 – 64% of 555 = ?
(a) 10.7 (b) 12.3 (c) 13.4
(d) 17.5 (e) None of these
51. How many litres of water should be added to a 30 litre
mixture of milk and water containing milk and water
in the ratio of 7 : 3 such that the resultant mixture has
40% water in it?
(a)5 (b)2 (c)3
(d)8 (e) 7
41. The total number of various crimes in HP is
(a) 37803 (b) 38903 (c) 37903 52. The S.I on certain sum of money for 15 months at rate
(d) 36903 (e) 37003 of 7.5% per annum exceed the S.I on same sum at
12.5% per annum for 8 months by Rs 3250 find sum?
42. Find the ratio of Stalking and Assault in UP to Theft and (a)160000 (b)20000 (c)170000
Criminal Trespass in Haryana. (d) 18000 (e) 312000
(a) 28 : 51 (b) 21 : 52 (c) 52 : 21
(d) 14 : 55 (e) 55 : 14 53. 4 men and 3 women finish a job in 6 days, and 5 men
and 7 women can do the same job in 4 days. How long
43. Find the approximate average of Murder and Theft in will 1 man and 1 woman take to do the work?
all the eight states together. 2
(a) 22( ) days
1
(b) 25( ) days
1
(c)5 ( ) days
(a) 1141 (b) 1132 (c) 1311 7
7
2 7

(d) 941 (e) 1021 (d) 12(22) days (e) None of these

52 www.bankersadda.com | www.sscadda.com | www.careerpower.in | Adda247 App


SBI CLERK 2021: The Success Guide
54. A and B started a business with initial investments in 62. The numerator of a fraction is decreased by 25% and
the ratio 5 : 7. If after one year their profits were in the the denominator is increased by 250%. If the resultant
ratio 1 : 2 and the period for A’s investment was 7 6
fraction is 5, what is the original fraction?
months, B invested the money for
22 24 27
(a) 6 months (b) 2 ½ months (c) 10 months (a) 5 (b) 5 (c) 6
(d) 4 months (e) 7 months (d)
28
(e)
30
5 11
55. An army lost 10% its men in war, 10% of the remaining
due to diseases and 10% of the rest were disabled. 63. What would be the area of a rectangle whose area is
Thus, the strength was reduced to 729000 active men. equal to the area of a circle of radius 7 cm?
Find the original strength. (a) 77 cm2 (b) 154 cm2 (c) 184 cm2
(a) 1000000 (b) 1200000 (c) 1500000 (d) 180 cm 2 (e) 150 cm 2

(d) 1800000 (e) none of these


56. What is the difference between the compound 64. In a village three people contested for the post of
interests on Rs. 5000 for 1 years at 4% per annum village Pradhan. Due to their own interest, all the
compounded yearly and half-yearly? voters voted and no one vote was invalid. The losing
(a)2 (b)3 (c)4 candidate got 30% votes. What could be the minimum
(d)8 (e)none of these absolute margin of votes by which the winning
57. The speeds of John and Max are 30 km/h and 40 km/h. candidate led by the nearest rival, if each candidate got
Initially Max is at a place L and John is at a place M. The an integral per cent of votes?
distance between L and M is 650 km. John started his (a) 4 (b) 2 (c) 1
journey 3 hours earlier than Max to meet each other. If (d) 3 (e) None of these
they meet each other at a place P somewhere between
L and M, then the distance between P and M is : 65. The price of an article is first increased by 20% and
(a) 220 km (b) 250 km (c) 330 km later on the price were decreased by 25% due to
(d) 320 km (e) None of these reduction in sales. Find the net percentage change in
final price of Article.
58. The average weight of boys in a class is 30 kg and the
(a) 20% (b) 18% (c) 38%
average weight of girls in the same class is 20 kg. If the
average weight of the whole class is 23.25 kg, what (d) 10% (e) None of these
could be the possible strength of boys and girls Directions (66–70): What will come in the place of the
respectively in the same class? question mark (?) in the following number series?
(a) 14 and 26 (b) 13 and 27 (c) 17 and 27
(d) 19 and 21 (e) 14 and 27 66. 48, 23, ?, 4.25, 1.125
(a) 10.5 (b) 10 (c) 2.5
59. A profit of 8% is made by selling a shirt after offering a
discount of 12%. If the marked price of the shirt is (d) 11 (e) None of the above
Rs.1080, find its cost price 67. 2, 15, 41, 80, 132, ?
(a) 890 (b) 780 (c) 880 (a) 197 (b) 150 (c) 178
(d) 900 (e) none of these
(d) 180 (e) None of the above
4
60. The difference between of a number and 45% of the
5 68. ?, 15, 75, 525, 4725, 51975
number is 56. What is 65% of the number? (a) 5 (b) 10 (c) 8
(a) 96 (b) 104 (c) 112 (d) 6 (e) None of the above
(d) 120 (e) None of these
69. 4, 19, 49, ?, 229
61. A man can row 24 km upstream and 54 km
downstream in 6 hours. He can also row 36 km (a) 75 (b) 109 (c) 65
upstream and 48 km downstream in 8 hours. What is (d) 169 (e) None of the above
the speed of the man in still water? 70. 840, ?, 420, 140, 35, 7
(a) 18.75 kmph (b) 19.25 kmph (a) 408 (b) 840 (c) 480
(c) 17.65 kmph (d) 15.55 kmph
(d) 804 (e) None of the above
(e)22.75 kmph

53 www.bankersadda.com | www.sscadda.com | www.careerpower.in | Adda247 App


SBI CLERK 2021: The Success Guide
ENGLISH LANGUAGE

Directions (71-80): Read the passage carefully and (a) Never trust anyone without confirming it yourself.
answer the questions given below it. Certain words/ (b) One who is not hard working is not loved by the
phrases have been given in bold to help you locate them employees.
(c) Good relation with your employees is most
while answering some of the questions.
important in your life.
One day all the employees reached the office and they saw
(d) Your relationship with your friend is the most
a big advice on the door on which it was written: important one that can influence your happiness.
“Yesterday the person who has been hindering your (e) The most important relationship you can have is
growth in this company passed away. We invite you to join the one you have with yourself.
the funeral in the room that has been prepared in the 73. What was inside the coffin?
gym”. In the beginning, they all got sad for the death of one (a) The corpse of the employee who died.
of their colleagues, but after a while they started getting (b) The coffin was empty.
curious to know who was that man who hindered the (c) A note on which there were instructions.
growth of his colleagues and the company itself. (d) There was a mirror inside the coffin.
(e) Photos of every employee.
The excitement in the gym was such that security agents
74. What was the reason of employee’s excitement?
were ordered to control the crowd within the room. The (a) As they were expecting a bonus or promotion.
more people reached the coffin, the more the excitement (b) Because the employee who hindered their growth
heated up. Everyone thought: “Who is this guy who was died.
hindering my progress? Well, at least he died!” One by one (c) As they were happy they will succeed now since no
the thrilled employees got closer to the coffin, and when one will hinder their growth now.
they looked inside it they suddenly became speechless. (d) They were excited because what they found inside
the box was completely opposite of what they
They stood nearby the coffin, shocked and in silence, as if
expected.
someone had touched the deepest part of their soul. There (e) They were curious to know the identity of the man
was a mirror inside the coffin: everyone who looked inside who hindered the growth of his colleagues.
it could see himself.
75. Why everyone was speechless and shocked?
There was also a sign next to the mirror that said: “There is (a) They were shocked as the employee who died was
only one person who is capable to set limits to your the most hardworking one.
growth: it is YOU.” You are the only person who can (b) As they never expected that their growth was
revolutionize your life. You are the only person who can actually hindered by one of their own.
influence your happiness, your realization and your (c) As they found a mirror inside the coffin.
success. You are the only person who can help yourself. (d) As one of the employees died.
(e) They were shocked to find that coffin was empty.
Your life does not change when your boss changes, when
your friends change, when your partner changes, when Directions (76-78): Choose the word/group of words
your company changes. Your life changes when YOU which is MOST SIMILAR in meaning to the word / group of
words printed in bold as used in the passage.
change, when you go beyond your limiting beliefs, when
you realize that you are the only one responsible for your 76. THRILLED
life. “The most important relationship you can have is the (a) Tedious (b) Monotonous (c)Delight
(d) Dull (e) Boring
one you have with yourself.’’
77. INFLUENCE
71. Which of the following is true in context of the passage? (a) Insignificance (b) Domination
(a) The crowd gathered in the office for the funeral. (c) Underwhelming (d) Triviality
(b) The coffin was empty. (e) Weakness
(c) The employee who died was hindering everybody’ 78. REVOLUTIONIZE
growth. (a) Harmony (b) Calm (c) Uprising
(d) Only a person is responsible for his own success. (d) Stagnation (e) Obedience
(e) The crowd loved the employee who died.
72. What can we learn from this passage?

54 www.bankersadda.com | www.sscadda.com | www.careerpower.in | Adda247 App


SBI CLERK 2021: The Success Guide
Directions (79-80): Choose the word/group of words 86. Suppose, if you (a)/ were left alone to (b)/ live on a
which is MOST OPPOSITE in meaning of the word/ group deserted island(c)/ what would you do? (d)/ No error
of words printed in bold as used in the passage. (e).
79. FUNERAL 87. He wondered that what (a)/would be the next move
(a) Nativity (b) Burial (c) Cremation (b)/of his opponents who had (c)/ vowed to see him
(d) Entombment (e) Inhumation dislodged from power? (d)/No error (e).

80. HINDERED 88. The nation should (a)/ be grateful to (b) the armed
(a) Hamper (b) Inhibit (c) Impede forces for (c)/ protecting it. (d)/No error (e).
(d) Retard (e) Expedite 89. For so many years(a)/it is almost his habit (b)/ to go to
the bed (c)/ at 10 pm daily. (d)/No error (e).
Directions (81-85): Rearrange the following six sentence
(A), (B), (C), (D), (E) and (F) in the proper sequence to form 90. To the men (a)/ who worked so hard (b)/ in the project
a meaningful paragraph and then answer the questions the news was(c)/ profound disappointing. (d)/No
given below. error (e).
A. The man who bought it immediately put it up for Directions (91-95): In each sentence below four words
auction. have been printed in bold which are numbered (a), (b), (c)
B. And he went on to list the many qualities of the animal. and (d) One of these words may be misspelt or
At the end of his sales talk a man said he would give 40 inappropriate in the context of the sentence. Find out the
dinars for it. wrongly spelt or inappropriate word. The number of that
C. "Look at this fine animal!" he shouted to passersby. word is the answer. If all the words are correctly spelt and
"Have you ever seen a better specimen of a donkey? are appropriate the answer is (e) i.e. ‘All correct.'
See how clean and strong it is!"
91. Rising (a)/prices (b)/of food grains will have an
D. Nasruddin Hodja took his donkey to the market place adverse impac (c)/on developing countries. (d)/ All
and sold it for 30 dinars. correct (e)
E. Another man offered 50. A third offered 55.
F. Hodja who was watching was amazed at the interest 92. To deal effectively (a)/with a crisis (b)/quick
everyone was showing in the donkey. decisions (c)/are requited. (d)/All correct (e)

81. Which of the following should be the FIRST sentence 93. The IT Company has succeeded (a)/in achieving high
after rearrangement? growth rate despite (b)/facing (c)/several
(a)F (b)E (c)B (d)/problems. All correct (e)
(d) A (e)D 94. On an average (a)/there are very fern persons
willingly (b)/to take on (c)/such responsibility.
82. Which of the following should be the SECOND sentence
(d)/All correct (e)
after rearrangement?
(a)A (b)B (c)C 95. More than halve (a)/the budget (b)/has been spent
(d)E (e) D (c)/on modernizing (d)/the factory/All correct (e)
83. Which of the following should be the THIRD sentence Directions (96-100): In the following passage, some of the
after rearrangement? words have been left out, each of which is indicated by a
(a)E (b)F (c)D number. Find the suitable word from the options given
(d)C (e)B against each number and fill up the blanks with
appropriate words to make the paragraph meaningfully
84. Which of the following should be the FOURTH complete.
sentence after rearrangement? Jack went to work for a Farmer and (96) a penny. But
(a) B (b) G (c)F while returning home, he dropped it in a brook. At home,
(d) E (e) C his mother (97) him and told him to put his earnings in his
85. Which of the following should be the LAST (SIXTH) pocket the next time.
sentence after rearrangement? The next day, he worked for a cow keeper, who (98) him a
(a)C (b)A (c)B jar of milk. Jack tried to put the jar of milk in his pocket and
(d)D (e)F spilled it everywhere. Once again, his mother rebuked him.
She told him that he
Directions (86-90): Read each sentence to find out should have carried it on his shoulders. The next day, Jack
whether there is any grammatical or idiomatic error in it. was given a donkey. He carried the donkey on
The error, if any, will be in one part of the sentence. The his shoulders. Now, the king had a daughter who never
number of that part is the answer. If there is ‘No error’, the laughed. (99) Jack carrying the donkey on his shoulders,
answer is 5). (Ignore errors of punctuation, if any.) she (100) laughing for the first time.

55 www.bankersadda.com | www.sscadda.com | www.careerpower.in | Adda247 App


SBI CLERK 2021: The Success Guide
96. (a) Collected (b)Earned (c)Had 99. (a)Detecting (b) Seeing (c)Watching
(d) Accumulated (e)Make
(d)Recognizing (e) Sawing
97. (a) Praised (b)complimented (c) Scolded
(d) lambaste (e)Criticize 100. (a)Start (b)Stopped (c)commence
98. (a) Sold (b)Provide (c) Made (d)Initiated (e) Began
(d) Gave (e)Issued

Solutions

REASONING ABILITY
Direction (1-5): 17. (b);
1. (a): M 7, Y 3, H 5 Direction (18-22)
2. (b):7 Σ , 2 =, 9 ©
3. (b):4th to the Left - 17th from the left=13th from the
left=E
4. (c): 2E….4©G
5. (d):5th to the right – 13th from the right= 8th from the
right=T
Direction (6-10):
6. (a);D > C = E(True) B ≥ C = E(False)
7. (b);S=Q≥P(False) S=Q>M≥N(True)
18. (b); 19. (a); 20. (c);
8. (d);V = S(False) Q > M(False) 21. (d); 22. (c);
9. (a);S ≥ V = U > T(True) V ≥ Q(False) Directions (23-27)
10. (a); E = J > L≥ W(True) M ≥ N > R > W≤ L(False) Floor Person
Direction (11-15): 7 I
6 Q
11. (b);Only 385 will be divisible by 3 when added 2 on
5 R
first digit of each number.
4 P
12. (c); 864 521 743 853 962
3 M
13. (a); 8÷4=2 2 S
14. (e); 786 614 539 487 398 1 L
15. (a); 864 152 347 845 926 23. (b); 24. (c); 25. (d);
Direction (16-17): 26. (a); 27. (c);
28. (a);

29. (a); 30. (a);


16. (a);

56 www.bankersadda.com | www.sscadda.com | www.careerpower.in | Adda247 App


SBI CLERK 2021: The Success Guide
31. (b);Opposite Letter according to English alphabetical 34. (a);
series.

32. (e; L=15th sifted 6 place to right 15+6=21st R=(37-


21)+1=17
33. (b);
35. (a);

QUANTITATIVE APTITUDE

36. (a); 81.2 + 52.2 = 133.4 42 = 27 + 3𝑥


3𝑥 = 15 ⇒ 𝑥 = 5
37. (a); 66.6 + 99.9 = 166.5
52. (e); Let sum = 𝑥
38. (c); 149834 − 85973 = 63861 15 1 8 1
x× 12 × 7.5 × 100 − 𝑥 × 12.5 × 12 × 100
39. (b); 33?+1 = 337.8+1.2−5
∴?= 9−5−1 = 3 = 3250
3 𝑥
𝑥 − 12 = 3250
40. (d); ?2 = 529 × 324 32
9𝑥−8𝑥
∴? = 23 × 18 = 414 = 3250
96
41. (d); Total No. of crimes in HP = 36903 𝑥 = 96 × 3250
𝑥 = 312000
42. (b); Ratio = 210 : 520
= 21 : 52 53. (a); Let men’s 1 day work = 𝑥
Let women’s 1 day work = 𝑦
2117+14220
43. (e); = 1
4𝑥 + 3𝑦 = 6 ………………(i)
16
16337
= 16 1
5𝑥 + 7𝑦 = 4 ……………..(ii)
≈ 1021 ∴ By solving eqn. (i) and (ii) —
628 1 5
44. (b); Required = × 100 𝑦 = 78 𝑥 = 156
1577
1 1 156 2
= 39.82% ∴Required days = 1 5 = 2+5 = = 22 7
+ 7
78 156 156
45. (c); Required difference = 1577 – 1432 = 145
54. (c); Let B invested money for 𝒙 months.
46. (c); ?= 7682 – 4909 = 2773 ∴ 5×7∶7×𝑥 =1∶2
47. (b); √? = √2601 − 14 = 51 − 14 = 37 ∴ 35 ∶ 7𝑥 = 1 ∶ 2
? = 1369 7𝑥 = 35 × 2
85 𝑥
𝑥 = 10 months
48. (c); 100 × 420 + 100 × 1080 = 735
55. (a); Let initial men = 100
⇒ x = 35 10
Lost in war = 100 × 100 = 10
49. (d); 980 10
Lost in diseases = 100 × 90 = 9
50. (b); ? = 367.5 – 355.2 81
Disables= 100 × 90 = 8.1
= 12.3
∴ Remaining men = 72.9
51. (a); Let Required quantity = 𝑥 When 72.9 remaining total men = 100
21 3
= When 729000 remaining total men = 1000000
9+𝑥 2

57 www.bankersadda.com | www.sscadda.com | www.careerpower.in | Adda247 App


SBI CLERK 2021: The Success Guide
24 54
56. (a); When compounded yearly, + =6
𝑢 33
Student = 200 𝑢 = 5.5
When compounded half – yearly ∴ Speed of the man in still water
r = 2%, n = 2 33+5.5
= 2 = 2
38.5

 interest = 202 = 19.25 kmph


difference = 202 – 200 = 2 25𝑥
𝑥− 6
57. (d); speed of john = 30 km/hr 62. (d); 100
250𝑦 =5
𝑦+
Speed of max = 40 km/hr 75𝑥
100
6
Let distance b/w p and m = x km =
350𝑦 5
650−𝑥 𝑥
= 40 + 3 75x = 420y
30 𝑥 420
7x = 2240 =
𝑦 75
x = 320 km 𝑥
=
28
𝑦 5
58. (b); Let Boys = 𝑥 22
Girls = 𝑦 63. (b); Required area = ×7×7
7
(30𝑥+20𝑦)
∴ 23.25 = = 154 cm2
𝑥+𝑦
23.25𝑥 + 23.25𝑦 = 30𝑥 + 20𝑦 64. (b); Since winning candidate and his rival got 70% of
6.75𝑥 = 3.25𝑦 total votes.
𝑥 13 ∴ 34 + 36 = 70
=
𝑦 27
Required minimum margin = 36 − 34 = 2
88 100
59. (c); Cost Price = 1080 × × = 880 25×20
100 108 65. (d); Net Change = 20 – 25 – 100
4
60. (b); = 80% =0–5–5
5
= – 10%
(80 – 45) = 35% of the no. = 56
56
65% of the no. = 35 × 65 = 104 66. (a); ÷ 2 – 1 = 23, ÷ 2 – 1 = 10.5, ÷ 2 – 1 = 4.25……
24 54 67. (a); 2 + 13 = 15, 15 + 26 = 41, 41 + 39 = 80, 80 + 52 =
61. (b) + = 6 … … … . (1) 132
𝑢 𝑣
36 48
+ = 8 … … . . (2) ∴ 132 + 65 = 197
𝑢 𝑣
eqn (1) × 3 – eqn (2) × 2 68. (a); 51975 ÷ 11 = 4725, 4725 ÷ 9 = 525,
72
+
162
= 18 525 ÷ 7 = 75, 75 ÷ 5 = 15,
𝑢
72
𝑣
96
15 ÷ 3 = 5
+ = 16
𝑢
66
𝑣 69. (b); 4 + 15 = 19, 19 + 30 = 49, 49 + 60 = 109,
=2 109 + 120 = 229
𝑣
𝑣 = 33
70. (b); 840 ÷ 1 = 840, 840 ÷ 2 = 420, 420 ÷ 3 = 140,
Put in the eqn (1)
140 ÷ 4 = 35, 35 ÷ 5 = 7

ENGLISH LANGUAGE

71. (d);Refer to the last paragraph, ''you are the only 75. (c); Refer to the second paragraph it is explained there
person who can influence your happiness, your that they were shocked to see the mirror.
realization and your success.''
76. (c); Thrill means cause (someone) to have a sudden
72. (e); Refer to the last paragraph, “the most important feeling of excitement and pleasure hence delight is
relationship you can have is the one you have with
most similar in meaning.
yourself.''
73. (d);Refer to second paragraph, ''there was a mirror 77. (b);influence means the capacity to have an effect on
inside the coffin: everyone who looked inside it the character, development, or behaviour of
could see himself''. someone or something, or the effect itself hence
74. (e); they were excited about the identity. domination is the word most similar in meaning.

58 www.bankersadda.com | www.sscadda.com | www.careerpower.in | Adda247 App


SBI CLERK 2021: The Success Guide
78. (c); Revolutionize means a forcible overthrow of a 86. (a); Remove 'suppose'.
government or social order, in favour of a new 87. (a); Remove 'that'.
system hence uprising is the word most similar in
meaning. 88. (e); No error.

79. (a); Funeral means a ceremony or service held shortly 89. (c); Remove 'the'
after a person's death, usually including the 90. (d);Use ‘profoundly’ in place of ‘profound’.
person's burial or cremation hence nativity is the
91. (c); Change ‘impac’ into ‘impact’
word most opposite in meaning.
80. (e); hindered means make it difficult for (someone) to 92. (d); Change ‘requited’ into ‘required’
do something or for (something) to happen hence 93. (e);
expedite is the word most opposite in meaning.
94. (b); Change ‘willingly’ into ‘willing’
Directions (81-85);The correct sequence is
DACBEF 95. (a); Change ‘halve’ into ‘half’
81. (e); D 96. (b);
82. (a); A 97. (c);
83. (d);C 98. (d);
84. (a); B 99. (b);
85. (e); F 100.(e);

59 www.bankersadda.com | www.sscadda.com | www.careerpower.in | Adda247 App


SBI CLERK 2021: The Success Guide

Most Important Questions | Quantitative Aptitude |


SBI Clerk Prelims 2021
Directions (1-10): What will come in place of question 12. 5, 15, 45, 135, ?, 1215
mark (?) in the following questions. (a) 415 (b) 395 (c) 410
4
4 of25
(d) 405 (e) 400
5 3 1
1. ( 5
) ÷ (4 of32 + 7 of 21) =? of 49
48 13. 90, 96, 102, 108, 114, ?
(a) 3.5 (b) 3 (c) 2.5 (a) 116 (b) 124 (c) 118
(d) 4 (e) 5 (d) 122 (e) 120

2. √? of 6 + 20% of 95 = 2 of 62
1 14. 389, 380, 370, 359, ?, 334
(a) 347 (b) 345 (c) 351
(a) 3 (b) 4 (c) 5
(d) 350 (e) 348
(d) 6 (e) 7
5 3 9 15. 1, 3, 6, ?, 18, 29
3. (3 of 6 5 of 11) +?2 = 45 (a) 10 (b) 11 (c) 9
(a) 5 (b) 7 (c) 4 (d) 12 (e) 8
(d) 8 (e) 6
16. 280, 295, 325, 370, 430, ?
4 14 3 4
4. ( × ÷ 2) − ( of ? ) = − 3 (a) 515 (b) 525 (c) 505
7 5 10 5
(a) 10 (b) 8 (c) 9 (d) 490 (e) 520
(d) 11 (e) 12 17. 4, 2, 3, 7.5, ?, 118.125
4
5. 4 5 + 2 15 −
1 27 1
= 2 5 ÷ 3 ×? (a) 24.25 (b) 28.25 (c) 27.25
2
5 (d) 25.25 (e) 26.25
(a) 9 (b) 1 (c) 2
1 18. 18, 25, 30, ?, 42, 49
(d) 3 (e) 9 (a) 37 (b) 35 (c) 39
4
of25
(d) 41 (e) 43
3 1
6. (5 ) ÷ (432 − 202 + of 21) × (82) =? of
64 7 64 19. 1, 2, 4, 8, ?, 32
(a) 50 (b) 45 (c) 35 (a) 32 (b) 24 (c) 12
(d) 30 (e) 40 (d) 16 (e) 20
7. 55% of 900 + 70% of 1050 = ? % of 3000 20. 121, ?, 169, 196, 225, 256
(a) 41 (b) 42 (c) 43 (a) 148 (b) 144 (c) 140
(d) 44 (e) 45 (d) 136 (e) 132
8. 73823 − 34156 + 4756 + 6758 − 9849 = 41499 − Directions (21-25): What approximate value will come in
160−? place of question mark (?) in the following questions. (You
(a) 5 (b) 7 (c) 4 are not expected to calculate the exact value)
(d) 8 (e) 6
21. 40.02% of 601 – 249.97 = ? – 69.98% of 910
5599 3773 88
9. × 2036 × 49 =? −6 2 (a) 607 (b) 627 (c) 637
1331
(a) 44 (b) 46 (c) 48 (d) 617 (e) 647
(d) 50 (e) 52 22. 42001 ÷ 60 × 29.95 = ? × 41.99
1
10. 84 × 4 ÷ 21 +? = 147 × 21 − 21
2 7 20 (a) 540 (b) 520 (c) 500
(d) 460 (e) 480
(a) 2 (b) 1 (c) 0
(d) 3 (e) 4 23. (42.02)2 + (6.98)2 − (27.02)2 = (33.01)2 −?
(a) 1 (b) 2 (c) 3
Directions (11-20): What should come in place of
(d) 4 (e) 5
question mark (?) in the following questions.
699.97 11 121
11. 0.5, 1, 1.5, ?, 0.75, 0 24. 52
÷ 207.99 × 77.02 = ?
(a) 2 (b) 1.5 (c) 1.25 (a) 400 (b) 410 (c) 390
(d) 1 (e) 0.75 (d) 420 (e) 380

60 www.bankersadda.com | www.sscadda.com | www.careerpower.in | Adda247 App


SBI CLERK 2021: The Success Guide
25. 29.97% 𝑜𝑓 ? + √399.81 = (14.98)2 + 31.99 (c) if x<y
(a) 750 (b) 730 (c) 760 (d) if x ≤y
(d) 790 (e) 830 (e) if x = y or no relation can be established between x and
y.
Directions (26-30): Find the wrong term in the following
number series questions. 31. I. 𝑥 2 + 5𝑥 + 6 = 0 II. 𝑦 2 + 9y + 14 = 0
26. -5, -10, -15, -30, -45, -90, -180 32. I. x 2 − 18x + 45 = 0 II. y 2 + 12y − 45 = 0
(a) -10 (b) -30 (c) -180 33. I. 9x 2 + 11x + 2 = 0 II. 8y 2 + 6y + 1 = 0
(d) -45 (e) -5
34. I. 6x² + 5x + 1 = 0 II. 4y² – 15y = 4
27. 5, 10, 30, 120, 600, 3000, 25200
(a) 10 (b) 600 (c) 30 35. I. x 2 + 3x = 0 II. x 2 + y = 10
(d) 3000 (e) 25200 Directions (36-40): In each of the following questions,
28. -12, -6, 2, 6, 12, 18, 24 two equations (I) and (II) are given. Solve the equations
(a) 2 (b) 6 (c) -6 and mark the correct option:
(d) 18 (e) 12 (a) if x>y
(b) if x≥y
29. 599, 591, 580, 569, 557, 544, 530
(c) if x<y
(a) 599 (b) 557 (c) 530
(d) if x ≤y
(d) 591 (e) 544
(e) if x = y or no relation can be established between x and
30. 700, 710, 675, 690, 660, 670, 640 y.
(a) 710 (b) 675 (c) 660
36. I. x 2 − 25x + 156 = 0 II. y 2 − 29y + 210 = 0
(d) 690 (e) 670
37. I. x 2 = 196 II. y = √196
Directions (31-35): In each of the following questions,
two equations (I) and (II) are given. Solve the equations 38. I. x 2 + 12x + 35 = 0 II. y 2 + 14y + 48 = 0
and mark the correct option:
39. I. 3x² + 23x + 30 = 0 II. y² + 15y + 56 =0
(a) if x>y
(b) if x≥y 40. I. x 2 + 17x + 72 = 0 II. y 2 + 13y + 42=0
Directions (41-45): Pie-chart given below shows percentage distribution of total income of Sandeep in six different
months. Study the data carefully and answer the following questions.

Total income = 15,000

Jan
Jun 8%
Feb
24%
12%

Mar
16%
May
20%
Apr
20%

41. Income of Sandeep in the month of Jan and April 42. Income of Sandeep in May and Jun together is how
together is what percent less than income of Sandeep much more than the income of Sandeep in Feb and
in the month of Mar and Jun together? March together? (in Rs.)
(a) 20% (b) 30% (c) 40% (a) 1500 (b) 1800 (c) 1200
(d) 50% (e) 70% (d) 2400 (e) 2700

61 www.bankersadda.com | www.sscadda.com | www.careerpower.in | Adda247 App


SBI CLERK 2021: The Success Guide
43. Which month shows the highest percent increment in (a) 115.2° (b) 158.4° (c) 144°
income as compare to previous month? (d) 100.8° (e) 129.6°
(a) Feb (b) March (c) April 45. Sandeep’s average income in starting four months
(d) May (e) Both (b) and (c) from the given six months is how much less than
Sandeep’s average income from last four months in the
44. Income in the month of March and April together given six months? (in Rs.)
(a) 300 (b) 600 (c) 900
makes how much central angle of the total?
(d) 1200 (e) 1500
Directions (46-50): Given line graph shows the details of number of cars sold by three different Showrooms P, Q and R
in five different months and answer the questions accordingly.

600

550

500

450
P
400
Q
350 R

300

250

200
Jan Feb Mar Apr May

46. Total cars sold by showroom Q in February and March 50. If in june, numbers of cars sold by showrooms P, Q and
together is what percent of cars sold by showroom R in R is 20% , 25% and 30% respectively more than that of
February and March together? cars sold in march by all the respective showrooms,
(a) 72.5% (b) 76.25% (c) 81.25% then find total cars sold by all the 3 showrooms
(d) 84.75% (e) 77.5% together in june ?
47. Find the difference between average numbers of cars (a) 1644 (b) 1686 (c) 1584
sold by the showroom P in all months together to the (d) 1728 (e) 1782
average number of cars sold by the showroom Q in all
Directions (51-55): Deepak, Dharam and Shivam invested
the months together ?
in partnership for one year. Ratio of investment of Deepak,
(a) 38 (b) 32 (c) 34
(d) 28 (e) 24 Dharam and Shivam for first 6 months, next four month
and for remaining time is 3:2:3, 2:5:3 and 4:3:3
48. Find the average number of cars sold by all the 3 respectively. Amount invested by Deepak in first 6 months,
showrooms in march month? Dharam in next four month and by Shivam in remaining
(a) 460 (b) 440 (c) 480
time is Rs.1500, Rs. 2000 and Rs. 900 respectively. Total
(d) 420 (e) 490
difference between profit share of Dharam and Shivam
49. Find the respective ratio of total numbers of cars sold together and Deepak and Dharam together is Rs. 450.
by showroom P in March, April and May together to the
total number of cars sold by showroom R in January, 51. Total investment of Deepak is approximately what
February and march together? percent of total investment of Shivam in one year?
(a) 7 : 8 (b) 8 : 9 (c) 8 : 7 (a) 96% (b) 95% (c) 97%
(d) 9 : 8 (e) 9 : 7 (d) 92% (e) 99%

62 www.bankersadda.com | www.sscadda.com | www.careerpower.in | Adda247 App


SBI CLERK 2021: The Success Guide
52. What is profit share of Dharam after one year? (a) Rs. 900 (b) Rs. 600 (c) Rs. 800
(a) Rs.7110 (b) Rs. 6570 (c) Rs. 7020 (d) Rs. 400 (e) Rs. 500
(d) Rs. 6560 (e) Rs. 7220
53. What is the ratio of investment made by Deepak for 4 55. Investment made by Deepak for 2 months is how much
months to investment made by Shivam for 2 months? percent more or less than investment made by Shivam
(a) 5:7 (b) 6:7 (c) 4:5
(d) 8:9 (e) 3:2 for 6 months?
54. What is the difference between investment made by (a) 20% more (b) 25% less (c) 25% more
Dharam for 6 months and 4 months together and total (d) 20% less (e) None of these.
investment made by Shivam?
Directions (56-60): The given bar graph shows the total no. of students of 5 different schools and no. of boys from each
school.
Study the graph carefully and answer the following questions.

5000
4500
4000
3500
3000
2500
2000
1500
1000
500
A B C D E

Total students Boys

56. What is the ratio between no. of boys of school B and Directions (61-65): The table given below shows the no.
no. of girls of school C? of books published by 4 different publishers in 4 months.
(a) 4 ∶ 3 (b) 1 ∶ 1 (c) 5 ∶ 4 Study the data and answer the following questions.
(d) 3 ∶ 4 (e) 4 ∶ 5
Month
57. No. of girls of school B and C together is what percent February March April May
Publisher
of total students of school A?
A 2000 2400 1800 2500
(a) 150% (b) 125% (c) 100%
(d) 66.67% (e) 75% B 1500 1850 2000 2100
C 1750 2000 2250 2400
58. What is the average no. of boys in school A, B, C and E?
D 1200 1350 800 1250
(a) 1800 (b) 2250 (c) 2300
(d) 1950 (e) 2875 61. What is the average no. of books published by A in all
59. Girls in school A and B together are what percent the given months?
more/less than girls of school B and D together? (a) 1740 (b) 2275 (c) 2050
(a) 60% (b) 50% (c) 40% (d) 2175 (e) 2250
(d) 70% (e) 80%
62. Books published by B in February and March together
60. No. of boys in school B and E together are how much is what percent more/less than that by C in March and
more/less than girls in school A, C and D together? April? (approximate)
(a) 500 (b) 1000 (c) 1500 (a) 21% (b) 24% (c) 16%
(d) 2000 (e) 0 (d) 12% (e) 27%

63 www.bankersadda.com | www.sscadda.com | www.careerpower.in | Adda247 App


SBI CLERK 2021: The Success Guide
63. Find the ratio between books published by C to D in all 72. The ratio of speed of boat in still water to speed of
given months. stream is 11 : 1. If the boat takes 1 hour more to cover
(a) 23 : 45 (b) 24 : 43 (c) 42 : 23 220 km upstream than to cover same distance in still
(d) 41 : 25 (e) 23 : 42 water. Then find the speed of boat in still water.
64. Find the revenue obtained by B in March is how much (a) 22 km/hr (b) 18 km/hr (c) 15 km/hr
more/less than that by D in same month, if selling price (d) 20 km/hr (e) 25 km/hr
of book is Rs 120 and all books are sold. (Note – cost
price and selling price of each book is same for all 73. In how many ways the word ‘INTICINCO’ can be
publishers) written so that ‘T’ always comes at end.
(a) Rs 50,000 (b) Rs 40,000 (c) Rs 55,000 (a) 720 (b) 1680 (c) 5040
(d) Rs 70,000 (e) Rs 60,000 (d) 1024 (e) 210
65. Books published by A in April is what percent of book 74. Harsh borrow Rs. 400 at 10% rate of interest. He paid
published by C in March? Rs.200 and Rs.64 at the end of 1st and 2nd year
1000
(a) 9 % (b) 90% (c) 10% respectively. Then find how much money he will pay at
(d)
100
% (e) 75% the end of 3rd year to clear his debt?
9 (a) Rs. 200 (b) Rs. 240 (c) Rs. 264
66. Ratio of speed of a bus to a car is 6 : 7. They start from (d) Rs. 220 (e) Rs. can’t be determined.
the same point and move towards the same direction.
After four hours distance between them is 28 km. Find 75. A container has mixture of water and acid in which
the time in which car will cover 196 km. water is 40% out of total mixture of 50 lit. if 30 lit of
(a) 6 hours (b) 4 hours (c) 4.5 hours the mixture is taken out and 50 lit of another mixture
(d) 2 hours (e) 8 hours of water and acid is added and in second mixture acid
is 40%, then find ratio of water to acid in final mixture.
67. Veer can do a work in x days. While Sameer can do the
same work in (x + 4) days. Ratio of work done by Veer (a) 16:19 (b) 8:7 (c) 19:16
in 3 days to work done by Sameer in 4 days is 15 : 16. (d) 7:8 (e) 17:19
Find the value of ‘x’. 76. Manoj is 25% less efficient than Hemant. Vikash and
(a) 24 (b) 18 (c) 12
Hemant working together can complete a task in 16
(d) 20 (e) 16
days and Vikash is half efficient as Hemant, then find in
68. By selling two articles a man earns 15% profit on first how many days Manoj alone can complete the task
article and 10% loss on second article. Find his overall with 150% of his original efficiency?
gain or loss percent if C.P. of both articles were same? 1 2
(a) 21 3 days (b) 21 days (c) 22 3 days
(a) 2% (b) 5% (c) 2.5%
(d) 3% (e) 3.5% (d) 24 days (e) 32 days
69. An alloy ‘A’ made of Copper and Zinc contain 40% 77. Each of two trains – A & B of different length can cross
copper and another alloy ‘B’ having same elements a pole in 5 seconds and when they are moving in same
contain 30% zinc. A new alloy is made by mixing both 1
direction, train – A crosses train – B in 28 3 sec. If sum
alloys which contain 60% copper. Find the ratio of
quantity of alloy A and alloy B in the new alloy? of their length is 0.85 km, then find the ratio of their
(a) 1 : 2 (b) 3 : 4 (c) 2 : 1 length?
(d) 4 : 3 (e) 5 : 3 (a) 7:11 (b) 10:7 (c) 11:13
(d) 10:13 (e) 5:4
70. Train A crosses a platform of 98 m length in 24 sec.
Another Train B of same length as Train A crosses a 78. When two cards are drawn from a pack of cards then
pole in 12 sec. If speed of train A is 20% more than find what is the probability that both cards are club
speed of train B. Find length of train A. cards or both cards are ace cards?
(a) 80 m (b) 65 m (c) 70 m 14 15 19
(a) 221 (b) 221 (c) 221
(d) 75 m (e) 90 m
9 12
71. The average weight of a class of 45 girls is 53 kg. It was (d) 221 (e) 221
latter found that weight of two girls was read as 49 kg
79. Diagonal of a square whose side is 22√2 cm is bent into
and 57 kg instead of 45 kg and 52 kg. Find the actual
average weight of the class. a circle, find area of the circle?
(a) 54 kg (b) 53.40 kg (c) 50.6 kg (a) 176 cm2 (b) 132 cm2 (c) 154 cm2
(d) 52.80 kg (e) 51.5 kg (d) 220 cm2 (e) 198 cm2

64 www.bankersadda.com | www.sscadda.com | www.careerpower.in | Adda247 App


SBI CLERK 2021: The Success Guide
80. A man purchases three type of item in ratio 2:3:4 and 87. A pipe can fill a cistern in 15 min and another pipe can
their cost price are Rs. 300, Rs. 500 and Rs. 700 fill the same cistern in 60 min. a third pipe can empty
respectively. If he sold first, second and third type of it in 10 min. the first two pipes are kept open for 10
article at 10%, 5% and 4% profit respectively, then min in the beginning and then the third pipe is also
find his overall approximate profit percentage? opened, what is the time taken to empty the cistern?
(a) 7% (b) 2% (c) 4% (a) 45 min (b) 60 min (c) 50 min
(d) 9% (e) 5% (d) 48 min (e) 55 min

81. A, B and C invest in a partnership in ratio 5:3:7 and 88. A boat cover 11.2 km distance in downstream in 48
minutes. If ratio between speed of boat in still water to
investment of A is Rs. 200 less than investment of C.
1 speed of stream is 3 : 1, then find in what time boat will
Partner B invests for 5 th and A and C invest for cover 42 km of distance in downstream and in
1 1
th and th respectively of total time of investment. upstream?
12 15
If profit of B is Rs. 800 more than that of C than find (a) 7 hours (b) 5 hours (c) 9 hours
profit of A. (d) 10hours (e) 3 hours
(a) Rs. 1000 (b) Rs. 2500 (c) Rs. 2800 89. Ratio between age of Veer and Sameer six years ago
(d) Rs. 1400 (e) Rs. 3600 was 3 : 7 and six years hence will be 5 : 9. Find the ratio
between present age of Veer and Sameer?
82. Average of first three no. out of seven no. is 11 more
(a) 2 : 3 (b) 1 : 4 (c) 1 : 3
than average of seven no. and 5th , 6th and 7th no. are 8, (d) 1 : 2 (e) 1 : 5
5 and 29 less than average of seven number
respectively. If 4th no. is 89, then find average of initial 90. Present population of two town A and B is in ratio 8 :
three no. 7. Present population of town B is 7000 and it
2
(a) 80 (b) 70 (c) 81 increased by 20% and 14 % in next two year. Now
7
(d) 91 (e) 61 ratio of population of town B to A is 24 : 25. Find
increment in population of town A.
83. A bag Contain 10 two-rupee coins, 5 ten-rupee coins
(a) 2500 (b) 2000 (c) 3500
and 7 one-rupee coins, If 3 Coins are withdraw at
(d) 3000 (e) 4500
random find the probability of getting minimum
amount. 91. Average age of a group of 30 people is 24 years. If two
(a)
1
(b)
1
(c)
1 persons of age 20 year and 30 year left the group.
50
1
22
1
66 When a new person joins the group and average age of
(d) 44 (e) 33 group becomes 25 year. Find age of person.
(a) 32 yrs (b) 45 yrs (c) 55 yrs
84. Manoj invested a sum at x% per annum at C.I. If first (d) 50 yrs (e) 70 yrs
year and second year C.I on that sum is Rs. 845 and Rs.
910 Find Amount invested? 92. When 30% of a no. y is subtracted from x it become 310
(a) Rs. 10985 (b) Rs. 10000 (c) Rs. 13000 and when 50% of y is added to x it becomes 550. Find
(d) Rs. 10900 (e) Rs. 13985 ratio of x to y.
(a) 5 : 3 (b) 4 : 3 (c) 2 : 1
85. In an examination, a candidate obtains 20% marks and (d) 7 : 6 (e) 6 : 5
fails by 75marks while another candidate obtains 55%
93. The total surface area of a right circular cylinder of
marks and passed by 20% of the maximum marks
base radius R and height h is obtained by dividing its
marks. What are the passing marks?
volume by
(a) 275 (b) 175 (c) 225 R Rh R
(a) (b) (c)
(d) 500 (e) 125 4 2(R+h) 2
(R+h) Rh
86. The average age of Amit, Dharam and Ankit at the time (d) 2Rh
(e) 4(R+h)
of the marriage of Dharam was 40 years. After one year
94. Shivam’s expense is 25% more than Dharam’s expense
a child was born to Dharam and after 5 years of and Dharam’s expense is 15% less than Harish’s
marriage the average of all of them is 36 years. What expense. If the sum of the their expense is Rs 4660,
was the age of the bride at the time of marriage? then what would be the Shivam’s expense?
(a) 30 (b) 40 (c) 36 (a) Rs 1360 (b) Rs 1700 (c) Rs 1600
(d) 42 (e) 32 (d) Rs 1156 (e) Rs 1165

65 www.bankersadda.com | www.sscadda.com | www.careerpower.in | Adda247 App


SBI CLERK 2021: The Success Guide
95. Dinesh started a business with investing Rs 12000 and 98. Find no. of words can be formed using letters of a
after some months, Sunny joined with investing Rs English word ‘CIRCUMSTANCES’ so that R and E
9000. At the end of the year, total profit was Rs 8000 occupied first and last position respectively.
and share of Sunny is Rs 1600. Find after how many 13! 13! 11!
(a) (b) (c)
months did sunny joined in the business? 3!2! 6 12
11!
(a) 6 months (b) 4 months (c) 9 months (d) (e) None of these
3!2!2!
(d) 3 months (e) 8 months
99. A shopkeeper has 70 buckets and each bucket contain
96. A motorcyclist covers a distance from point A to point
B at an average speed of 32 kmph and return journey 25 packet of biscuit and each packet has 10 biscuits. If
from point B to point A at an average speed of 60 kmph. shopkeeper reduces no. of buckets by 20 and increase
If he takes 11.5 hours for the whole journey, then find no. of packet by 10. Find percentage change in no. of
the distance between point A and point B. biscuits in each packet. Assume total no. of biscuits are
(a) 480 km (b) 360 km (c) 240 km same?
(d) 180 km (e) 300 km (a) 4% (b) 5% (c) 15%
97. The taxi fare in a city comprise a fixed charge and (d) 10% (e) 0%
variable charge which varies according to the number
100. If ratio of time periods of investment of P and Q is 4:5,
of kilometers travelled. In a journey of 220 kms, a
profit at the end of the year is 75000 and P’s share is
person is charged Rs. 1440, while for another journey
of 360 kms, he is charged Rs. 2280. Then find the fare Rs 15000, then what is the ratio of Q’s and P’s
for a journey of 150 kms. investment?
(a) Rs. 1080 (b) Rs. 960 (c) Rs. 1020 (a) 5:16 (b) 6:7 (c) 12:13
(d) Rs. 1060 (e) Rs. 1120 (d) 16:5 (e) 8:5

Solutions
4 4
4 of25 5 3 1 of25 3 1
1. (c): ( 5
48
) ÷ (4 of32 + 7 of 21) =? of 49 6. (e): (5 64 ) ÷ (432 − 202 + 7 of 21) × (82) =? of 64
24 25 1 5 1
( 5 × 48) ÷ (40 + 9) =?× 49 (16) ÷ (432 − 400 + 9) × (82) =?× 64
5 5 5 1
? = 49 × = = 2.5 ? = 16 × 41 × 82 × 64 = 40
98 2

2. (b): √? of 6 + 20% of 95 = of 62
1 7. (a): 55% of 900 + 70% of 1050 = ? % of 3000
2 55 70 ?
62 20 × 900 + × 1050 = × 3000
√? of 6 = 2

100
× 95 = 12 100 100 100
2 495 + 735 = 30 ×?
?= 2 = 4
30 ×? = 1230
5 3 9 ? = 41
3. (e): (3 of 6 5 of 11) +?2 = 45
5 33 9
( × × ) +?2 = 45
3 5 11
2
? = 36
? = ±6
4 14 3 4
4. (a): (7 × 5
÷ 2) − (10 of ? ) = 5 − 3
4 14 1 3 11
(7 × 5
× 2) − (10 ×? ) = − 5
4 3 11
− ?= −
5 10 5
? = 10
4 1 27 1
5. (c): 4 5 + 2 15 − 5
= 2 5 ÷ 3 ×?
24 31 27 11 1
5
+ 15 − 5
= 5
× 3 ×?
22 11
= ×?
15 15
?= 2

66 www.bankersadda.com | www.sscadda.com | www.careerpower.in | Adda247 App


SBI CLERK 2021: The Success Guide
8. (b): 73823 − 34156 + 4756 + 6758 − 9849 18. (a):
= 41499 − 160−?
41332 = 41339−?
?= 7
5599 3773 88
9. (d): 1331
× 2036 × 49 =? −62
14 =? −36
? = 50 19. (d):
1 7 20
10. (c): 84 × ÷ 212+? = × 21 −
4 147 21
1 1 20
84 × × +? = 1 −
4 441 21
1 1
+? = 20. (b):
21 21
?= 0
11. (b): Pattern is
0.5 × (2 − 0) = 1
1 × (2 − 0.5) = 1.5
1.5 × (2 − 1) = 1.5
1.5 × (2 − 1.5) = 0.75 21. (b): 40.02% of 601 – 249.97 ≈ ? – 69.98% of 910
0.75 × (2 − 2) = 0 40% of 600 − 250 ≈ ? −70% of 910
12. (d): Pattern is 240 − 250 ≈ ? −637
5 × 3 = 15 ? ≈ 627
15 × 3 = 45 22. (c): 42001 ÷ 60 × 29.95 ≈ ? × 41.99
45 × 3 = 135 42000
× 30 ≈?× 42
135 × 3 = 405 60
405 × 3 = 1215 21000 ≈ 42 ×?
? ≈ 500
13. (e): Pattern is
90 + 6 = 96; 96 + 6 = 102 23. (e): (42.02)2 + (6.98)2 − (27.02)2 ≈ (33.01)2 −?
102 + 6 = 108; 108 + 6 = 114 422 + 72 − 272 ≈ 332 −?
114 + 6 = 120 1764 + 49 − 729 ≈ 1089−?
14. (a): Pattern is ? ≈5
389 − (9 + 0) = 380 24. (a):
699.97 11 121
÷ 207.99 × 77.02 ≈ ?
380 − (9 + 1) = 370 52
700 11 121
370 − (9 + 2) = 359 ÷ × ≈?
52 208 77
359 − (9 + 3) = 347 700
×
208
×
121
≈?
347 − (9 + 4) = 334 52 11 77
? ≈ 400
15. (b): Pattern is addition of prime no.
1+2=3 25. (d):
3+3=6 29.97% of ? + √399.81 ≈ (14.98)2 + 31.99
6 + 5 = 11 30% of ? +√400 ≈ 152 + 32
11 + 7 = 18 30% of ? +20 ≈ 225 + 32
18 + 11 = 29 30% of ? ≈ 237
16. (c): ? ≈ 790
26. (c): Pattern is
−5 × 2 = −10
−10 × 1.5 = −15
−15 × 2 = −30
−30 × 1.5 = −45
17. (e): −45 × 2 = −90
−90 × 1.5 = −135
wrong number is -180 which should be
replaced with – 135

67 www.bankersadda.com | www.sscadda.com | www.careerpower.in | Adda247 App


SBI CLERK 2021: The Success Guide
27. (d): Pattern is II. y 2 + 12y − 45 = 0
5 × 2 = 10 y 2 + 15y − 3y − 45 = 0
10 × 3 = 30 (y − 3)(y + 15) = 0
30 × 4 = 120 y = 3, −15
120 × 5 = 600 Clearly, x ≥ y
600 × 6 = 3600
33. (e): I. 9x 2 + 11x + 2 = 0
3600 × 7 = 25200
9x 2 + 9x + 2x + 2 = 0
wrong number is 3000 which should be
(9x + 2)(x + 1) = 0
replaced with 3600 2
x = − 9 , −1
28. (a): Pattern is
II. 8y 2 + 6y + 1 = 0
−12 + 6 = −6;
8y 2 + 4y + 2y + 1 = 0
−6 + 6 = 0
(4y + 1)(2y + 1) = 0
0 + 6 = 6; 1 1
6 + 6 = 12 y = − ,−
2 4
12 + 6 = 18; Clearly, no relation can be established
18 + 6 = 24
34. (c): I. 6x² + 5x + 1 = 0
wrong number is 2 which should be replaced
6x 2 + 3x + 2x + 1 = 0
with 0
(3x + 1)(2x + 1) = 0
29. (d): Pattern is 1
x = −3,−2
1

599 − (9 + 0) = 590
II. 4y 2 – 15y = 4
590 − (9 + 1) = 580
4y 2 − 16y + y − 4 = 0
580 − (9 + 2) = 569
(4y + 1)(y − 4) = 0
569 − (9 + 3) = 557 1
557 − (9 + 4) = 544 y = −4,4
544 − (9 + 5) = 530 Clearly, x < y
wrong number is 591 which should be replaced
35. (c): I. x 2 + 3x = 0
with 590
x(x + 3) = 0
30. (b): Pattern is x = 0, −3
700 + 10 = 710 II. x 2 + y = 10
710 − 30 = 680 y = 10 − x 2
680 + 10 = 690 if x = 0, y = 10
690 − 30 = 660 if x = −3, y = 10 − (−3)2 = 1
660 + 10 = 670 Clearly, x < y
670 − 30 = 640
(alternate addition of 10 and subtraction of 30 36. (c): I. x 2 − 25x + 156 = 0
is followed in the given pattern) x2 -12x -13x +156=0
wrong number is 675 which should be replaced x(x-12)-13(x-12)=0
with 680 (x-12)(x-13)=0
x =12,13
31. (e): I. x 2 + 5x + 6 = 0 II. y 2 − 29y + 210 = 0
x 2 + 3x + 2x + 6 = 0 y2-14y -15y +210=0
(x + 3)(x + 2) = 0 y(y-14) -15(y-14)=0
x = −2, −3 (y-14)(y-15)=0
II. y 2 + 9y + 14 = 0 y=14,15
y 2 + 7y + 2y + 14 = 0 So,x<y
(y + 2)(y + 7) = 0
y = −2, −7 37. (d): I. x 2 = 196
Clearly, no relation can be established x =√196
x =±14
32. (b): I. x 2 − 18x + 45 = 0
II. y = √196
x 2 − 15x − 3x + 45 = 0
y =14
(x − 3)(x − 15) = 0
So,x ≤ y
x = 3,15

68 www.bankersadda.com | www.sscadda.com | www.careerpower.in | Adda247 App


SBI CLERK 2021: The Success Guide
38. (e): I. x + 12x + 35 = 0
2 45. (c): Sandeep’s average income in starting four
x2+5x+7x+35=0 months
x(x+5)+7(x+5)=0 =
(8+12+16+20)
× 15000
(x+5)(x+7)=0 4×100
= Rs. 2100
x= -5,-7
Sandeep’s average income in Last four months
II. y 2 + 14y + 48 = 0 (16+20+20+24)
y2+ 6y+8y+48 =0 = 4×100
× 15000 = Rs. 3000
y(y+6)+8(y+6)=0 Required difference = 3000 – 2100 = Rs.900
(y+8)(y+6)=0
y = -8,-6 46. (c): Total cars sold by showroom Q in February and
So, no relation. March together= 270 +380
= 650
39. (a): I. 3x² + 23x + 30 = 0 Total cars sold by showroom R in February and
3x2 +18x +5x +30 =0 March together =390 +410
3x(x+6) +5(x+6)=0 =800
(3x+5)(x+6)=0 650
5 Required percentage = 800 × 100
x= -6,−
3 =81.25%
II. y² + 15y + 56 =0
y2 +8y +7y +56 =0 47. (e): Average numbers of cars sold by the showroom
y(y +8)+7(y+8)=0 P in all months together
380+440+530+290+440
(y+7)(y+8)=0 =
5
y =-7,-8 =416
So, x > y Average number of cars sold by the showroom
40. (c): I. x 2 + 17x + 72 = 0 Q in all the months together
460+270+380+340+510
x2+ 8x+9x+72=0 =
5
x(x+8)+9(x+8)=0 =392
(x+9)(x+8)=0 Required difference = 416 – 392
x = -8,-9 =24
II. y 2 + 13y + 42=0
y2 +6y+7y+42 =0 48. (b): Average number of cars sold by all the 3
y(y+6)+7(y+6)=0 showrooms in March month
530+380+410
(y+6)(y+7)=0 =
3
y = -6,-7 =440
So,x<y
16+24 –8–20
49. (d): Total numbers of cars sold by showroom P in
41. (b): Required% = (16+24)
× 100 March, April and May together
12
= 40 × 100 = 530+290+ 440 =1260
Total number of cars sold by showroom R in
= 30%
January, February and march together
42. (d): Required difference = 320+390+410 =1120
[20+24 –12 –16] 1260
= × 15000 Required ratio = =9:8
100 1120
= Rs. 2400
50. (a): Total cars sold by all the 3 showrooms together
43. (a): It can be seen easily from the pie-chart that in June
February month shows the highest percent 120 125 130
= 530 × 100 + 380 × 100 + 410 × 100
increase in income as compare to previous
= 636+ 475+533
month which is equal to
12 –8 = 1644
= 8 × 100
4 Solutions (51-55):
= × 100
8 Investment of Deepak for first 6 months = Rs. 1500
= 50% increment. Investment of Dharam for first 6 months
2
44. (e): Required central angle = 1500 × 3 = Rs. 1000
18
= (20 + 16) × 5 Investment of Shivam for first 6 months
3
= 129.6° = 1500 × 3 = Rs. 1500

69 www.bankersadda.com | www.sscadda.com | www.careerpower.in | Adda247 App


SBI CLERK 2021: The Success Guide
Investment of Dharam for next 4 months = Rs. 2000 59. (c): Total girls in school A and B
Investment of Deepak for next 4 months = (3000 − 2000) + (2500 − 2000)
2 = 1000 + 500 = 1500
= 2000 × 5 = Rs. 800
Total girls in school B and D
Investment of Shivam for next 4 months = (2500 − 2000) + (3500 − 1500)
3
= 2000 × 5 = Rs. 1200 = 500 + 2000 = 2500
2500−1500
Investment of Shivam for remaining time = Rs. 900 So, required percentage = 2500 × 100
Investment of Deepak for remaining time 1000
4
= × 100 = 40%
2500
= 900 × 3 = Rs. 1200
Investment of Dharam for remaining time 60. (e): Total no. of girls in school A, C and D
3 = (3000 − 2000) + (4000 − 2500) + (3500 −
=900 × = Rs. 900 1500)
3
Profit share of Deepak, Dharam and Shivam = 1000 + 1500 + 2000 = 4500
(1500 × 6 + 800 × 4 + 1200 × 2): (1000 × 6 + Required difference
2000 × 4 + 900 × 2): (1500 × 6 + 1200 × 4 + 900 × 2) = (2000 + 2500) − (4500) = 0
⇒ 73: 79: 78 2000+2400+1800+2500 8700
61. (d): Required average = =
Let profit of Deepak, Dharam and Shivam are Rs.73x, Rs. 4 4
79x and Rs.78x respectively. = 2175
ATQ 62. (a): Required percentage
(79x + 78x − 79x − 73x) = 5x = Rs. 450 =
(2000+2250)−(1500+1850)
× 100
(2000+2250)
x = 90 4250−3350 900
Profit share of Deepak = Rs. 6570 = × 100 = × 100
4250 4250
360
Profit share of Dharam = Rs. 7110 = = 21.176 ≃ 21%
17
Profit share of Shivam = Rs. 7020
1750+2000+2250+2400 8400
63. (c): Required ratio = =
51. (c): Total investment of Deepak 1200+1350+800+1250 4600

= (1500 + 800 + 1200) = Rs. 3500 = 42: 23


Total investment of Shivam 64. (e): Difference in revenue
= (1500 + 1200 + 900) = Rs. 3600 = (1850 − 1350) × 120 = 500 × 120
3500 = Rs 60,000
Required percentage = 3600 × 100 ≈ 97%
1800
52. (a): profit of Dharam after one year 65. (b): Required percentage = 2000 × 100
= 79 × 90 = Rs. 7110 = 90%

53. (d): required ratio = 800: 900 = 8:9 66. (b): Let speed of bus and car be 6x km/hr and 7x
km/hr respectively
54. (b): required difference So,
= (1500 + 1200 + 900) − (1000 + 2000) (7x – 6x) × 4 = 28
= Rs. 600 x=7
196
1500−1200 Required time = 7×7 = 4 hours
55. (d): Required percentage = × 100
1500
= 20% less

2000 2000
56. (a): required ratio = (4000−2500) = 1500
4
=3
(2500−2000)+(4000−2500)
57. (d): required % = × 100
3000
2000
= 3000 × 100
= 66.67%
2000+2000+2500+2500 9000
58. (b): required average = 4
= 4
= 2250

70 www.bankersadda.com | www.sscadda.com | www.careerpower.in | Adda247 App


SBI CLERK 2021: The Success Guide
67. (e): One day work of Veer = x
1
74. (d): sum left at the end of first year
10
3 day work of Veer =
3 = (400 + 400 × 100) − 200 = Rs. 240
x
1 sum left at the end of 2nd year
One day work of Sameer = (x+4) 10
4 = (240 + 240 × 100) − 64 = Rs. 200
4 day work of Sameer =
x+4 sum paid at the end of 3rd year to clear his debt.
ATQ, 10
3 = (200 + 200 × 100) = Rs. 220
x 15
4 = 16
(x+4) 75. (c): quantity of water and acid in initial mixture
(3x + 12) 16 = 60x 40 60
= 50 × 100 , 50 × 100
48x + 192 = 60x
= 20lit and 30 lit respctively.
x = 16
Quantity of acid left after 30 lit of mixture taken
68. (c): Let C.P. of articles be Rs. 100 each. 3
out= 30 − 30 × 5 = 12 lit.
∴ S.P. of Ist article = Rs. 115
Quantity of water left after 30 lit of mixture
S.P. of 2nd article = Rs. 90 2
Total S.P. = Rs. 205 taken out= 20 − 30 × 5 = 8 lit.
205−200
Overall profit percent = ( 200 ) × 100 = 2.5% Required ratio
60 40
Alternate method = (8 + 50 × 100) : (12 + 50 × 100) = 19: 16
+15%−10% +5%
Required percentage = 2
= 2
= 2.5% 76. (a): let total work = 96 units (multiple of 16)
Let efficiency of Hemant = 4x units/day
69. (a):
Then, efficiency of Manoj and Vikash
= 3x units/day and 2x units/day respectively
ATQ
96
4x + 2x =
16
6x = 6
x=1
96 1
Required time = 150 = 21 3 days.
3×1×
100

77. (b): let speed of trains – A & B are x m/s and y m/s
70. (c): Let length of train A = length of train B = ℓ m respectively.
ATQ, ATQ
ℓ+98 ℓ 120 5x + 5y = 850
⇒ = ×
24 12 100 x + y = 170 ……. (i)
ℓ = 70 m And
[(49+57)–(45+52)] 850
71. (d): Required average = 53 – x − y = 85
45 3
9
= 53 – 45 x − y = 30 …….. (ii)
= 52.80 kg From (i) and (ii)
x = 100 m/s and y = 70 m/s
72. (a): Let speed of boat in still water = 11x Required ratio =
5×100
= 10: 7
Speed of stream = x 5×70

Now ATQ, 13
78. (a): required probability = 52 × 51 + 52 × 51
12 4 3
220 220
(11x–x)
– =1 156+12
11x = 52×51
220 220
– 11x = 1 14
10x = 221
1
(22– 20) = 1
x
79. (c): circumference of circle of radius ‘r’ = 2πr
x=2
So speed of boat in still water = 11 × 2 = 22 = 22√2 × √2
km/hr ⇒ 2πr = 22 × 2
r = 7 cm
73. (b): Word are = ( 3-I, 2-N, 1-T, 2-C, 1-O) 22
Area of circle (πr2 ) = × 7 × 7
8! 7
Required no. of ways = = 1680
3!×2!×2! = 154 cm2

71 www.bankersadda.com | www.sscadda.com | www.careerpower.in | Adda247 App


SBI CLERK 2021: The Success Guide
80. (e): let no of first, second and third type of items are 87. (c): Let total capacity of the tank be 60 units (LCM of
2x, 3x and 4x respectively 15, 60, 10)
Total cost price = 2x × 300 + 3x × 500 + 4x × Now, efficiency of the First, second and third
700 = Rs. 4900x pipe be 4 units/min, 1 units/min and 6
10 5
Total profit = 600x × 100 + 1500x × 100 + units/min respectively.
4 Tank filled in first 10 min = (4 + 1) × 10
2800x × = Rs. 247x
100
247x
= 50 𝑢𝑛𝑖𝑡𝑠
Overall profit percentage = 4900x × 100 ≈ 5% Now, when all the pipe work together, 1 unit of
water will out in every minute from tank.
81. (b): let total time of investment = 60t months
So, 50 units of water will be emptied in 50 min.
Ratio of profit of A, B and C
1 1 1 60
5 × 60t × ∶ 3 × 60t × ∶ 7 × 60t × 88. (c): Downstream speed of boat = 11.2 ×
12 5 15 48
⇒ 25: 36: 28 = 14 km/hr
ATQ 3
Speed of boat = 14 × = 10.5 km/hr
4
(36 − 28)units = Rs. 800 1
Then, 25 units = Rs. 2500 Speed of current = 14 × = 3.5 km/hr
4
42 42
82. (d): let average of seven no. = a Required time = (10.5+3.5)
+ (10.5–3.5)
Then, average of first three no. = a + 11 =3+6
ATQ = 9 hours
3(a + 11) + 89 + a − 8 + a − 5 + a − 29
=7×a 89. (d): Let age of Veer & Sameer six years ago be 3x
6a + 80 = 7a years and 7x years respectively.
a = 80 ATQ—
3𝑥+12 5
So, a + 11 = 91 7𝑥+12
=
9
83. (d): Minimum amount will be when all coins are one- 27x + 108 = 35x + 60
rupee coin 8x = 48
7C 7 × 6 ×5 1 𝑥 = 6 years
Required probability = 22 = 3
= 3𝑥+6
C3 22 ×21×20 44 Required ratio =
7𝑥+6
910 – 845 65 3×6+6
84. (a): Rate of interest (x) = 845
× 100 = 845 × 100 =
7×6+6
9 24
= 7 13 % = 48
Total CI = 910 + 845 = Rs. 1755 =1:2
Let ‘p’ amount was invested 7000
𝑅 𝑇 90. (b): Population of town A = 7 × 8
C.I. = 𝑃 [(1 + 100) − 1]
= 8000
1 2
1755 = 𝑃 [(1 + 13) − 1] After two - year population of town B
6 8
27 = 7000 × ×
1755 = P × 169 5 7
1755 ×169 = 9600
P= = 𝑅𝑠. 10985 9600
27 After two years population of town A = 24 × 25
85. (b): let total marks = 100x = 10000
ATQ Increment in population = 10000 – 8000
20𝑥 + 75 = 55𝑥 − 20𝑥 = 2000
15𝑥 = 75
𝑥=5 91. (c): Let age of person = x
Passing marks = 20𝑥 + 75 = 175 x + 30 × 24 – 20 – 30 = 29 × 25
x = 29 × 25 – (30 × 24 – 20 – 30)
86. (c): sum of age of Amit, Dharam and Ankit at the time x= 55
of marriage = 120 𝑦𝑒𝑎𝑟𝑠
Sum of age of Amit, Dharam, Ankit, Child and 92. (b): Atq,
bride after 5 years of marriage = 180 𝑦𝑒𝑎𝑟𝑠 x − 0.3y = 310 …(i)
So, sum of age of Amit, Dharam and Ankit and x + 0.5y = 550 ..(ii)
bride at the time of marriage Dividing (i) by (ii)
= 180 − (5 + 5 + 5 + 4 + 5) = 156 𝑦𝑒𝑎𝑟𝑠 x − 0.3 y = 310
So, age of bride at the time of marriage
x + 0.5 y = 550
= 156 − 120 = 36 𝑦𝑒𝑎𝑟𝑠

72 www.bankersadda.com | www.sscadda.com | www.careerpower.in | Adda247 App


SBI CLERK 2021: The Success Guide
⇒ 55 (x – 0.3y) = 31 (x+ 0.5y) D=
480
= 240 km
2
= 55x – 16.5y = 31x + 15.5y
⇒ 24x = 32y 97. (c): Let fixed cost be Rs x and varying cost per km be
x 32
= 24 Rs y.
y
So, x + 220y = 1440 ………. (i)
x∶y=4∶3
And x + 360y = 2280 …….. (ii)
93. (b): Total surface area of a cylinder = 2πR(R + h) From these two equations value of x and y are
Volume of the cylinder = πR2 h x = 120, y = 6
Where R = radius of the base required fare = 120 + 6 × 150 = Rs 1020
h = height of the cylinder 11! 11! 11!
πR2 h Rh 98. (c): Required no. of words = 3!2! = 3×2×1×2 =
∴ dividing factor = 2πR(R+h) = 2(R+h) 12

99. (e): Let no. of biscuits in each packet in 50 buckets


94. (b): Let expenses of Shivam, Dharam and Harish be are y.
Rs S, Rs D and Rs H respectively. ATQ.
ATQ 70 × 25 × 10 = 50 × 35 × y
S + D + H = 4660 y = 10
125 100 10−10
100
× D + D + 85 × D = 4660 Required percentage change = × 100 = 0%
10
5 20
D (4 + 1 + 17) = 4660
100. (d): Let ratio of P’s investment and Q’s investment be
D = Rs 1360 x:y
125
So, expense of Shivam = 100 × 1360 = Rs 1700 Therefore, profit will be shared in the ratio 4x:5y
4x
Given, 4x+5y× 75000 =15000
95. (e): Let Sunny invested for T months.
4x 1
Ratio of profit share of Dinesh and Sunny = =
4x+5y 5
12000×12 16
9000×T
= T 20x =4x +5y
Let profit share of Dinesh and Sunny be Rs 16x 16x =5y
and Rs Tx respectively. y : x=16:5
ATQ
16x + Tx = 8000
And Tx = 1600
So, 16x = 6400
x = 400
T=4
Sunny invested for 4 months.
∴ after 8 months sunny joined the business.
96. (c): Let the distance between point A and point B be
D km.
ATQ
D D
11.5 = 32 + 60
23 15+8 23
= D[ ] = D[ ]
2 480 480 v

73 www.bankersadda.com | www.sscadda.com | www.careerpower.in | Adda247 App


SBI CLERK 2021: The Success Guide

Most Important Questions | English Language |


SBI Clerk Prelims 2021
Directions (1-5): Read each sentence to find out if 6. There were a number (a)/ of intellectuals in the
there is any error in it. The error, if any will be in one seminar (b)/ but it was Nisha (c)/ who remained
part of the sentence. The number of that part is the a centre of attraction. (d)/ No error. (e)
answer. If there is no error, the answer is (e).
7. A cell (a)/ is the smallest (b)/ identifiable unit of
1. Mr. Singh later flew with (A)/ Ms. Parly to life and cannot be (c)/ seen with a naked eye. (d)/
Merignac, which Dassault (B)/ Aviation’s Rafale No error. (e)
final assembly (C)/ facility is located (D).
8. A Quarrel arose between the five members (a)/
(a) (B) (b) (C) (c) (D)
and for a time (b)/ it appeared as if the party (c)/
(d) (A) (e) No error
had been heading for a split. (d)/ No error. (e)
2. The meeting between the two leaders,(A)/ which
9. The smuggler yielded (a)/ for the temptation (b)/
follows the Wuhan summit (B)/last April that
and fell into (c)/ the police trap. (d)/No error. (e)
reset ties, are expected (C)/ to enhance bilateral
cooperation (D). 10. Found guilty (a)/ on murder (b)/ the accused was
(a) (B) (b) (C) (c) (D) (c)/ sentenced to death. (d)/ No error. (e)
(d) (A) (e) No error Directions (11-18): Each question below has two blanks.
3. Though the Congress government has claimed There are five pairs of words below the sentence. Each
(A)/ that a “transparent process” was adopted pair is lettered. Choose the pair of words which can be
(B)/ for transfers yet the teachers, (C)/students filled up in the blanks in the sentence in the same order
and their parents are agitated over its impact on so as to complete the sentences meaningfully.
studies(D). 11. With just three months left in state assembly
(a) (B) (b) (C) (c) (D) elections, Rashtriya Swayamsevak Sangh (RSS)
(d) (A) (e) No error Sarsanghchalak Dr Mohan Rao Bhagwat will be on a
4. If India will agree to the provisions (A)/ of 10-day _________ of Rajasthan where he will
_________ in Jodhpur, Nagaur and Jaipur.
Chapter 10 on e-commerce, (B)/ it will mean it
(a) pleasure, stay (b) stay, sleep (c) tour, camp
won’t be allowed to impose data localisation
(d) running, run (e) visit, sleep
(C)/rules on companies looking to do business in
India (D). 12. Jet Airways (India) Ltd _________ it would
(a) (B) (b) (C) (c) (D) _________ its 49% stake in Jet Privilege Pvt. Ltd, the
(d) (A) (e) No error frequent-flyer loyalty programme subsidiary.
(a) walked, sell
5. The Bihar police said that they found the (A)/ (b) slept, eat
sedition case lodged against 49 celebrities,(B)/ (c) asserted, play
who wrote an open letter to Prime Minister,(C)/to (d) said, monetize
be “false” and would take action against the (e) asserted, manages
complainant(D).
13. For the first two decades after the ___________ of the
(a) (B) (b) (C) (c) (D)
Soviet Union, Ukraine was one of the most poorly
(d) (A) (e) No error
governed of the ___________ states.
Direction (6-10): Read each sentence to find out whether (a) rebate, abundant
there is any grammatical or idiomatic error in it. The (b) collapse, successor
error, if any, will be in one part of the sentence. The (c) deduction, untidy
number of that part is the answer. If there is ‘No error’, (d) brotherhood, urging
the answer is (e). (Ignore errors of punctuation, if any.) (e) None of these

74 www.bankersadda.com | www.sscadda.com | www.careerpower.in | Adda247 App


SBI CLERK 2021: The Success Guide
14. Former JNU Students’ Union vice-president 18. The bench said Gandhi had (1)/to the top court
Shehla Rashid, who made her ________into (A)/ observations and findings” in (B)/
mainstream politics six months ago, incorrectly attributed “views, (C)/ the Rafale case
____________that she was quitting electoral (D)
politics in Kashmir. (a) BADC (b) CBDA (c) ACBD
(a) Maiden, declare (d) DABC (e) No arrangement required
(b) debuted, preached Directions (19-23): There are two different sentences
(c) foray, announced with a blank space in each sentence. Choose the word
(d) attempt, tell from the given options which fits appropriately in
(e) None of these. both the blanks adding a proper and logical meaning
15. The Delhi Development Authority (DDA) to the sentences.
_____________the amendments to the Master Plan 19. (I) Heart attacks are __________ by destruction of
for Delhi (MPD-2021) to ________________fitness heart tissue.
centres, gyms and yoga centres, officials said. (II) The nurse __________ the old lady
(a) Pass, inculcate everywhere.
(b) Apologized, included (a) accepted (b) agreed (c) bagged
(c) Approved, include (d) accompanied (e) bathed
(d) Rights, posterior
20. (I) Mosquito buzzing in my ear really
(e) None of these. ____________ me.
16. Considering various (II) Indulging me in conversation on certain topics
___________________received for continuation, __________ me.
fitness centres have been allowed to (a) annoys (b) become (c) begins
_________________, considering their role to make (d) cites (e) brings
the Fit India Movement a success. 21. (I) It would have been __________ at twice this
(a) Applications, receive price.
(b) Representations, continue (II) The higher usage of public transport in
(c) Appeals, make Chennai is because local buses in Chennai are
(d) Approvals, submit more reliable and __________.
(e) None of these. (a) Buddhist (b) Cheaper (c) Burned
Directions (17): In the following questions, a (d) Capital (e) Careful
grammatically correct and meaningful sentence is 22. (I) The Senate __________ the President’s
given which is divided into four parts, (A), (B), (C) and candidate for Secretary of Defense.
(D). You have to arrange the four parts to make a (II) The witness __________ the victim’s account in
contextually and grammatically meaningful sentence the court of law.
(the meaning can be different from the one given in (a) dragged (b) kidnapped (c) deleted
the question). If no such rearrangement is possible (d) confirmed (e) developed
mark (e) as your answer i.e. 'No rearrangement is 23. (I) The mother was greatly concerned not to
possible'. __________ her small child.
17. landfills are seedbeds (A)/ contribute to global (II) The Prime Minister of Greece doesn’t want to
warming (B)/ greenhouse gases, which (C)/ of __________ everyone by postponing the visit to
methane and other (D) India.
(a) ADCB (b) BACD (c) CDBA (a) do (b) drag (c) excuse
(d) DABC (e) No rearrangement is possible (d) fashion (e) disappoint

Directions (18): In each question below a sentence are Directions (24-28): In each of the questions given
given which is divided into five parts. The first part of below a sentence is given with one blank. Below each
the sentence (1) is correct and is given in bold followed sentence FOUR words are given out of which two can
by four parts named A, B, C and D. Rearrange the four fit the sentence. Five options are given with various
parts to make a coherent sentence. The rearranged combinations of these words. You must choose the
combination with the correct set of words which can
sequence of the parts will be your answer.
fit in the given sentence.
75 www.bankersadda.com | www.sscadda.com | www.careerpower.in | Adda247 App
SBI CLERK 2021: The Success Guide
24. At 6.30 everyone gathered for drinks and 29. (I) Minimalist theorists believe that democracy is
__________ in the lounge. valuable
(A) Titbit (II) Democracy enables the peaceful transfer of
(B) Bite power through free and fair elections.
(C) Appetizer (i) Because democracy enables….
(D) Sweetness (ii) Though democracy enables ….
(a) A-B (b) B-C (c) A-D (iii) Whether minimalist theorists …..
(d) A-C (e) B-D (a) Only (i)
(b) Only (iii)
25. They took her resignation from TCS as (c) Both (i) and (iii)
___________ of their suspicions. (d) Both (ii) and (iii)
(A) Flaying (B) Confirmation (C) (e) All of these
Testimony 30. (1) The subjective interpretive reasoning cannot be
(D) Drubbing formulated in mathematical ways.
(a) C-D (b) B-C (c) A-D (2) It seems to me that logic should be unique and
(d) A-C (e) B-D hence quantifiable.
(I) While the subjective interpretive…
26. The situation in Syria, where so many proxies of
(II) The formulation of mathematical and…
so many nations are fighting each other, is very (III) That logic of mathematical ways…
__________. (a) Only (I)
(A) Incessant (B) Complicated (C) Constant (b) Only (II)
(D) Perplexing (c) Both (I) and (III)
(a) B-D (b) A-D (c) A-B (d) Both (I) and (II)
(d) A-C (e) B-C (e) All (I), (II) and (III)

27. Earlier, people used to think that clearing the Directions (31-37): In the following passage there are
English section of the SSC CGL Mains exam was blanks, each of which has been numbered. These
__________, but it isn’t as confirmed by the low numbers are printed below the passage and against
cut-off marks for the English section. each, five options are given. Find the word which fits
(A) Easy (B) Exacting (C) Formidable the blank most appropriately.
(D) Facile Last Saturday, as _________(31) _________ explosions
(a) A-B (b) B-C (c) A-D went off at Aramco’s Khurais oilfield and Abqaiq
(d) A-C (e) B-D processing facility in Saudi Arabia, an all-too-familiar
sense of dread swept over Indian markets and
Direction (28-30): Select the phrase/connector
consumers. At the pump the spectre of rising oil prices
(STARTERS) from the given three options which can came back to ________(32) __________ people while
be used to form a single sentence from the two RBI governor Shaktikanta Das told CNBC-TV18 that
sentences given below, implying the same meaning as India’s current account and fiscal _________(33)
expressed in the statement sentences. _________ could take a hit if oil prices continued to
28. (I) Many of economy’s revival measures are rise.
sensible. Just how long will India allow itself to be held
(II) A wider discussion on the matter would serve _________(34) _________ by the global oil mafia?
That 10 automated drones could disrupt 5% of the
the country well.
global oil supply is a cautionary note on their
(i) Subjected to a wider…..
_________(35) _________ powers but is also about how
(ii) While many of….
oil continues to be the lynchpin around which global
(iii) A wider discussion… business revolves. In the immediate aftermath of the
(a) Only (i) attack, US oil futures spiked 14.7%, the biggest spike
(b) Only (ii) in the last 10 years. It forced US President Donald
(c) Both (i) and (iii) Trump to authorize release of oil from the country’s
(d) Both (ii) and (iii) 645 million barrels of Strategic Petroleum Reserves.
(e) All of these With analysts now saying that even a disruption of a
76 www.bankersadda.com | www.sscadda.com | www.careerpower.in | Adda247 App
SBI CLERK 2021: The Success Guide
few weeks could increase the risk premium in oil 38. (a) hiring
prices, the _________(36) _________ is/are grim for (b) virtualization
India whose dependence on oil imports for meeting its (c) collapse
energy needs is at an alarmingly high 84% and whose (d) affiliation
strategic reserves are the lowest among the countries (e) No replacement required
most ________(37) __________.
39. (a) included
31. (a) vibrant (b) fetter (c) simultaneous (b) dominated
(d) exorcist (e) None of these (c) demonstrated
32. (a) haunt (b) excerpt (c) diatribe (d) departed
(d) invective (e) None of these (e) No replacement required
40. (a) tailored (b) legitimated (c) sequenced
33. (a) deficient (b) deficit (c) defiant
(d) shuffled (e) No replacement required
(d) dispel (e) None of these
41. (a) obligatory (b) dividend (c) zealous
34. (a) detente (b) exhort (c) docile
(d) liability (e) No replacement required
(d) hostage (e) None of these
42. (a) immediate (b) deduction (c) pious
35. (a) conspicuous (b) bristle (c) assiduous
(d) dearth (e) No replacement required
(d) destructive (e) None of these
Directions (43-52): In each of the questions given
36. (a) grudge (b) portents (c) fiasco below four words are given in bold. These four words
(d) enigma (e) None of these may or may not be in their correct position. The
37. (a) grilled (b) impacted (c) frisked sentence is then followed by options with the correct
(d) garbled (e) None of these combination of words that should replace each other
in order to make the sentence grammatically and
Directions (38-42): In the following questions, a
contextually correct. Find the correct combination of
paragraph is given. There are several words which are
the words that replace each other. If the sentence is
highlighted following numbers. These words may or
correct as it is then select option (e) as your choice.
may not be used in their correct sense in the given
paragraph. The numbers provided with these words are 43. Senator Van Hollen was requested (A) when
then followed by five options. These options contain rebuffed (B) permission to visit Srinagar in an
words that can replace the words in the paragraph. You effort (C) to assess the situation (D) on the
need to find the words that can replace the highlighted ground.
words and mark them as your answers. If the word (a) A-B (b) B-C (c) C-D
highlighted is used correctly then mark option (e) i.e. (d) A-C (e) None of these
“No replacement required” as your answer.
44. After the Wuhan changed (A), many things have
The major news story of 23 September 2019 was the (38) Summit (B), altering the circumstances (C)
stiffen of the 178 year old Thomas Cook UK. While surrounding India-China relations(D).
newspaper headlines were (39) authored by the story of (a) A-B (b) B-C (c) C-D
this unexpected collapse, there were smaller articles (d) A-C (e) None of these
tucked in on the side highlighting that Thomas Cook
45. China and India continue(A) to compete and have
India Ltd (TCIL) is not part of Thomas Cook UK and is
a contradictory(B) outlook on many issues (C)
not affected by the collapse.
and civilisational strategic (D).
Now TCIL is at pains to highlight through public notices
and press articles that it is (40) owned by Fairfax of
(a) A-B (b) B-C (c) C-D
Canada since 2012. The brand name, for which they were (d) A-C (e) None of these
reportedly paying a fee of ₹2 crore per year, overnight 46. It has to be acknowledged (A) that since the
turned into a (41) department for TCIL. Even while TCIL Wuhan Summit (B), little has concerned (C) as far
has issued press releases sharing its healthy financials, as India-China relations are changed (D).
its stocks fell by 2.68% at the end of the day’s trading in (a) A-B (b) B-C (c) C-D
the (42) transitive aftermath of the news. (d) A-C (e) None of these

77 www.bankersadda.com | www.sscadda.com | www.careerpower.in | Adda247 App


SBI CLERK 2021: The Success Guide
47. Hopes raised (A) at the Wuhan Summit that the Directions (53-60) Read the following passage and
two evanescent (B) would jointly work together answer the following questions based on the given
(C) on an economic project in Afghanistan have passage. Some of the words are highlighted which
proved to be countries (D). would help you to answer some of the questions
(a) A-B (b) B-D (c) C-D given.
(d) A-C (e) None of these People who have been yearning for major economic
48. The move to cut (A) corporate taxes, for which an reforms from the Narendra Modi government, it
(B) sentiment has already been issued by the (C) seems, will have to wait to have their dreams come
government, is on a par with the(D) ordinance - true. Finance Minister Nirmala Sitharaman on
boosting ‘dream budget’ of 1997. Saturday presented the third round of stimulus
(a) Only D-A measures to resuscitate the struggling economy, but
(b) Only C-B once again these have largely failed to live up to the
(c) Both A-B and A-C initial hype around them. The previous two rounds of
(d) Only B-D the stimulus plan, presented at press conferences held
(e) All are correct by the Minister over the last few weeks, focused
primarily on reviving the automobile sector, boosting
49. The government’s (A) statistical machinery has the confidence of foreign investors who were spooked
begun work on revising the (B) workers that opens by the Budget announcements in July, and improving
up the (C) prospect for an upward revision in the the health of dangerously fragile state-owned banks
wages paid out to (D) indices under the act. by doing everything short of privatising them. This
(a) Only D-A time around the focus has been on helping out the
(b) Only B-D underperforming export and real estate sectors
(c) Both C-B and C-D through piecemeal fiscal reforms. Among other
(d) Only C-D things, Ms. Sitharaman announced a new tax refund
(e) All are correct scheme and greater priority sector lending for the
50. The (A) conservative to win a fifth term in office export sector to incentivise exports. It is expected that
the new tax breaks to the exports sector will cause a
could (B) elude Prime Minister Benjamin
dent of up to ₹50,000 crore to the government’s
Netanyahu of the (C) ambition Likud (D) party.
revenue. Further, external commercial borrowing
(a) Only B-D
norms have been eased to make it easier for Indian
(b) Only A-C
real estate companies to tap funds from abroad, and
(c) Both A-B and A-D
funds worth ₹10,000 crore have also been allocated to
(d) Both (a) and (b)
aid the completion of affordable housing projects.
(e) All are correct
With lack of demand and major supply-side stoppage
51. Banks such as Bank of Maharashtra and Punjab being the primary issues facing exports and real
and Sind Bank that have (A) spread manpower, estate, it is doubtful whether the present measures will
network, and (B) resources thin could be turned be enough to revive these flailing sectors.
into (C) vibrant regional institutions to serve Overall, cutting across all three stimulus rounds
agriculture, trade and (D) commerce. announced till date, the government has been relying
(a) Only D-A (b) Only C-B (c) Both A-C almost entirely on providing fiscal relief, in the form
(d) Only C-D (e) All are correct of tax cuts coupled with a tiny amount of government
spending, to wade through what seems like a
52. Liberal parties have (A) roots faced a unique
structural crisis in the economy. The hope seems to be
‘strategic conundrum’ (B) arising from their (C) that these measures combined with a looser monetary
unique beliefs and movement (D) always. policy stance adopted by the RBI will boost spending
(a) Only D-A and revive growth. This is, however, a far cry from
(b) Only C-B what many expected from a government that
(c) Both C-A and D-B promised radical structural reforms when it rose to
(d) Only C-D power in 2014. Without enacting any major supply-
(e) All are correct side reforms like land and labour reforms that can
78 www.bankersadda.com | www.sscadda.com | www.careerpower.in | Adda247 App
SBI CLERK 2021: The Success Guide
raise potential growth, it is also hard to see how 57. Which of the following facts strengthen the
greater spending can raise growth for very long. The argument that the reforms presented by the
government may believe that the present slowdown, government won’t suffice?
marked by five consecutive quarters of dropping (a) The automobile sector and banking sector are
growth, is merely a cyclical one. But given the size of still under huge debt.
its victory in two consecutive elections, the (b) The export sector is facing an issue lack of
government should aim higher by trying to push supply.
through long-pending structural reforms that can (c) The prominent bottlenecks at the supply-side.
raise India’s growth trajectory to the next level. (d) Both (b) and (c)
53. What steps were taken by the government to (e) None of these
revive the economy as per the given passage? 58. Which of the statement(s) explain the exact
(I) Encouraging the foreign investors. meaning of the phrase “to wade through” as used
(II) Privatising the state owned banks. in the passage?
(III) Resuscitating the automobile sector. (I) The enemy waded through the power and
(a) Only (I) yielded to them.
(b) Only (II)
(II) The officials had to wade through pages of
(c) Both (I) and (II)
legal jargon before we could sign the contract.
(d) Both (II) and (III)
(III) Wading through waist-high water is quite a
(e) All (I), (II) and (III)
workout.
54. What would be the effect of the tax refund scheme (a) Only (I)
as per the given passage? (b) Only (III)
(a) The products produced will be of inferior (c) Both (I) and (III)
quality. (d) Both (II) and (III)
(b) The GDP of the country would be enhanced. (e) All of these
(c) The end users will have to bear the burden and
will have to adjust accordingly. 59. Which of the following words is most similar to
(d) The government revenue would shrunk by the word “coupled” as given in the passage?
huge margin. (a) married (b) conjoined (c) adjourned
(e) None of these (d) prorogued (e) None of these
55. Which of the following reform(s) has/have the 60. Which of the following words is most OPPOSITE
author advised in the passage? to the word “affordable” as given in the passage?
(a) Holding elections with a quantified budget. (a) temperate (b) exorbitant (c) nominal
(b) Providing funds to the schemes like affordable (d) subjugate (e) None of these
housing.
(c) Growth can be increased drastically by Directions (61-70): In the following questions, a
implementing supply-side reforms. sentence is divided into four parts consisting of a
(d) Both (b) and (c) highlighted word in each part. Choose the option
(e) None of these reflecting the word which is either misspelt or
grammatically incorrect. If all the highlighted words
56. What can be inferred from the given passage?
(a) The stimulus incentives hold promise, but are correct, choose option (e) i.e. “all are correct” as
structural reforms are nowhere on the horizon your answer choice.
(b) Taking one step forward and two steps back 61. After the unconventional 35 basis points cut in
undermines the legitimacy of policy change intereste rates in August, the Reserve Bank of
(c) While RBI looks at how to create a broad-based India (RBI) returned to a normal 25 basis points
debt market, it should lay the regulatory cut .
groundwork (a) unconventional
(d) A ₹50,000-crore package for housing, half
(b) intereste
funded by the Centre, is expected to rescue
(c) Reserve
affordable housing projects that haven’t yet
(d) returned
gone bust
(e) All are correct
(e) None of these
79 www.bankersadda.com | www.sscadda.com | www.careerpower.in | Adda247 App
SBI CLERK 2021: The Success Guide
62. Hitting out at the Modi gorment over the “massive 67. Indian stoke markets slumped in afternoon trade
joblessness” and the dwindling economy, Mr. with Sensex falling over 700 points at day's low.
Gandhi said the “GDP growth was not visible” (a) stoke (b) slumped (c) afternoon
anymore. (d) falling (e) All are correct
(a) gorment (b) joblessness (c) dwindling
68. The review provided a detailed overview of the
(d) visible (e) All are correct
synthesis, properties and applications of
63. The President commuted death sentences to life nanoparticles (NPs) exist in different form.
impresonment in at least 20 cases over the past (a) review (b) detailed (c) applications
nine years, based on the recommendations (d) exist (e) All are correct
received from the Ministry of Home Affairs 69. India began its campaign in the ICC World Test
(MHA). Championship on a rousing note a few weeks ago,
(a) commuted overwhelming the West Indies 2-0 in the
(b) imprisonment Caribbean.
(c) recommendations (a) began
(d) Ministry (b) campaign
(e) All are correct (c) rousing
64. The Supreme Court sort a response from the CBI (d) overwhelming
to a petition of former Union Finance Minister P. (e) All are correct
Chidambaram challenging a Delhi High Court 70. Few people today would credit Mahatma Gandhi,
order that rejected his plea. whose 150th birth aniversary falls today, with
(a) sort (b) petition (c) challenging great rigour in economic thinking.
(d) rejected (e) All are correct (a) people (b) credit (c) aniversary
65. Hong Kong’s embatled leader invoked colonial- (d) rigour (e) All are correct
era emergency powers for the first time in more Directions (71-75): Two columns are given in each
than 50 years in a dramatic move intended to question and each column has three parts of a
quell escalating violence. sentence. Choose the most suitable pair, which makes
(a) embatled (b) emergency (c) dramatic a grammatically correct and contextually coherent
(d) quell (e) All are correct statement.
66. The shadow bank may also announce a buyback 71. COLUMN I
of its oversees bonds next week, and plans to fund (A) The Delhi Cabinet approved a
the trustee account on its Masala bond. Bill to set up “India’s first” sports university in
(a) shadow (b) announce (c) oversees Delhi,
(d) trustee (e) All are correct (B) Making the announcement, the CM said that
the university will come up in 90 acres of land
(C) A passionate sportsperson can now devote all
his time to playing and excelling in sports
COLUMN II
(D) emphasis on sports and to groom the students
for a career in sports.
(E) which will offer graduation, post graduation
and doctorate degrees in different sports.
(F) then it will be passed in the winter session of
the Assembly.
(a) A-E (b) B-E (c) C-F
(d) Both (a) and (b) (e) None of these

80 www.bankersadda.com | www.sscadda.com | www.careerpower.in | Adda247 App


SBI CLERK 2021: The Success Guide
72. COLUMN I 75. COLUMN I
(A) They were joined by students in a protest (A) Though Mr. Narbir’s name did not figure in
march through the first list of 78 candidates,
(B) The protesters were against a recent circular by (B) The most surprising exclusion was Haryana
the university PWD Minister Rao Narbir Singh,
(C) The teachers have been protested lack of (C) Though ticket was denied to Mr. Inderjit
promotions over the last 10 years, Singh’s daughter Arti Rao Singh
COLUMN II COLUMN II
(D) will have major implications on the quality of (D) ticket to his daughter and supporters could
teaching. proved harmful.
(E) of over 3,000 teachers are due for years now. (E) it was expected that he could be fielded from
(F) which allows only guest appointments against Rewari or Kosli.
full-time regular vacancies. (F) had openly opposed him in Lok Sabah
(a) A-E (b) B-F (c) C-F election.
(d) C-D (e) None of these (a) A-E (b) B-F (c) C-F
(d) C-D (e) None of these
73. COLUMN I
(A) Now every working professional here Directions (76-80): In the following questions a
(B) I am happy that from now on the officials sentence is given, some parts of the sentence are given
working here will be in bold which may have grammatical or spelling error
(C) The initiative will ensure working women or they may be contextually incorrect. Two columns
spend quality time with the (A) and (B) are given, column (A) consists of bold
COLUMN II parts of the sentence and column (B) consists of the
(D) children which will create a favourable appropriate replacement for the bold parts. Match the
correct replacements. If the sentence is correct mark no
atmosphere for both.
error as your answer.
(E) who has a child at home and wants to spend
quality time with him or her can do so. 76. By launching a trade war against China, the
(F) able to stay closer to their children, visit them, United States government that had pressured
feed them and play with them. many a country to liberalise trade and globalise
(a) A-E (b) B-F (c) C-D seems to have turned against its own agenda
(d) A-E, B-F and C-D (e) None of these COLUMN (A) COLUMN (B)
(1) against (4) on
74. COLUMN I
(2) many a country (5) many country
(A) The Grey Line is a standard-gauge section and
(3) agenda (6) perspective
out of the 4.295 km, 2.57 km is elevated
(a) 1-4 (b) 3-6 (c) 2-5
(B) The Grey Line of Delhi Metro, which will
(d) (a), (b) and (c) (e) No error
connect Najafgarh to the rapid transit network,
(C) The Dwarka-Najafgarh corridor of Delhi 77. The U.S. justifies its actions against China by
Metro will be formally citing that country’s significance as a source of
COLUMN II inadequate reciprocated imports into the U.S.
(D) inspecting the corridor for safety on Imports from China account for more than a fifth
Wednesday. of aggregate U.S. imports
(E) the Delhi Metro network will expand to 377 COLUMN (A) COLUMN (B)
km with 274 stations. (1) citing (4) siting
(F) will be opened for services on Friday, officials (2) inadequate (5) inadequately
said. (3) aggregate (6) aggregating
(a) A-E (b) B-F (c) C-F (a) 1-4 (b) 2-5 (c) 3-6
(d) C-D (e) None of these (d) 2-5 and 3-6 (e) No error

81 www.bankersadda.com | www.sscadda.com | www.careerpower.in | Adda247 App


SBI CLERK 2021: The Success Guide
78. Bhutan’s unequivocal support to India on (E) For example, would create plastic-free zones to
strategic issues had meant a lot to India on the begin with—around heritage sites.
international stage and at the United Nations. (F) Prime Minister Narendra Modi told the UN
COLUMN (A) COLUMN (B) General Assembly that India was embarking on a
(1) unequivocal (4) unequivocally “very large campaign" to get rid of single-use
(2) issues had meant (5) issues has meant plastic.
(3) on the (6) form the 81. Which of the following should be the FIRST
international stage international stage sentence after the rearrangement?
(a) 1-4 (b) 3-6 (c) 2-5 (a) A (b) B (c) C
(d) 1-4 and 2-5 (e) No error (d) F (e) E
79. The fourth national tiger survey has generated 82. Which of the following should be the LAST
much euphoria, whereas the first one in 2006 had sentence after the rearrangement?
casted a pall of gloom (a) A (b) B (c) F
COLUMN (A) COLUMN (B) (d) D (e) E
(1) has generated (4) generates 83. Which of the following should be the SECOND
(2) euphoria (5) euthanasia sentence after the rearrangement?
(3) had casted (6) had cast (a) A (b) F (c) C
(a) 1-4 (b) 2-5 (c) 3-6 (d) D (e) E
(d) (b) and (c) (e) No error
84. Which of the following should be the FOURTH
80. These methods for tiger monitoring could sentence after the rearrangement?
estimate numbers using ‘distance sampling’ and (a) A (b) B (c) C
the extent of tiger habitat employing ‘occupancy (d) D (e) E
sampling of tiger spoor’.
85. Which of the following should be the THIRD
COLUMN (A) COLUMN (B) sentence after the rearrangement?
(1) could (4) is to (a) A (b) B (c) C
(2) sampling (5) sampled (d) D (e) E
(3) habitat (6) habbitat
(a) 1-4 (b) 3-6 (c) 2-5 Directions (86-90): Given sentences are not in their
exact position. Rearrange them to make a coherent
(d) 1-4 and 2-5 (e) No error
paragraph and then answer the questions given
Directions (81-85): Given sentences are not in their below.
exact position. Rearrange them to make a coherent (A) SBI suffered in that year simply because its bad
paragraph and then answer the questions given loans ate up much of the profits.
below. (B) It has a business model that rests on making
(A) The government, however, has since clarified that individuals take equate monthly installments to
it would rather spread awareness about the fund their desires and lifestyle purchases.
menace of plastic. (C) Bajaj Finance, on the other hand, is no troubled
(B) In the absence of sufficient alternatives to this NBFC.
ubiquitous and cheap material, an outright ban (D) In FY19, Bajaj Finance generated a net profit of
would have caused much disruption across the ₹3,995 crore, far higher than SBI’s ₹862 crore.
country. (E) Apart from bad loans, SBI’s book is dominated by
(C) Many assumed that for the campaign to be corporate loans, which also means that in times of
effective, New Delhi was preparing to ban the use stress, the bank will face exponential increase in
of disposable plastic. delinquencies compared with others simply
(D) While users of some flexible items such as carry because of the size of its exposures to troubled
bags can easily switch to slightly more expensive NBFCs.
material, as many already have, those of hard- 86. Which of the following should be the FOURTH
plastic products, such as disposable syringes, sentence after the rearrangement?
would have found an overnight switch-over (a) A (b) B (c) C
difficult to achieve. (d) D (e) E
82 www.bankersadda.com | www.sscadda.com | www.careerpower.in | Adda247 App
SBI CLERK 2021: The Success Guide
87. Which of the following should be the SECOND Direction (94-96): Which of the following phrases (I),
sentence after the rearrangement? (II), and (III) given below each sentence should replace
(a) A (b) B (c) C the phrase printed in bold letters to make the sentence
(d) D (e) E grammatically correct? Choose the best option among
the five given alternatives that reflect the correct use
88. Which of the following should be the THIRD
sentence after the rearrangement? of phrase in the context of the grammatically correct
(a) A (b) B (c) C sentence. If the sentence is correct as it is, mark (e) i.e.,
(d) D (e) E "No correction required" as the answer.

89. Which of the following should be the FIRST 94. In the current knowledge-driven economy, the
sentence after the rearrangement? higher education system playing a major role in
(a) A (b) B (c) C social development and national economic
(d) D (e) E competitiveness.
90. Which of the following should be the FIFTH (I) plays a major role
sentence after the rearrangement? (II) has played a major role
(a) A (b) B (c) C (III) may play a major role
(d) D (e) E (a) Only (I) is correct
Directions (91-93): In the questions given below, there (b) Both (I) and (II) are correct
is a sentence in which one part is given in bold. The (c) Both (II) and (III) are correct
part given in bold may or may not be grammatically (d) All are correct
correct. Choose the best alternative among the four (e) No correction required
given which can replace the part in bold to make the
sentence grammatically correct. If the part given in 95. India is in transition towards a knowledge-based
bold is already correct and does not require any economy and its competitive edge determined
replacement, choose option (e), i.e. “No replacement from the abilities of its people to create, share and
required” as your answer. use knowledge more effectively.
91. To overlook the strides India has made since 1947 (I) will be determined by the abilities
for miss the lessons of history. (II) can be determined by the abilities
(a) is to miss the lessons (III) have to be determined from the abilities
(b) on miss the lessons (a) Only (I) is correct
(c) to miss the lessons (b) Both (I) and (II) are correct
(d) about miss the lessons (c) Both (I) and (III) are correct
(e) No replacement required (d) All are correct
92. Atal Bihari Vajpayee was a poet for a pluralist (e) No correction required
approach to politics.
96. The success of the cadets from the Boys Sports
(a) poet with a pluralist approach
(b) poet and had a pluralist approach Company of the Army Sports Institute, Pune, at
(c) poet who possessed a pluralist approach the recent ‘Khelo India’ school games has indeed
(d) Options (a), (b) and (c) a welcome.
(e) No replacement required (I) will be indeed a welcome
(II) is indeed welcomed
93. When he went out he left the radio on so that his
parents shall think that he was still in the house. (III) should be welcomed
(a) might think that (a) Only (I) is correct
(b) will thought that (b) Both (I) and (II) are correct
(c) should have been thought that (c) Both (II) and (III) are correct
(d) would thought that (d) All are correct
(e) No correction required (e) No correction required

83 www.bankersadda.com | www.sscadda.com | www.careerpower.in | Adda247 App


SBI CLERK 2021: The Success Guide
Directions (97-100): In the following question, four (d) The Delhi Cabinet on Thursday approved a
sentences are given which may be grammatically and Bill to set up “India’s first” sports university in
contextually correct. You need to find the one which Delhi.
has an error and mark that as your answer. If all the (e) No error
given sentences are correct then mark option (e) i.e.
‘no error’ as your answer. 99. (a) The performance of the cricket team in the last
series was fairly ordinary and below stature.
97. (a) To argue that someone is born great requires
believing in astrology or unproven fantasies (b) ‘The Delhi Sports University Bill’ approved by
such as reincarnation. the Cabinet, grants the university the right to
(b) Punjab Chief Minister Amarinder Singh has establish constituent schools and colleges.
asked the Centre to take up canalisation of the (c) The Chancellor of the Delhi Sports University
State’s three eastern rivers of the Indus water. will be the Lt. Governor.
(c) India is one of the leading rice producing (d) Security has been beefed up across the national
country in the world. capital after police received intelligence inputs
(d) Mexico is most successful in attracting cross on a possible terror strike.
listings.
(e) No error
(e) No error
100. (a) Consumers have the right to reject faulty goods
98. (a) The prevailing system will be result in the
and demand a refund.
failure of the mission, hence it should be
changed.. (b) Glasses help my vision improve and allow me to
(b) Foreign Direct Investment (FDI 1.0) has been see far away.
welcomed in India irrespective of whether or (c) A defect of title or undisclosed liability would
not its equity structure includes Indian public invalidate the sale at any time.
shareholding. (d) Along with a positive attitude and a healthy diet,
(c) The world has undergone a structural change your fitness level play a major role in how you
with the emergence of Internet MNCs such as feel.
Microsoft, Google, Facebook and Twitter. (e) All are correct

Solutions
1. (a): There is a grammatical error in (B) part of the 5. (b): There is a grammatical error in (C) part of the
sentence. Here, use of “which” is incorrect sentence. Here, past perfect tense must have
because “where” is the pronoun which is used been used. When two actions take place in past
for a place not “which”. one after the other, the subsequent action is
Hence, option (a) is correct answer choice. written in past perfect tense.
2. (b): There is a grammatical error in (C) part of the Hence, replace “wrote” with “had written” to
sentence. “Is” must be replaced with “are” to make the sentence grammatically correct.
make the sentence grammatically correct Hence, option (b) is correct answer choice.
because subject of the sentence is “The meeting” 6. (d): 'a centre of attraction' should be replaced with
which is singular. 'the centre of attraction'. Since, 'the' follows the
Hence, option (b) is correct answer choice.
former noun in sentence with the structure
3. (e): There is no error in the given sentence. 'none + of + noun'. For eg: 'The art of living.'
Hence, option (e) is correct answer choice.
7. (d): In place of ‘a’, ‘the’ is used as ‘with the naked eye’
4. (d): There is a grammatical error in (A) part of the is the correct expression.
sentence. In a conditional sentence both the
8. (a): Use ‘among’ in place of ‘between’ because
clauses cannot be in future tense. In this case “if”
clause must be in simple present tense. Hence ‘between’ is used for the comparison of only two
“will agree” has to be replaced with “agrees”. while ‘among’ is used for the comparison of
Hence, option (d) is correct answer choice. more than two.

84 www.bankersadda.com | www.sscadda.com | www.careerpower.in | Adda247 App


SBI CLERK 2021: The Success Guide
9. (b): In place of ‘for’, ‘to’ will be used because 15. (c): Only option (c) can fill both the blanks
preposition ‘to’ is used after ‘yield’. Ex. She grammatically and contextually.
yielded to bribery. Meanings of some words:
10. (b): Use ‘of’ in place of ‘on’ because preposition ‘of’ Inculcate: to make somebody learn or
is used after ‘guilty’. Ex. “guilty of murder, guilty remember ideas, habits, etc. especially by
of theft.” repeating them often
Posterior: positioned at or towards the back
11. (c): The context of the sentence suggests that RSS
Hence, option (c) is correct answer choice.
Sarsanghchalak will be visiting Rajasthan where
he stay there for days. Among the given options, 16. (b): Only option (b) can fill both the blanks
option (c) provides the best alternatives to the grammatically and contextually.
blank and is the correct answer. Hence, option (b) is correct answer choice.
12. (d): The word ‘manages’ for the second blank doesn’t 17. (a): The correct rearrangement of the given parts of
impart complete sense to the sense, but an the sentence will be ADCB. The statement thus
incomplete sense. Jet Airways would manage its formed will be:
49% stake… If the stakes were already belonged “Landfills are seedbeds of methane and other
to Jet Airways, what is the need to state that it
greenhouse gases, which contribute to global
would manages its own stake. That’s already
warming.”
understood.
The option which provides correct and relevant 18. (b): The phrase (C) provides main verb ‘attributed’
alternatives to the blank is the option (d). to the auxiliary verb ‘had’. So, (C) should
13. (b): The statement discusses that after the break up
follow (1). (B) completes the list (“views,
of Soviet Union, Ukraine was one of the poorly observations and findings”) started in (C). So,
governed states that broke from SU. Here, (B) should follow (C). ‘observations and
among the given options, only ‘collapse’ fits in findings’ where? The answer is ‘in the Rafale
contextually in the given blank. In the second case’. So, (D) should follow (B) and (A) should
blank, we can fit in ‘successor’, as it will indicate follow (D).
that the state emerged after the break up of SU. The correct sequence should be CBDA.
Hence, option (b) is the most suitable answer ‘The bench said Gandhi had incorrectly
choice. attributed “views, observations and findings”
Collapse: suddenly fall down or give way. in the Rafale case to the top court.’
Rebate: a deduction or discount on a sum of Hence, option (b) is the correct answer.
money due
Abundant: existing or available in large 19. (d): Bagged [bag, verb] = put (something) in a bag.
quantities; plentiful. Among the given options, the options (a), (b),
Urging: the action of urging someone to do (c) and (e) are completely irrelevant to the
something. context of the sentence.
The option (d) ‘accompanied’ is the correct
14. (c): Only option (c) can fill both the blanks
answer.
grammatically and contextually.
Meanings of some words: 20. (a): The options (b), (c), (d) and (e) are completely
Preached: to say that something is good and irrelevant.
persuade other people to accept it What feeling a person gets when mosquito
Foray: a sudden attack or incursion into enemy buzzes in someone’s ears?
territory, especially to obtain something; a The option (a) is the most relevant word.
raid. Moreover, the option (a) is also relevant for the
Maiden: being the first of its type. second sentence.
Hence, option (c) is correct answer choice. Hence, the correct answer is the option (a).

85 www.bankersadda.com | www.sscadda.com | www.careerpower.in | Adda247 App


SBI CLERK 2021: The Success Guide
21. (b): What could be attributed to the higher usage Hence, the option (b) is the correct answer.
of public transport in Chennai?
26. (a): The hint for the phrase is given by the
Among the given options, ‘cheaper’ travel on
subordinate clause ‘where so many proxies of
local buses is the most relevant option and is
so many nations are fighting each other’. The
the correct answer.
correct adjective which could correctly qualify
The options (a), (c), (d) and (e) are completely
the noun ‘situation’ is ‘complicated’ and
irrelevant.
‘perplexing’.
Hence, the correct answer is the option (b).
The correct answer is the option (a).
22. (d): Among the given options, only option (d)
27. (c): Facile [adjective] and easy [adjective] are
‘confirmed’ is the most appropriate choice and
the correct answer. synonyms.
What could witnesses do to the victim’s Exacting [adjective] and formidable [adjective]
account in the court of law? are too synonyms, but they mean ‘to be
They could ‘confirm’ it. difficult’.
What could the Senate do to the President’s The first part of the sentence before the
candidate for Secretary of Defense? conjunction ‘but’ is showing a contrast to the
The senate could ‘confirm’ it. second part of the sentence after the
The options (a), (b), (c) and (e) are completely conjunction ‘but’. Having ‘a low cut-off marks
irrelevant as per the context of the sentence. for the English section’ means that the English
section of the SSC CGL Mains exam isn’t easy.
23. (e): What, among the options, could be the concern The adjective to be used in the first part of the
of a mother for her small child?
sentence, when the first part of the sentence is
The mother wouldn’t desire to disappoint her
in contradiction to the second part of the
small child.
sentence, is ‘easy’.
Similarly, ‘the Prime Minister of Greece
Among the given options, the options (A) and
doesn’t want to disappoint everyone by
(D) are the most appropriate. Hence, the
postponing the visit to India’ is a sensible,
option (c) is the correct answer.
contextually meaningful and grammatically
correct sentence. 28. (b): Only starter (ii) can be used to frame a
Among the given options, the option (e) meaningful sentence without altering the exact
‘disappoint’ is the most relevant word to fill meaning of the given sentences. However, it is
the blank. not possible to construct a contextual sentence
Hence, the correct answer is the option (e). using the first and third starter as they would
24. (d): Titbit [noun] and Appetizer [noun] are alter the intended meaning. Hence option (b)
synonyms which means ‘the first course of a is the correct choice.
meal. It consists of a small amount of food. (ii) While many of economy’s revival
Bite means an act of biting something in order measures are sensible, a wider discussion on
to eat it. the matter would serve the country well.
Sweetness means the quality of being sweet. 29. (a): Only starter (i) can be used to frame a
Among the given options, options (A) and (C) meaningful sentence without altering the exact
are relevant to the context of the sentence. meaning of the given sentences. However, it is
Hence, the option (d) is the correct answer. not possible to construct a contextual sentence
25. (b): Flay [verb] means ‘to strip the skin off’; using the second and third starter as they
Drub [verb] means ‘defeat, beating or would alter the intended meaning. Hence
thrashing’; option (a) is the correct choice.
Testimony [noun] means ‘confirmation, proof (i) Because democracy enables the peaceful
etc.’ transfer of power through free and fair
Among the given options, options (B) and (C) elections minimalist theorists believe that
satisfy the context of the sentence. democracy is valuable

86 www.bankersadda.com | www.sscadda.com | www.careerpower.in | Adda247 App


SBI CLERK 2021: The Success Guide
30. (a): Among the three starters, only (I) can give a 35. (d): Conspicuous: clearly visible.
meaningful sentence without altering the Bristle: a short, stiff hair on an animal's skin or
intended meaning of the given two statements. a man's face.
The other two options will give inappropriate Assiduous: showing great care and
meanings to the sentence. Hence option (a) is the perseverance.
correct choice. Thus, option (d) is correct answer choice.
(I) While the subjective interpretive reasoning
36. (b): Grudge: a persistent feeling of ill will or
cannot be formulated in mathematical ways, it
resentment resulting from a past insult or
seems to me that logic should be unique and
injury.
hence quantifiable.
Portents: a sign or warning that a momentous
31. (c): Vibrant: full of energy and life. or calamitous event is likely to happen.
Fetter: restrain with chains or manacles, Fiasco: a complete failure, especially a
typically around the ankles. ludicrous or humiliating one.
Simultaneous: occurring, operating, or done at Enigma: a person or thing that is mysterious or
the same time. difficult to understand.
Exorcist: a person who expels or attempts to Hence, option (b) is correct answer choice.
expel a supposed evil spirit from a person or 37. (b): Grilled: subject (someone) to intense
place. questioning or interrogation.
Hence, option (c) is correct answer choice. Impacted: strongly affected by something.
Frisked: skip or leap playfully; frolic.
32. (a): Haunt: (of a ghost) manifest itself at (a place)
Garbled: (of a message, sound, or
regularly. transmission) confused and distorted; unclear.
Excerpt: take (a short extract) from a text. Hence, option (b) is correct answer choice.
Diatribe: a forceful and bitter verbal attack
against someone or something. 38. (c): From the given words, ‘collapse’ fits in the
Invective: insulting, abusive, or highly critical sentence grammatically and contextually. The
following sentence. The hint can be drawn
language.
from the statement following the highlighted
Hence, option (a) is correct answer choice.
word.
33. (b): Deficient: not having enough of a specified
39. (b): To form a grammatically and contextually
quality or ingredient. meaningful sentence ‘dominated’ can be used
Deficit: the amount by which something, to replace ‘authored’. Hence, option (b) is
especially a sum of money, is too small. correct answer choice
Defiant: showing defiance.
Dispel: make (a doubt, feeling, or belief) 40. (e): The highlighted word is correctly used
grammatically and contextually in the given
disappear.
sentence. Hence, option (e) is correct answer
Hence, option (b) is correct answer choice.
choice.
34. (d): Detente: the easing of hostility or strained
41. (d): The word ‘department’ is contextually
relations, especially between countries.
incorrect. Rather ‘liability’ should be used as
Exhort: strongly encourage or urge (someone) the theme of the passage explains that the
to do something. TCIL was using the brand Thomas cook but
Docile: ready to accept control or instruction; was owned by Fairfax. And the brand Thomas
submissive. Cook became a liability for TCIL. Hence,
Hostage: If you say you are hostage to option (d) is correct answer choice.
something, you mean that your freedom to
42. (a): From the given words ‘immediate’ will
take action is restricted by things that you
replace ‘transitive’ to form a grammatically
cannot control.
and contextually correct sentence. Hence,
Thus, option (d) is correct answer choice. option (a) is correct answer choice.

87 www.bankersadda.com | www.sscadda.com | www.careerpower.in | Adda247 App


SBI CLERK 2021: The Success Guide
43. (a): In the given statement, we will notice that the The correct statement thus formed will be:
positions of (C) effort and (D) situation are Hopes raised at the Wuhan Summit that the
correct in context of the sentence. But (A) two countries would jointly work together on
requested and (B) rebuffed have been an economic project in Afghanistan have
incorrectly placed and interchanging them will proved to be evanescent.
make the sentence grammatically and
contextually correct. 48. (d): The given sentence needs the rearrangement
The correct statement thus formed will be: of the words ‘sentiment’ and ‘ordinance’. This
Senator Van Hollen was rebuffed when is because there is use of ‘an’ in front of
requested permission to visit Srinagar in an ‘sentiment’, hence, it is grammatically
effort to assess the situation on the ground. incorrect. Moreover ‘sentiment-boosting’ is
contextually correct. Other words are correct.
44. (a): In the given statement, we will notice that the
Hence, option (d) is correct answer choice.
positions of (C) circumstances and (D)
relations are correct in context of the sentence. 49. (b): The phrase ‘revising the workers’ is
But (A) changed and (B) Summit have been contextually incorrect. Instead it should be
incorrectly placed and interchanging them will ‘revising the indices’. Moreover the pages
make the sentence grammatically and should be paid to ‘workers’. Other to words
contextually correct. are in their correct places. Hence, option (b) is
The correct statement thus formed will be: correct answer choice.
After the Wuhan Summit, many things have
changed, altering the circumstances 50. (b): The use of ‘conservative’ and ‘ambition’ in the
surrounding India-China relations. given sentence is incorrect. It should be ‘The
ambition to win’ and ‘the ambition Likud
45. (c): In the given statement, we will notice that the
party’. So these words need interchanging.
positions of (A) continue and (B)
contradictory are correct in context of the Hence, option (b) is correct answer choice.
sentence. But (C) issues and (D) strategic have 51. (e): The given sentence is grammatically and
been incorrectly placed and interchanging contextually correct. Hence, option (e) is
them will make the sentence grammatically correct answer choice.
and contextually correct.
The correct statement thus formed will be: 52. (a): The words ‘roots’ and ‘always’ need to be
China and India continue to compete and have interchanged to form a grammatically and
a contradictory outlook on many strategic and contextually correct sentence. Hence, option
civilisational issues. (a) is correct answer choice.
46. (c): In the given statement, we will notice that the 53. (e): As per the lines “focused primarily on
positions of (A) and (B) are correct in context reviving the automobile sector, boosting the
of the sentence. But (C) and (D) have been confidence of foreign investors who were
incorrectly placed and interchanging them will spooked by the Budget announcements in
make the sentence grammatically and July, and improving the health of dangerously
contextually correct. fragile state-owned banks by doing everything
The correct statement thus formed will be: short of privatising them.” All the given
It has to be acknowledged that since the statements are true. Hence, option (e) is correct
Wuhan Summit, little has changed as far as answer choice.
India-China relations are concerned.
54. (d): As per the lines “It is expected that the new
47. (b): In the given statement, we will notice that the
tax breaks to the exports sector will cause a
positions of (A) raised and (C) together are
dent of up to ₹50,000 crore to the government’s
correct in context of the sentence. But (B)
revenue.” Option (d) is correct. For options (a),
evanescent and (D) countries have been
incorrectly placed and interchanging them will (b) and (c), they are either not mentioned in the
make the sentence grammatically and passage or are incorrect. Hence, option (d) is
contextually correct. correct answer choice.

88 www.bankersadda.com | www.sscadda.com | www.careerpower.in | Adda247 App


SBI CLERK 2021: The Success Guide
55. (c): As per the lines “enacting any major supply- 59. (b): The meaning of the word “coupled” is “link
or combine (something) with something else”.
side reforms like land and labour reforms that
As per the given words “conjoined” is the
can raise potential growth, it is also hard to see synonym of the given word. Hence, option (b)
how greater spending can raise growth for is correct answer choice.
very long” option (c) is correct. Option (b) is Adjourn: break off (a meeting, legal case, or
the reform already implemented by the game) with the intention of resuming it later.
government. Option (a) is not mentioned in Prorogue: discontinue a session of (a
the passage. Hence, option (c) is correct parliament or other legislative assembly)
answer choice. without dissolving it.
60. (b): The meaning of the word “affordable” is
56. (a): From the given options (a) can be easily
“being within the financial means of most
inferred as the passage we can check that the people”. From the given words “temperate”
government is trying to stimulate the economy and “nominal” are the synonym of the given
but the reforms introduced need more word. “Exorbitant” is the antonym of the
structural architecture. Option (d) is incorrect given word. Hence, option (b) is correct
as there is no mention of quantity of budget answer choice.
allocated for the housing projects. Hence, Subjugate: to bring under one's control by
option (a) is correct answer choice. force of arms.
61. (b): The correct spelling is “INTEREST”.
57. (d): As per the lines “With lack of demand and
major supply-side stoppage being the primary 62. (a): The correct spelling is “GOVERNMENT”.
issues facing exports and real estate, it is 63. (b): The correct spelling is “IMPRISONMENT”.
doubtful whether the present measures will be
64. (a): “sort” in the sentence is used incorrectly. It
enough to revive these flailing sectors.” option must be replaced by “sought” to make the
(b) and (c) are correct. Hence, option (d) is sentence contextually correct. “Sought” which
correct answer choice. is past form of seek means to attempt or desire
to obtain or achieve (something).
58. (d): The meaning if the phrase “wade through” is
Whereas sort means (V) to arrange
“to struggle to do or complete something systematically in groups; separate according to
(usually due to some factor that is slowing type.
down progress).” And as per the statements
65. (a): The correct spelling is “EMBATTLED” which
given (I) is contextually incorrect. As for (II) means involved in or prepared for war,
and (III), these correctly explain the meaning especially because surrounded by enemy
of the given phrase. Hence, option (d) is correct forces.
answer choice.
66. (c): The use of word ‘oversees’ is incorrect
contextually and grammatically. As ‘oversees’
means “supervise (a person or their work),
especially in an official capacity”, which does
not go as per the context of the sentence.
Rather ‘overseas’ should be used. Hence,
option (c) is correct answer choice.
67. (a): The sentence is discussing about the fall in
Sensex. Sensex is related to ‘Stock market’. So
the use of ‘Stoke’ in the sentence is
contextually incorrect.
Stoke means “encourage or incite (a strong
emotion or tendency).”
Thus, correct answer will be (a).

89 www.bankersadda.com | www.sscadda.com | www.careerpower.in | Adda247 App


SBI CLERK 2021: The Success Guide
68. (d): The word ‘exist’ should be ‘existing’ in order 74. (b): Only B-F can make a contextually correct and
to form a grammatically correct sentence. grammatically meaningful sentence. Rest of
Thus, correct answer choice will be (d). the phrases fails to do so either because they
are part of some other sentence or because they
69. (e): All the words used are grammatically and
are independent sentence.
contextually correct. Thus, correct answer will
Sentence thus formed by combining B-F is :
be (e).
The Grey Line of Delhi Metro, which will
70. (c): The word ‘anniversary’ is misspelt. Thus, connect Najafgarh to the rapid transit network,
correct answer will be (c). will be opened for services on Friday, officials
71. (d): Only A-E and B-E can make a contextually said.
correct and grammatically meaningful 75. (a): Only A-E can make a contextually correct and
sentence. Rest of the phrases fail to do so either grammatically meaningful sentence. Rest of
because they are part of some other sentence the phrases fail to do so either because they are
or because they are independent sentence. part of some other sentence or because they are
Sentences thus formed are: independent sentence.
A-E: The Delhi Cabinet approved a Bill to set Sentence thus formed by combining A-E is:
up “India’s first” sports university in Delhi, Though Mr. Narbir’s name did not figure in
which will offer graduation, post graduation the first list of 78 candidates, it was expected
and doctorate degrees in different sports. that he could be fielded from Rewari or Kosli.
B-E: Making the announcement, the CM said
that the university will come up in 90 acres of 76. (e): The given sentence has no error. ‘Many’ is
land, which will offer graduation, post followed by plural countable noun whereas
graduation and doctorate degrees in different ‘many a’ is followed by countable noun in
sports. singular.

72. (b): Only B-F can make a contextually correct and 77. (b): In phrase ‘inadequate reciprocated imports’
grammatically meaningful sentence. Rest of ‘imports’ which is a noun qualified by
the phrases fail to do so either because they are reciprocated (an adjective). To qualify an
part of some other sentence or because they are adjective an adverb is required so inadequate
independent sentence. must be replaced by inadequately.
Sentence thus formed by combining B-F is: 78. (c): Past perfect tense cannot be used without
The protesters were against a recent circular by reference to past time. Hence, option (c) is the
the university which allows only guest correct option.
appointments against full-time regular
vacancies. 79. (c): Past and past participle form of cast is cast
only. Hence, option (c) would be the correct
73. (d): All of the options can make a contextually answer.
correct and grammatically meaningful
sentence.
Sentences thus formed are:
A-E: Now every working professional here
who has a child at home and wants to spend
quality time with him or her can do so.
B-F: I am happy that from now on the officials
working here will be able to stay closer to their
children, visit them, feed them and play with
them.
C-D: The initiative will ensure working
women spend quality time with the children
which will create a favourable atmosphere for
both.

90 www.bankersadda.com | www.sscadda.com | www.careerpower.in | Adda247 App


SBI CLERK 2021: The Success Guide
80. (e): No improvement required. will be the next statement in the
rearrangement as it gives an example of the
81. (d): Reading through the lines, it can be easily
decision of the government. Next statement
inferred that the passage formed will be about
will be (B) as it explains why the government
the use of plastic. The first sentence will be (F)
did not ban the plastic. (D) will be the last
as it informs that the PM has started a
statement as it explains what steps the people
campaign to get rid of single use plastic. Next
have already taken. Hence, the rearrangement
sentence will be (C) as it discusses about what
will be FCAEBD. The correct answer choice
the people thought would be the decision of
will be (c).
the government. And then (A) will follow (C)
as it tells the actual thinking of the 84. (e): Reading through the lines, it can be easily
government. (E) will be the next statement in inferred that the passage formed will be about
the rearrangement as it gives an example of the the use of plastic. The first sentence will be (F)
decision of the government. Next statement as it informs that the PM has started a
will be (B) as it explains why the government campaign to get rid of single use plastic. Next
did not ban the plastic. (D) will be the last sentence will be (C) as it discusses about what
statement as it explains what steps the people the people though would be the decision of the
have already taken. government. And then (A) will follow (C) as it
Hence, the rearrangement will be FCAEBD. tells the actual thinking of the government. (E)
The correct answer choice will be (d) will be the next statement in the
82. (d): Reading through the lines, it can be easily rearrangement as it gives an example of the
inferred that the passage formed will be about decision of the government. Next statement
the use of plastic. The first sentence will be (F) will be (B) as it explains why the government
as it informs that the PM has started a did not ban the plastic. (D) will be the last
campaign to get rid of single use plastic. Next statement as it explains what steps the people
sentence will be (C) as it discusses about what have already taken. Hence, the rearrangement
the people though would be the decision of the will be FCAEBD. The correct answer choice
government. And then (A) will follow (C) as it will be (e).
tells the actual thinking of the government. (E)
85. (a): Reading through the lines, it can be easily
will be the next statement in the
inferred that the passage formed will be about
rearrangement as it gives an example of the
the use of plastic. The first sentence will be (F)
decision of the government. Next statement
as it informs that the PM has started a
will be (B) as it explains why the government
campaign to get rid of single use plastic. Next
did not ban the plastic. (D) will be the last
sentence will be (C) as it discusses about what
statement as it explains what steps the people
have already taken. Hence, the rearrangement the people though would be the decision of the
will be FCAEBD. The correct answer choice government. And then (A) will follow (C) as it
will be (d) tells the actual thinking of the government. (E)
will be the next statement in the
83. (c): Reading through the lines, it can be easily rearrangement as it gives an example of the
inferred that the passage formed will be about decision of the government. Next statement
the use of plastic. The first sentence will be (F) will be (B) as it explains why the government
as it informs that the PM has started a
did not ban the plastic. (D) will be the last
campaign to get rid of single use plastic. Next
statement as it explains what steps the people
sentence will be (C) as it discusses about what
have already taken. Hence, the rearrangement
the people though would be the decision of the
will be FCAEBD. The correct answer choice
government. And then (A) will follow (C) as it
will be (a).
tells the actual thinking of the government. (E)

91 www.bankersadda.com | www.sscadda.com | www.careerpower.in | Adda247 App


SBI CLERK 2021: The Success Guide
86. (c): Going through the statements it can be the sequence will be (A) as it tells the reason
inferred that in the passage profits and why SBI suffered the loss. (E) will be the next
business models of Bajaj Finance and SBI is line as it gives another reason as well. (C) will
compared. The first line will be (D) as it be the following statement as it follows (E) as
introduces the passage, it tells about the profits both talk about NBFCs. (B) will follow (C) as it
earned by the entities in FY19. The next line in adds meaning to (C). Hence, the arrangement
the sequence will be (A) as it tells the reason will be DAECB. Thus, correct answer will be
why SBI suffered the loss. (E) will be the next (d).
line as it gives another reason as well. (C) will
90. (b): Going through the statements it can be
be the following statement as it follows (E) as
inferred that in the passage profits and
both talk about NBFCs. (B) will follow (C) as it
adds meaning to (C). Hence, the arrangement business models of Bajaj Finance and SBI is
will be DAECB. Thus, correct answer will be compared. The first line will be (D) as it
(c). introduces the passage, it tells about the profits
earned by the entities in FY19. The next line in
87. (a): Going through the statements it can be
the sequence will be (A) as it tells the reason
inferred that in the passage profits and
business models of Bajaj Finance and SBI is why SBI suffered the loss. (E) will be the next
compared. The first line will be (D) as it line as it gives another reason as well. (C) will
introduces the passage, it tells about the profits be the following statement as it follows (E) as
earned by the entities in FY19. The next line in both talk about NBFCs. (B) will follow (C) as it
the sequence will be (A) as it tells the reason adds meaning to (C). Hence, the arrangement
why SBI suffered the loss. (E) will be the next will be DAECB. Thus, correct answer will be
line as it gives another reason as well. (C) will (b).
be the following statement as it follows (E) as
both talk about NBFCs. (B) will follow (C) as it 91. (a): A sentence is a group of words which has at
adds meaning to (C). Hence, the arrangement least one independent-clause. An
will be DAECB. Thus, correct answer will be independent-clause has at least one subject
(a). and one verb and makes complete sense.
88. (e): Going through the statements it can be In the given sentence, ‘to overlook the strides
inferred that in the passage profits and India has made since 1947’ is the subject. ‘for’
business models of Bajaj Finance and SBI is is a preposition and generally, a noun-phrase
compared. The first line will be (D) as it follows a preposition. The phrase ‘miss the
introduces the passage, it tells about the profits lessons of history’ starts with a verb ‘miss’. A
earned by the entities in FY19. The next line in verb-phrase doesn’t follow a preposition.
the sequence will be (A) as it tells the reason Hence, a preposition (i.e., ‘for’) cannot
why SBI suffered the loss. (E) will be the next precedes the verb ‘miss’, and the part given in
line as it gives another reason as well. (C) will bold needs to be replaced.
be the following statement as it follows (E) as
Also, the given sentence doesn’t have any
both talk about NBFCs. (B) will follow (C) as it
verb-phrase, and so, it is an incorrect.
adds meaning to (C). Hence, the arrangement
Among the options, only option (a) ‘is to’
will be DAECB. Thus, correct answer will be
(e). satisfies the requirement of the sentence.
Replacing ‘is to’ with ‘for’ satisfies the
89. (d): Going through the statements it can be
requirement of the sentence. Now, ‘is’ is a verb
inferred that in the passage profits and
and ‘to’ along with ‘miss’ would make a to-
business models of Bajaj Finance and SBI is
infinitive, and the phrase ‘to miss the lessons
compared. The first line will be (D) as it
introduces the passage, it tells about the profits of history’ would be a to-infinitive phrase.
earned by the entities in FY19. The next line in Hence, option (a) is the correct answer.

92 www.bankersadda.com | www.sscadda.com | www.careerpower.in | Adda247 App


SBI CLERK 2021: The Success Guide
meaning to the sentence. They both make sure
92. (d): The usage of the preposition ‘for’ is incorrect
that the sentence follows the correct grammar
in the context of the sentence. ‘Atal Bihari
structure and the intended meaning of the
Vajpayee’ Ji possessed a pluralist approach to
sentence remains intact. However, the third
politics. Here, the correct preposition to be
expression is not befitting to the sentence as
used would be ‘with’.
the plural verb “have” cannot be used for the
Options (b) and (c) are also correct.
singular subject [India] in this case. Hence
‘Atal Bihari Vajpayee was a poet and had a
option (b) is the correct choice.
pluralist approach to politics.’
‘Atal Bihari Vajpayee was a poet who 96. (c): The use of the article “a” in the highlighted
possessed a pluralist approach to politics.’ portion makes the sentence incomplete as the
For option (b), the phrases ‘was a poet’ and expression suggests no proper meaning.
‘had a pluralist approach to politics’ are Among the given options, both the expressions
predicate for the subject ‘Atal Bihari Vajpayee’. (II) and (III) can be used to replace the existing
In the option (c), the phrase ‘,who possessed a phrase to make the sentence grammatically
pluralist approach to politics’ is a participle- viable. Hence option (c) is the correct choice.
phrase (starting word is a participle (2nd form
of verb) ) and functioning as an adjective for 97. (c): There is an error in the option (c) as with ‘one
the noun (or Subject of the sentence) ‘Atal of the’ plural noun is used. Hence, instead of
Bihari Vajpayee’. ‘country’, ‘countries’ should be used.
Hence, all the options, option (a), (b) and (c), Hence, option (c) is correct answer choice.
are correct, and thus, option (d) is the correct 98. (a): There is an error in the first statement. In (a)
answer. ‘will be result’ is incorrect. The use of ‘be’ with
93. (a): In place of ‘shall’, ‘might’ or ‘should’ will be ‘will’ is not needed.
used because the clause ‘he left the radio’ is in Hence, the correct answer choice is option (a).
past tense and according to this, the purpose of 99. (a): The use of ‘fairly’ is incorrect as words like
clause will also be in past tense.
‘very’ or ‘rather’ should be used as ‘fairly’
94. (d): All the above expressions (I, II and III) can be means ‘moderately’ and is only used in front
used to replace the existing phrase to make the of pleasant adjective.
sentence grammatically correct and logically Hence, option (a) is correct answer choice
meaningful. It is to be noted that the
100. (d): All the sentences are grammatically correct
expression “playing a major role” misses a
and contextually meaningful except for
helping verb to complement the sentence
sentence (d). Phrases such as together with, as
grammatically. If we consider the grammatical
well as, and along with are not the same as
aspect of the sentence, all the three expressions
‘and’. The phrase introduced by ‘as well as’ or
given as alternatives follow the correct
‘along with’ will modify the earlier word, but
grammar syntax that make the sentence
it does not compound the subjects (as the word
contextually meaningful as well. Hence option
‘and’ would do). Therefore, when (the head
(d) is the correct choice. word of) the subject is separated from the verb
95. (b): Read the sentence carefully, it can be verified by expressions starting with words such as
that the given sentence is grammatically ‘along with’, ‘as well as’, and ‘besides’, ignore
incorrect as there is an error in the highlighted these expressions when determining whether
part of the sentence which needs to be replaced to use a singular or plural verb. Thus, “play”
by a correct and meaningful expression. should be replaced by its singular form
Among the three options, the first and the “plays”. Hence, option (d) is the most suitable
second expressions provide a contextual answer choice

93 www.bankersadda.com | www.sscadda.com | www.careerpower.in | Adda247 App


SBI CLERK 2021: The Success Guide

Most Important Questions | Reasoning Ability |


SBI Clerk Prelims 2021
Directions (1-5): Study the information carefully and 8. How is ‘property’ coded in the given language?
answer the questions given below. (a) la (b) ja (c) sa
Eight people P, Q, R, S, T, U, V and W are sitting in a circular (d) ta (e) Cannot be determined
table. Some of them facing towards center and some of 9. What can be the code for ‘senior wing science’?
them facing outside the center but not necessarily in the (a) op ja nm (b) op ta nm (c) ty nm op
same order. (d) sa ta op (e) Cannot be determined
R sits second to the right of U. One person sits between R
and S. T sits third to the right of V. T face opposite direction 10. How is ‘water’ coded in the given language?
of R. T is not immediate neighbor of S and U. V sits second (a) vs (b) rt (c) op
to the left of P, who sits second to the right of Q. Q and P are (d) nm (e) Cannot be determined
not immediate neighbors of S. W and S sit immediate right
Directions (11-15): Following questions are based on the
to each other. R faces the same direction as P. T faces
five three-digit numbers given below.
towards the center.
947, 376, 694, 739, 863
1. Who among the following sits second to the right of V?
(a) P (b) W (c) T 11. If all the digits in each of the numbers are arranged in
(d) S (e) None of these increasing order within the number then, which of the
2. Who among the following sits opposite to T? following number will become the lowest in the new
(a) P (b) U (c) W arrangement of numbers?
(d) T (e) R (a) 947 (b) 863 (c) 739
(d) 694 (e) 376
3. Who among the following sits immediate right of Q?
(a) U (b) T (c) P 12. If all the numbers are arranged in ascending order
(d) W (e) None of these from left to right then, which of the following will be
the sum of all the three digits of the number which is
4. Four of the following five are alike in a certain way
2nd from the right in the new arrangement?
based and from a group, find the one that does not
(a) 18 (b) 19 (c) 15
belong to that group?
(a) Q (b) U (c) T (d) 16 (e) None of these
(d) P (e) S 13. What will be the difference, when third digit of the 3rd
5. How many persons sit between P and S when counted lowest number is multiplied with the second digit of
to the right of P? the highest number and third digit of the 2nd highest
(a) Two (b) Three (c) Four number is multiplied with the second digit of the
(d) Five (e) None lowest number?
(a) 21 (b) 20 (c) 15
Directions (6-10): Study the following information
(d)16 (e) None of these
carefully and answer the questions given below.
In a certain code language, 14. If the positions of the second and the third digits of
‘masked peace night property’ is coded as ‘la ja ta sa’, each of the numbers are interchanged then, how many
‘senior peace property’ is coded as ‘sa ty ta’. even numbers will be formed?
‘masked peace science wing’ is coded as ‘ja sa op nm’ (a) None (b) One (c) Two
‘water save wing’ is coded as ‘op vs rt’ (d) Three (e) Four
6. Which of the following represents the code for ‘night’? 15. If one is added to the second digit of each of the
(a) sa (b) ta (c) la numbers and one is subtracted to the third digit of each
(d) ja (e) None of these number then, how many numbers thus formed will be
7. How is ‘peace’ coded in the given language? divisible by three in new arrangement?
(a) ja (b) sa (c) op (a) None (b) One (c) Two
(d) nm (e) Cannot be determined (d) Three (e) Four

94 www.bankersadda.com | www.sscadda.com | www.careerpower.in | Adda247 App


SBI CLERK 2021: The Success Guide
Directions (16-20): Study the information carefully and 24. Four of the following five are alike in a certain way
answer the questions given below. based on the above arrangement and hence form a
Nine boxes are placed one above others in a stack. Two group. Which of the following does not belong to that
boxes are placed between P and F. Box H is just above Box group?
F. As many boxes placed between box H and box P as many (a) G (b) E (c) P
between box P and Q. Box M is placed above box C and (d) C (e) D
below box D. Box M is placed above box P. three boxes are
25. Who among the following is exactly between P and C?
placed between M and O. Box P is placed fourth from the
(a) G (b) P (c) A
bottom. Box O is placed just above box J.
(d) B (e) None
16. How many boxes are placed between box D and box H?
(a) One (b) Three (c) Two Directions (26-28): Study the following information
(d) More than three (e) None carefully and answer the given questions.

17. Which of the following box is placed just below box M? Six persons U, V, W, X, Y and Z are of different age. W is
(a) Q (b) C (c) D older than X and younger than Z. Y is younger than U.
(d) H (e) None of these Neither U nor Z is oldest. Y is second youngest. Z is not
second oldest. U is older than W. The one who is oldest is
18. Which of the following box is placed just above box H? 65years.
(a) D (b) M (c) C
(d) P (e) None of these 26. Who among the following is second oldest?
(a) V (b) Z (c) W
19. How many boxes are placed above box D?
(d) U (e) None of these
(a) One (b) Two (c) Five
(d) Three (e) Four 27. Who among the following is youngest?
(a) X (b) W (c) U
20. Four of the following five are alike in a certain way
(d)Z (e) None of these
based and from a group, find the one that does not
belong to that group? 28. If Z’s age is 55years, then what may be the age of U?
(a) F-C (b) P-Q (c) J-D (a) 61 years (b) 60 years (c) 51 years
(d) O-D (e) H-P (d) 68 years (e) Either (a) or (b)
Directions (21-25): Study the following information 29. Rakhi is 22nd from the left end of a row of 36 students
carefully and answer the question given below: and Preet is 24th from the right end in the same row.
Eight persons i.e. A, B, C, D, E, F, G and P are sitting in a row How many students are there between them in the
at equidistant. Some of them are facing north while others row?
are facing south but not necessary in same order. (a) 9 (b) 7 (c) 8
(d) 5 (e) None of these
E sits 2nd from one of the end. A sits third to the left of E. No
one sits to the right of F. B is not the immediate neighbor of 30. Doctor Rahul ranked 14th from the top and 19th from
A and E and faces north. Two persons sit between P and B. the bottom of the total doctors in the batch. How many
G sits second to the right of D. G doesn’t sit at any extreme doctors are there in the batch?
end. D faces north and is not immediate neighbor of B. (a) 32 (b) 33 (c) 31
immediate neighbor of C faces same direction. G, P and C (d) 30 (e) None of these
faces same direction but opposite of A. C does not sit
Directions (31-35): In each of the questions below are
immediate left of A.
given some statements followed by some Conclusions. You
21. Who among the following is third to the right of G? have to take the given statements to be true even, if they
(a) D (b) C (c) A seem to be at variance from commonly known facts. Read
(d) B (e) no one all the conclusions and then decide which of the given
22. The immediate neighbours of E are: conclusions logically follows from the given statements
(a) F and G (b) G and P (c) D and G disregarding commonly known facts.
(d) G and A (e) C and A (a) If both conclusions I and II follow.
(b) If either conclusion I or II follows.
23. How many persons are sitting exactly between B and
(c) If neither conclusion I nor II follows.
P?
(d) If only conclusion II follows.
(a) Three (b) Four (c) One
(e) If only conclusion I follows.
(d) Two (e) None
95 www.bankersadda.com | www.sscadda.com | www.careerpower.in | Adda247 App
SBI CLERK 2021: The Success Guide
31. Statements: Directions (36-38): In each of the questions below are
Some Star are moon given some statements followed by some Conclusions. You
Some moon are sun have to take the given statements to be true even, if they
All sun is Helium seem to be at variance from commonly known facts. Read
Conclusion: all the conclusions and then decide which of the given
I: Some star are helium conclusions logically follows from the given statements
II: All sun can never be star disregarding commonly known facts.
32. Statements: (a) If only conclusion I follows.
Only a few Noise are Train (b) If only conclusion II follows.
All Train are Eye (c) If either conclusion I or II follows.
Only a few Eye are Music (d) If neither conclusion I nor II follows.
Conclusion: (e) If both conclusions I and II follow.
I: All Music can be Train
36. Statements:
II: No Noise is music
All Petrol is Diesel
33. Statements: No Diesel is CNG
All Thief are Good No CNG is LPG
Only a few Good are bad Conclusion:
No bad is Brave I: All LPG can be Petrol
Conclusions: II: Some Diesel are not LPG
I. Some Good are not Brave
II. All Thief can be Bad 37. Statements:
Only a few wood are glass
34. Statements: All wood are iron
Some black are swan All glass is steel
All swan are red Conclusion:
No swan are green I: All steel can be Iron
Conclusions:
II: All iron can be glass
I. All green can never be red
II. Some black are not green 38. Statements:
Only tags are subject
35. Statements:
Only a few tags are ear
Some May are April
Some ear are nose
No April is July
Conclusions:
All July are June
I. Some nose can be tags
Conclusions:
I: Some May are June II. All nose can be ear
II: No June is May 39. In a row Ram sits 23rd from the left end of the row and
Aman sits 25th from the right end of the row. If there
are 7 people sit in between Ram and Aman then find
out how many people sit in the row?
(a) 54 (b) 47 (c) 52
(d) 60 (e) Can’t be determined

40. How many pairs of letters are there in the word


“VALUATION” each of which have as many letters
between them (both forward and backward
directions) in the word as they have between them in
the English alphabetical series?
(a) None (b) Two (c) Three
(d) More than three (e) None of these

96 www.bankersadda.com | www.sscadda.com | www.careerpower.in | Adda247 App


SBI CLERK 2021: The Success Guide
Directions (41-45): Study the information carefully and 48. If words are arranged according to the alphabetical
answer the questions given below: series from left to right, which word is third from the
Eight persons are sitting around a square table in such a right end?
way that four-person sits at four corners and one person (a) MEF (b) KSY (c) JFG
sits at each side of the table. Some of them face towards the (d) THY (e) NOE
center and some of them face outside the center. Not more
49. If each letter in the each word is arranged according to
than two persons face the same direction sit together.
R sits third to the right of X, who faces outside the center. V the alphabetical series from left to right, and then first
sits opposite to R. U sits opposite to T and T does not sit at and second letter are interchanged in each word then
the corner of the table. Y sits second to the right of W. Y is how many meaning full word will be formed?
not an immediate neighbor of R. Z sits immediate right of (a) One (b) Two (c) None
the one who sits fourth to the right of U. (d) More than three (e) None of these
41. Who among the following sits immediate right of Y? 50. How many pairs of letters are there in the word
(a) T (b) U (c) V “Collection” (both forward and backward direction)
(d) R (e) None of these
each of which have as many letters between them in
42. How many persons face towards the center? the word as they have between them in the English
(a) Three (b) four (c) Two alphabetical series?
(d) More than four (e) None (a) Two (b) None (c) Three
43. Who among the following sits second to the right of R? (d) More than three (e) One
(a) U (b) W (c) Z
Directions (51-55): Study the information carefully and
(d) X (e) None of these
answer the questions given below.
44. Which of the following statement is true about X?
(a) X sits immediate right of V There are eight employees i.e. A, B, C, D, E, F, G and H who
(b) X sits opposite to W are born in eight different months start from March to
(c) T sits immediate left of X October of a same year. C is born in June. There is only one
(d) Three persons sit between X and Z employee whose birth month is between C and B. E is born
(e) None is true in last month of given data. H is born just before the month
45. How is R related to Y? on which A is born but after C’s born month. F is younger
(a) Third to the right than C but older than A. G is Younger than D.
(b) Fourth to the right
51. Who among the following is younger than A?
(c) Fifth to the right
(d) Second to the right (a) B (b) D (c) F
(e) None of these (d) E (e) H

Directions (46-49): Study the following questions and 52. How many employees are older than F?
answering the questions referring to the word sequence (a) Four (b) Two (c) Three
given below: (d) Five (e) No one
MEF, THY, JFG, KSY, NOE, RXB 53. If G’s born month is interchanged with A’s born month,
46. When first and second letter of each word is then G is older than which among the following
interchanged, then how many meaning full word will employees?
be formed ? (a) H (b) B (c) E
(a) Three (b) Two (c) One (d) F (e) D
(d) Five (e) None of these
54. How many employees born between B and G?
47. If each Consonant is changed to previous letter in the
(a) No one (b) One (c) Two
alphabetical series and each vowel is changed to next
letter in the alphabetical series, then How many word (d) Three (e) Four
contains at least one vowel? 55. Which of the following month D is born?
(a) None (b) One (c) Two (a) August (b) June (c) May
(d) Three (e) None of these
(d) March (e) None of these

97 www.bankersadda.com | www.sscadda.com | www.careerpower.in | Adda247 App


SBI CLERK 2021: The Success Guide
Directions (56-58): Read the following information Direction (64-65): In these questions, relationship
carefully and answer the given questions: between different elements is show in the statements. The
Seven people A, C, G, K, S, M and X are having different ages. statements are followed by conclusions. Study the
K is elder to S but younger to G. The difference between the conclusions based on the given statements and select the
ages of 2nd eldest and 4th youngest is 7 years. only two appropriate answer:
persons are elder to C. G is younger to M who is not the (a) If only conclusion I follows.
eldest. Age of K is not more than 5 years. X is elder to G but (b) If only conclusion II follows.
younger to C. Age of 2nd eldest person is the square of the (c) If either conclusion I or II follows
number which is the age of K. Age difference between X and (d) If neither conclusion I nor II follows.
third youngest person is more than 10 years. (e) If both conclusions I and II follow.

56. Who among the following is the 4th youngest? 64. Statements: S>W=E≥R, F<C<V=N, W>K≥V>L
(a) A (b) K (c) C Conclusion I: F<S II: E>N
(d) S (e) X 65. Statements: Q=V>B≥N>I, G<H=B<M, S<V<K=L
57. What is the age of M? Conclusion I: L>Q II: H>I
(a) 36 (b) 64 (c) 25 Directions (66-70): Study the following alphanumeric
(d) 30 (e) Can’t be determined series carefully and answer the questions given below:
58. What could be the possible age of C?
(a) 28 (b) 30 (c) 15 9O2$NDU%M8E45*A76#R©WD&B3F¥
(d) 22 (e) None of the above 66. How many vowels are present in the above series
Directions (59-60): Read the following information which are immediately followed or preceded by a
carefully to answer the questions that follow: perfect square?
(a) None (b) One (c) Two
Sameer starts from city E travels 25km in East direction to (d) Three (e) More than three
reach city G. He then turns left and travels 22 km to city L.
He then turns left and travels 13 km to city S and stop in 67. Which element is exactly between the elements which
city M after turning left and travelling 13km. are 14th from the left and 16th from the right end?
(a) E (b) A (c) 5
59. In which direction is city M with respect to city G? (d) 6 (e) none of these
(a) North-west (b) South-east (c) North-east
(d) South-west (e) North 68. How many consonants are present in the above series
which are immediately followed by an odd number and
60. What is the shortest distance between city M and city immediately preceded by a symbol?
E? (a) None (b) One (c) Two
(a) 9km (b) 13km (c) 25km (d) Three (e) More than three
(d) 15 km (e) None of these
69. If all the consonants are removed from the above
Direction (61-63): Study the given information and series, then which element is 10th to the right of the
answer the following question. element which is 8th from the left end?
Point T is 5m west of point G. Point G is 7m north of point (a) 3 (b) © (c) #
D. Point D is 5m south of M. Point M is 3m east of point B. (d) 6 (e) None of these
Point B is 8m west of point O. Point O is 9m North of point
H.
61. What is the shortest distance between Point M and
Point H?
(a) √107m (b) 10m (c) 12m
(d) √106m (e) 11m
62. Point D is in which direction with respect to point B?
(a) North-west (b) South (c) East
(d) West (e) South-East
63. What is the shortest distance between Point B and
Point G?
(a) 5m (b) 4m (c) √13m
(d) 13m (e) None of these

98 www.bankersadda.com | www.sscadda.com | www.careerpower.in | Adda247 App


SBI CLERK 2021: The Success Guide
70. How many symbols are between the elements which 78. Statements: B≤L<O>G≥E=R≥S
are 7th from the left end and 9th from the right end? Conclusions:
(a) None (b) One (c) Two I. S>O
(d) Three (e) More than three II. G<B
71. Introducing a girl, a boy says, "She is the daughter of 79. Statements: T≥R=B>L≥E=U>O
my father's only brother-in-law". How is the girl Conclusions:
related to the boy? I. E<T
(a) Sister-in-law (b) Grand-daughter (c) Cousin II. O<L
(d) Daughter (e) Aunt
80. Statements: R=A≥F=I>C, T>R≤Z
Directions (72-75): Study the following information Conclusions:
carefully and answer the question given below- I. Z>F
There are eight members in a family with three married II. I=Z
couples. J is son-in-law of L, who is married to K. N has one Directions (81-85): Study the information carefully and
son and one daughter. M is aunt of S. K is mother-in-law of answer the questions given below.
G, who has no children. R is sister of S.
Seven people P, Q, R, S, T, U and V are living at seven
72. How J is related to R? different floors of the building in such a way that ground
(a) Brother (b) Uncle (c) Grandfather floor is no. 1 and top floor is no. 7. Each of them likes
(d) Father (e) None of these different colours i.e. Red, Blue, green, Yellow, Pink, Orange
73. Who among the following is sister-in-law of G? and White but not necessary in same order.
(a) R (b) N (c) M U lives on an even number floor. One floor is between U and
(d) J (e) None of these the one who likes Red colour. The one who likes White
colour lives one of the floors above P. The one who likes
74. Which of the following statement is not true? blue colour live on the 2nd floor. Q lives above the one who
(a) R is daughter of N (b) N is brother- likes red colour. R lives just below Q. T lives above Q. The
in-law of G (c) J is brother-in-law of M one who likes white colour live on the 5th floor. R does not
(d) S is grandson of K (e) Both (b) and like white colour. Not more than four persons live between
(d) T and V, who lives below S. S likes green colour. The one
75. How M is related to K? who likes yellow colour live just above the one who likes
(a) Daughter-in-law (b) Niece Pink colour.
(c) Daughter
81. Who among the following person lives on the 3rd floor?
(d) Sister-in-law
(a) R (b) S (c) Q
(e) None of these
(d) P (e) None of these
Directions (76-80): In each of the question, relationships
82. How many floors between the one who likes Pink
between some elements are shown in the statements.
colour and the one who likes Blue colour?
These statements are followed by conclusions numbered I
(a) None (b) One (c) Three
and II. Read the statements and give the answer.
(d) Four (e) Two
(a) If only conclusion I follows.
(b) If only conclusion II follows. 83. Who among the following person likes Red colour?
(c) If either conclusion I or II follows. (a) U (b) S (c) T
(d) If neither conclusion I nor II follows. (d) R (e) None of these
(e) If both conclusions I and II follow.
84. Four of the following five are alike in certain way based
76. Statements: G>S≥J>T<P=B≤A from a group, find the one that does not belong to that
Conclusions: group?
I. S>P (a) P (b) S (c) R
II. A>T (d) Q (e)T
77. Statements: E<N≤V=L≤O≤P=A 85. Who among the following person lives at the topmost
Conclusions: floor?
I. P≥N (a) U (b) Q (c) T
II. O<E (d) P (e) None of these

99 www.bankersadda.com | www.sscadda.com | www.careerpower.in | Adda247 App


SBI CLERK 2021: The Success Guide
Directions (86-90): Study the following information 95. Inder ranked 34th from the top and 23th from the
carefully and answer the question given below- bottom among those who passed an exam. 15 boys did
not participate in the exam and 9 failed in it. How many
There are eight people i.e. M, N, O, P, Q, R, S and T who sits boys were there in the class?
around a circular table facing towards the centre. R sits (a) 80 (b) 75 (c) 77
immediate left of the one who actually sits 2nd right of M. S (d) 81 (e) 79
faces the one who sits immediate left of P. There is one Directions (96-100): Study the information carefully and
person sits between P and Q. T is not the neighbor of P. N answer the questions given below:
sits near to M. Q is neighbor of S. T sits immediate left of N
A certain number of persons sit in a linear row. Each of
who is not neighbor of P. them faces north direction. G sits second to the right of F.
86. Who among the following sits immediate right of Q? Four persons sit between G and P. Three persons sit
(a) O (b) S (c) M between P and H. Y sits immediate neighbor of G. As many
(d) R (e) N persons sit between H and M as many between M and Y. H
sits one of the extreme ends of the row. L sits third to the
87. How many person sits between T and M when counted right of M. L sits fourth from one of the extreme ends of the
in clockwise direction with respect to M? row.
(a) One (b) Three (c) Two 96. How many persons sit in the row?
(d) Four (e) No one (a) 11 (b) 13 (c) 12
(d) 14 (e) None of these
88. Who among the following faces N?
(a) R (b) S (c) M 97. Who among the following person sits immediate left of
(d) Q (e) O G?
(a) Y (b) F (c) L
89. If M is related to T and Q is related to P, then R is related (d) Y (e) None of these
to whom among the following? 98. Who many persons sit between H and F?
(a) S (b) M (c) N (a) Four (b) Two (c) More than four
(d) P (e) None of these (d) Three (e) None
90. Who among the following faces the one who sits 2nd 99. Who among the following sits second to the right of
right of T? person, who sits ninth from the right end of row?
(a) M (b) Q (c) O (a) Y (b) F (c) L
(d) P (e) None of these (d) G (e) None of these

91. How many meaningful words can be formed using 1st, 100. If O sits fourth to the left of M then how O is related to
Y?
3rd, 6th and 8th letters of the word “ECONOMIC” using (a) Third to the left
each letter only once in each word? (b) Seventh to the left
(a) One (b) Two (c) Three (c) Sixth to the right
(d) More than Three (e) None (d) Second to the right
(e) None of thes
92. If all the letters in the word “REVOLUTION” are
arranged in alphabetical order from left to right in such
a way that consonants are arranged first followed by
vowels, then how many letters are there in between N
and U after the arrangement?
(a) Six (b) Five (c) Seven
(d) Three (e) None
93. If in the number 39475132, 2 is multiply to each of the
digit which is less than 4 and 3 is subtracted from each
of the digit which is more than 4 and equal to 4 then
how many digits are repeating in the number thus
formed?
(a) None (b) Four (c) One
(d) Three (e) Two
94. What should come in the place of question mark (?) in
the following series based on above arrangement?
BY24 EV21 HS18 ?
(a) JQ16 (b) JK16 (c) IR17
(d) KP15 (e) None of these

100 www.bankersadda.com | www.sscadda.com | www.careerpower.in | Adda247 App


SBI CLERK 2021: The Success Guide

Solutions
Solutions (1-5): Solutions (21-25):

21. (e): 22. (c): 23. (d):


24. (e): 25. (c):
Solutions (26-28):
26. (d):

1. (b): 2. (e): 3. (a): 27. (a):


4. (d): 5. (c):
Solutions (6-10): 28. (e):
Word Code
Masked ja 29. (c): Preet position from Left end = (36+1-24) =13
Peace sa Students between them= (22-13-1) =8
Night la
30. (a): Number of doctors in the class= 14+19-1=32
Property ta
Senior ty Solutions (31-35):
Science nm 31. (c):
Wing op
Water/save rt/vs
6. (c): 7. (b): 8. (d):
9. (c): 10. (e):
32. (e):
Solutions (11-15):
11. (e): 12. (e): 13. (c):
14. (c): 15. (a):
Solutions (16-20):
33. (a):
Box
O
J
D
Q 34. (d):
M
P
C
H 35. (b):
F

16. (d): 17. (e): 18. (c):


19. (b): 20. (c):

101 www.bankersadda.com | www.sscadda.com | www.careerpower.in | Adda247 App


SBI CLERK 2021: The Success Guide
Solutions (36-38): 47. (d): MEF THY JFG KSY NOE RXB
LFE SGX IEF JRX MPF QWA
36. (a):
48. (e): NOE
49. (a): SKY
50. (c):
Solutions (51-55):
37. (a):
Month Employees
March D
April B
May G
38. (e):
June C
July F
August H
September A
39. (e): October E
40. (c):
51. (d): 52. (a): 53. (c):
54. (a): 55. (d):
56. (e): A > M (25) > C > X (18) > G > K (5) > S
Solutions (41-45): 57. (c): A > M (25) > C > X (18) > G > K (5) > S
58. (d): A > M (25) > C > X (18) > G > K (5) > S
Solutions (59-60):

41. (b): 42. (b): 43. (e):


44. (b): 45. (a):
46. (b): SKY, ONE
59. (a):
60. (d): √122 + 92 = 15km
Solutions (61-63):

61. (d): 62. (e): 63. (c):

102 www.bankersadda.com | www.sscadda.com | www.careerpower.in | Adda247 App


SBI CLERK 2021: The Success Guide
Solutions (86-90):
Solutions (64-65):
64. (e): I: F<S(True) II: E>N(True)
65. (e): I: L>Q(True) II: H>I(True)
Solutions (66-70):
66. (c): 67. (c):
68. (b): &B3
69. (a): 70. (d): 71. (c):
Solutions (72-75):
86. (b): 87. (a): 88. (e):
89. (c): 90. (b):
91. (a): COME
92. (c): Original Word- REVOLUTION
After Arrangement- LNRTVEIOOU
93. (d): 94. (d):
95. (a): Number of boys who passed
= (34 + 23 - 1) = 56
72. (d): 73. (b): 74. (b): ∴ Total number of boys in the class
= (56 + 15 + 9) = 80
75. (c):
Solutions (96-100):
Solution (76-80):
76. (b): I. S> P (False)
II. A> T (True)
77. (a): I. P≥ N (True) 96. (c): 97. (c): 98. (c):
II. O< E (False) 99. (e): 100. (e):
78. (d): I. S>O (False)
II. G< B (False)
79. (e): I. E <T (True)
II. O<L (True)
80. (c): I. Z > F (False)
II. I = Z (False)
Solutions (81-85):
Floor Person Colour
7 T Yellow
6 U Pink
5 Q White
4 R Red
3 S Green
2 V Blue
1 P Orange
81. (b): 82. (c): 83. (d):
84. (c): 85. (c):

103 www.bankersadda.com | www.sscadda.com | www.careerpower.in | Adda247 App

You might also like